Tạp chí Epsilon số 2

197 62 0
Tạp chí Epsilon số 2

Đang tải... (xem toàn văn)

Tài liệu hạn chế xem trước, để xem đầy đủ mời bạn chọn Tải xuống

Thông tin tài liệu

Bây giờ tôi sẽ cùng các bạn trở lại với chủ đề ban đầu mà chúng ta đã tạm trì hoãn ngay từ nãy giờ, chủ đề đó là tại sao “Ai và Ky” lại không phải một cuốn sách hay, và tại sao Giáo sư N[r]

(1)

Người viết cho hầu hết khái niệm tưởng trừu tượng có cội nguồn thao tác tốn học cụ thể thơng dụng Người viết cho nâng lên tầm khái niệm, thao tác toán học trở thành công cụ tư thật mạnh mẽ

Phương trình đại số ẩn số Ngơ Bảo Châu

Phương trình

đại số ẩn số

Ngơ Bảo Châu

Bài tốn Frobenius

về đồng xu Trần Nam Dũng

Bài tốn đội nón

Đặng Nguyễn Đức Tiến Khám phá tốn học thơng qua

định lý hình học.

Đào Thanh Oai

Laurent Schwartz

Hà Huy Khoái Giới thiệu đề

Vietnam TST 2015,

bình luận sơ tóm tắt cách giải Trần Nam Dũng

VÀ CÁC

CHUYÊN MỤC KHÁC

Đại số tuyến tính VẠN TUẾ

Nghịch đảo Mưbius Ngơ Quang Hưng T p c h í

online cộng đồng

những n g i y ê u T o n

Tạp

chí

onl

ine

của

cộng

đồng

những

người

yêu

T

(2)

Tạp

chí

onl

ine

của

cộng

đồng

những

người

yêu

T

oán

(3)

Tạp

chí

onl

ine

của

cộng

đồng

những

người

u

T

ốn

Tạp chí online cộng đồng người yêu Toán

EPSILON

Chủ biên: abc TRẦN NAM DŨNG

Biên tập viên: ĐẶNG NGUYỄN ĐỨC TIẾN

Biên tập viên: VÕ QUỐC BÁ CẨN

Biên tập viên: TRẦN QUANG HÙNG

Biên tập viên: LÊ PHÚC LỮ

(4)

Tạp

chí

onl

ine

của

cộng

đồng

những

người

yêu

T

oán

(5)

T p c h í

online

cộng đồng

những n g i y ê u T o n

Tạp

chí

onl

ine

của

cộng

đồng

những

người

yêu

T

oán LỜI NGỎ CHO EPSILON SỐ 2

Ban biên tập Epsilon

Epsilon số mắt đón nhận cách nồng nhiệt bạn đọc Đó niềm động viên lớn lao dành cho Ban biên tập, giúp chúng tơi có thêm lượng, nhiệt huyết để bước tiếp đường mà khơng hẳn có hoa hồng

Epsilon nhận vận tốc ban đầu gia tốc Bé dương Với đóng góp cộng đồng, hy vọng Epsilon giữ nhịp đặt mắt vào ngày 13 tháng chẵn để phục vụ cộng đồng, đem đến ăn tinh thần ý vị sống nhiều ăn

Epsilon mong muốn nhịp cầu để kết nối đối tượng vốn xa cách nhau: lý thuyết thực tiễn, tốn học mơn khoa học khác, giáo viên học sinh, nhà toán học chuyên nghiệp người làm toán nghiệp dư, tốn hàn lâm tốn giải trí, tốn cao cấp tốn sơ cấp Vì thế, Epsilon có hòa quyện viết với nội dung phong cách khác Ban biên tập tôn trọng cách hành văn tác giả mà không áp đặt ý kiến mình, chỉnh sửa tốt hơn, xác

Epsilon số mà bạn cầm tay có 12 viết tác giả đến từ nhiều quốc gia, nhiều thành phần cấp độ chuyên nghiệp Kể từ số này, Epsilon dành trang viết để giới thiệu tiểu sử nhà tốn học tiếng giới, lần viết GS Hà Huy Khoái Loran Schwarz, nhân kỷ niệm 100 năm ngày sinh ông viết thiên tài đoản mệnhEvariste Galoissong hành với viết

(6)

Tạp

chí

onl

ine

của

cộng

đồng

những

người

yêu

T

oán

toán nghiệp dư, kỹ sư Đào Thanh Oai Phương pháp mở rộng sáng tạo định lý hình học cổ điển

Các bạn học sinh yêu tốn chắn tìm nhiều điều bổ ích qua viết toán thi chọn HSG quốc gia (VMO 2015) chọn đội tuyển dự IMO 2015 (Vietnam TST 2015) tác giả Trần Nam Dũng, Nguyễn Tất Thu, Trần Quang Hùng Bài bình luận Nguyễn Văn Lợi (Budapest) Nguyễn Hùng Sơn (Warsaw) đề TST giúp bạn hiểu rõ toán đề thi Đặc biệt số có viết Inequalities, A Journey into Fibonacci and Lucas numbers

của hai tác giả nước Vandanjav Adiyasuren Bold Sanchir đến từ ĐH QG Mơng Cổ Những u tốn học giải trí tiếp tục phiêu lưu kỳ thú vào vương quốc nón đủ màu sắc với người hướng dẫn viên Đặng Nguyễn Đức Tiến (Trento, Italy) Một nhà ảo thuật độc đáo khác Nguyễn Quốc Khánh mắt bạn đọc chuyên mục lý thú bổ ích: Giới thiệu sách

Hy vọng với viết thế, độc giả tìm 10% điều u thích số Như thế, Ban biên tập cảm thấy thật mãn nguyện cho nhiệm vụ hoàn thành

(7)

T p c h í

online

cộng đồng

những n g i y ê u T o n

Tạp

chí

onl

ine

của

cộng

đồng

những

người

yêu

T

oán MỤC LỤC

1 Lời ngỏ 2 Nhân 100 năm ngày sinh Laurent Schwartz

Hà Huy Khoái

3 Phương trình đại số ẩn số

Ngơ Bảo Châu 15

4 Évariste Galois

Lưu Trọng Luân 35

5 Nghịch o Măobius

Ngụ Quang Hng 41

6 Các tốn đội nón

Đặng Nguyễn Đức Tiến 53

7 Bài toán Frobenius đồng xu

Trần Nam Dũng - Nguyễn Tất Thu 73

8 Việt Nam TST 2015

Trần Nam Dũng 89

9 Lời giải bình luận hai hình thi chọn đội tuyển Việt Nam năm 2015

Trần Quang Hùng 97

(8)

Tạp

chí

onl

ine

của

cộng

đồng

những

người

yêu

T

oán

Trần Nam Dũng 115

11 Bất đẳng thức Shapiro 121 12 Phương pháp mở rộng sáng tạo định lý hình

học cổ điển

Đào Thanh Oai 125

13 A journey into Fibonacci and Lucas numbers

V Adiyasuren - B Sanchir 153

14 Toán học mắt ai

(9)

T p c h í

online

cộng đồng

những n g i y ê u T o n

Tạp

chí

onl

ine

của

cộng

đồng

những

người

yêu

T

oán NHÂN 100 NĂM NGÀY SINH

LAURENT SCHWARTZ

Hà Huy Khoái

Hà Nội

Tơi nhà tốn học Tốn học đầy ắp đời tôi

Laurent Schwartz viết lời mở đầu hồi

ký ơng Ơng nói rằng, ngồi tốn học, ơng giành nhiều thời gian đời cho đấu tranh quyền người, quyền dân tộc, ban đầu người Troskit, sau đứng ngồi tất đảng phái! Việt Nam chiếm vị trí quan trọng hoạt động ơng Trong nhiều năm, ơng ln đứng hàng đầu đội ngũ trí thức lớn Phương Tây đấu tranh ủng hộ kháng chiến nhân dân Việt Nam Trong hồi ký dày 500 trang ơng, tìm thấy khoảng 100 trang có nhắc đến Việt Nam Laurent Schwartz sinh ngày tháng năm 1915 Paris Cha ông bác sĩ phẫu thuật, mẹ ông người yêu thiên nhiên, ơng nói, suốt ngày quanh quẩn với mảnh vườn ba đứa Tuổi thơ ông trôi qua êm đềm làng quê Autouillet, mà ông gọi cách trìu mến hồi ký “Khu vườn Eden” Mãi sau này, ơng thường xun trở khu vườn đó, ơng kể lại, định lý hay ông tìm thấy khu vườn Eden

Ngay từ nhỏ, Laurent Schwartz bộc lộ thiên hướng nghiên cứu Nếu hầu hết trẻ em hài lòng với lời giải thích sơ lược bố mẹ chúng hỏi “tại sao?”, cậu bé Laurent khơng Cậu ln địi hỏi lời giải thích cặn kẽ, mà thoả mãn Mẹ cậu lúng túng trước câu hỏi: Tại cắm gậy vào nước thấy cong, nhiệt độ mà khơng khí lúc

(10)

Tạp

chí

onl

ine

của

cộng

đồng

những

người

yêu

T

oán

lạnh hơn, lúc nóng nước, lật úp thìa cà phê khơng hết cà phê, mà cịn dính lại thìa, nhiều câu hỏi khác

Ở lớp tiểu học, Laurent Schwartz học sinh giỏi mơn tốn Ơng nhớ lời thầy Thoridenet, người dạy ơng mơn văn năm lớp nói với mẹ ơng: “Tơi chưa có học sinh nào giỏi môn tiếng Latinh, tiếng Pháp, ngôn ngữ tốn cậu ta chút Tuy vậy, cho dù người ta nói với bà nữa, cậu ta trở thành nhà toán học!” Laurent Schwartz nói rằng, khơng có lời khun ơng thầy dạy văn có lẽ ơng trở thành nhà ngôn ngữ học, nhà toán học! May mắn cho Laurent cậu gặp thầy giáo dạy toán đầy nhiệt tâm, thầy Julien Ơng giải thích cho học sinh cách vui vẻ đơn giản điều kì diệu mơn hình học, mở cho họ giới tốn học mà trước họ chưa biết đến Laurent Schwartz kể, sau suy nghĩ vài ba tuần, ơng định trở thành nhà tốn học Theo ông, thiên hướng có sẵn người ông, trở thành thực nhờ thầy giáo Vì ơng cho rằng, vai trị người thầy tương lai học sinh có ý nghĩa định

Laurent Schwartz thi đỗ vào trường Ecole Normale Supérieure (Paris) năm 1934.Ở Ecole Normale, ông học với giáo sư tiếng thời giờ: Fréchet, Montel, Borel, Denjoy, Julia, Elie Cartan, Lebesgue Hadamard Trong khố đó, ơng với Choquet, Marot ba người xuất sắc

Năm 1937, ông tốt nghiệp đại học Ecole Normale, làm nghiên cứu sinh trường đại học Strasbourg bảo vệ luận án Tiến sĩ năm1943.Giáo sư hướng dẫn luận án ông Valiron, nhà tốn học tiếng thời lý thuyết hàm Vài năm sau, Valiron người hướng dẫn giáo sư Lê Văn Thiêm

Trong năm 1944 ´ 1945 ông giảng dạy khoa Khoa học Grenoble, sau chuyển Nancy, nhận chức giáo sư khoa Khoa học Chính thời gian này, ơng sáng tạo cơng trình tiếng lý thuyết hàm suy rộng

Năm 1953 Laurent Schwartz trở Paris, làm giáo sư

(11)

Tạp

chí

onl

ine

của

cộng

đồng

những

người

yêu

T

oán

1980,rồi làm việc trường Đại học Paris7ba năm, ngày nghỉ hưu năm1983

Cống hiến lớn cho tốn học Laurent Schwartz cơng trình ông lý thuyết phân bố, viết vào năm 40 Những tư tưởng ông theo hướng trình bày lần năm 1948 “Mở rộng khái niệm hàm, đạo hàm, biến đổi Fourier ứng dụng toán học, vật lý” Lý thuyết phân bố mở rộng đáng kể phép tính tích phân vi phân Do nhu cầu Vật lý học, Heaviside Dirac mở rộng phép tính với ứng dụng đặc biệt Tuy nhiên, phương pháp họ, phương pháp tương tự phép tính hình thức khơng xây dựng tảng toán học chặt chẽ Để nghiên cứu họ trở thành lý thuyết thực vật lý học, cần trang bị cho sở tốn học vững Chính Dirac có lần nói:

Khi bạn định xây dựng lý thuyết vật lý, duy nhất mà bạn tin tưởng toán học Laurent Schwartz đã phát triển lý thuyết làm sở cho phương pháp tính tốn nêu vật lý, làm cho phương pháp tìm được ứng dụng rộng rãi lĩnh vực khác nhau.

(12)

Tạp

chí

onl

ine

của

cộng

đồng

những

người

yêu

T

oán

của Laurent Schwartz buổi trao Giải thưởng Fields ngày

30 tháng năm 1950 Harvard mơ tả cơng trình Laurent Schwartz viết năm 1948 sau: “Chúng chắn sẽ trở thành cơng trình kinh điển tốn học thời đại chúng ta Tơi nghĩ rằng, người trích dẫn cơng trình ơng, cũng giống tơi, phải kìm nén niềm phấn khích dễ chịu, để nhìn thấy hài hoà tuyệt vời cấu trúc tính tốn mà lý thuyết dẫn đến, để hiểu tầm quan trọng và ưu việt chúng nhiều phần giải tích cao cấp, như Lý thuyết phổ, Lý thuyết vị, toàn lý thuyết phương trình đạo hàm riêng.

Ngồi giải thưởng Fields, Laurent Schwartz nhận giải thưởng Viện hàn lâm khoa học Paris năm1955, 1964, 1972

Năm 1972 ông bầu làm Viện sĩ Viện hàn lâm Pháp Ông phong tiến sĩ danh dự nhiều trường đại học, có Humboldt(1960),Brussels(1962),Lund(1981),Tel-Aviv(1981),

Montreal (1985) Athens (1993)

Không nhà tốn học tiếng, Laurent Schwartz cịn biết đến trí thức lớn suốt đời đấu tranh tự dân tộc Laurent Schwartz nói rằng, năm Ecole Normale xác định hồn tồn khuynh hướng trị ơng: Chống chiến tranh bảo vệ giá trị người Cuốn sách “Đông Dương cấp cứu” (Indochine SOS) Andrée Viollis cho ông thấy rõ tội ác chủ nghĩa thực dân Pháp Đông Dương Quan điểm trị ơng thể rõ phong trào chống chiến tranh xâm lược đế quốc Mỹ Việt Nam Ông đề xướng hiệu “Mặt trận dân tộc giải phóng chiến thắng” thay cho hiệu mà ông cho mơ hồ phong trào chống chiến tranh Việt Nam Pháp thời “Hồ bình Việt Nam“ Hoạt động Uỷ ban quốc gia Việt Nam ông sáng lập gây tiếng vang lớn Ông tự hào vào khoảng lễ Noel năm 1966, nhận điện cám ơn chúc mừng Chủ tịch Hồ Chí Minh Ơng đến Việt Nam nhiều lần thời kì chiến tranh, với tư cách thành viên Toà án quốc tế xét xử tội ác chiến tranh Mỹ Việt Nam (một tổ chức quốc tế nhà toán học, nhà triết học tiếng người Anh, giải thưởng Nobel văn học năm1950,

(13)

Tạp

chí

onl

ine

của

cộng

đồng

những

người

u

T

ốn

tình cảm ơng với Việt Nam lời ơng viết hồi ký mình: “Việt Nam ghi dấu ấn đời tôi Tôi biết đến Đông Dương thuộc địa, qua sách của André Viollis viết năm 1931, mà tơi đọc năm 1935 Lúc tơi vừa trịn 20 tuổi Cuộc đấu tranh tơi cho tự đất nước này là đấu tranh dài đời Tôi yêu, mãi mãi yêu Việt Nam, phong cảnh, người tuyệt vời, những xe đạp Trong tơi, có chút người Việt Nam Gặp người Việt Nam, nghe tiếng họ nói chuyện với nhau trong xe buýt (mà tất nhiên không hiểu), cảm thấy một niềm hạnh phúc không cắt nghĩa Sợi giây tình cảm nối liền tơi với đất nước này.

Năm 1998, Viện Toán học tổ chức Hội nghị quốc tế nhân 80

(14)

Tạp

chí

onl

ine

của

cộng

đồng

những

người

yêu

T

(15)

T p c h í

online

cộng đồng

những n g i y ê u T o n

Tạp

chí

onl

ine

của

cộng

đồng

những

người

u

T

ốn PHƯƠNG TRÌNH ĐẠI SỐ

MỘT ẨN SỐ

Ngô Bảo Châu

Đại học Chicago, Mỹ

Tóm tắt

Từ kỷ20trước Công nguyên, người dân thành Babylon biết giải phương trình bậc hai Nhưng phải đến kỷ

16sau Cơng ngun, nhà tốn học thời Phục hưng: Tartaglia, Cardano, Ferrari, tìm lời giải cho phương trình bậc ba bậc bốn Đầu kỷ 19, Abel Galois, hai thiên tài toán học bạc mệnh, chứng minh nghiệm phương trình đại số tổng quát bậc từ năm trở đi, biểu diễn biểu thức đại số với thức trường hợp đa thức bậc không bốn Công trình Galois, viết lời trăng trối trước đấu súng, sau xem mốc khai sinh Đại số đại Lý thuyết Galois đại phát biểu sở khái niệm mở rộng trường nhóm Galois Những khái niệm khơng dễ nắm bắt Mục đích viết giúp người học nắm bắt khái niệm đó, thơng qua việc tìm hiểu mơ thức mà chúng xuất trình tìm nghiệm phương trình đại số cụ thể

(16)

Tạp

chí

onl

ine

của

cộng

đồng

những

người

yêu

T

oán

1 Lịch sử toán

Vào kỷ thứ bảy trước công nguyên, lời giải cho phương trình bậc hai tổng quát

x2+ax+b=0, (3.1)

đã nhà toán học Brahmagupta, người Ấn độ, trình bày cách tường minh dạng

x= ´a˘ ?

d

2 , (3.2)

với d=a2

´4blà biệt thức phương trình bậc hai

Trước đó, từ khoảng kỷ20trước cơng ngun, người Babylon tìm lời giải hình học cho tốn tương đương tìm hai cạnh hình chữ nhật biết trước chu vi diện tích Dấu vết phương pháp hình học khác để giải phương trình bậc hai phát hầu hết văn minh cổ đại từ Babylon, Ai cập, Hy lạp, Ấn độ, Trung Hoa

Phương trình bậc ba tổng quát người Babylon nghiên cứu Người Hy lạp cổ đại thử xây dựng nghiệm phương trình bậc ba thước kẻ compa khơng thành cơng Nhà tốn học Trung Hoa Wang Xiaotong đưa lời giải cho 27

phương trình bậc ba khác nhau, khơng đưa phương pháp để giải phương trình bậc ba tổng quát

Đáng kể phát nhà thơ người Ba tư Omar Khayyam sống vào mười Ông chứng minh nghiệm xây dựng nghiệm phương trình bậc ba cách lấy giao hai đường conic Ngồi ra, ơng phát biểu khơng thể xây dựng nghiệm phương trình bậc ba thước kẻ compa Omar Khayyam không đưa công thức cho nghiệm phương trình bậc ba giống cơng thức (3.2) cho phương trình bậc hai

Phải chờ đến thời kỳ phục hưng, nhà toán học Tartaglia, sống Ý vào kỷ thứ mười sáu, đưa công thức tổng quát cho nghiệm phương tình bậc ba

ax3+bx2+cx+d=0, (3.3) dạng

x=´ 3a

b+C+∆0

C

(17)

Tạp

chí

onl

ine

của

cộng

đồng

những

người

u

T

ốn

trong

C= d

∆1 +

a

∆2 1´4∆30

2 , (3.5)

với ∆0, ∆1 đa thức tường minh với biến sốa, b, c, d

Lời giải cho phương trình bậc ba rắc rối, lời giải cho phương trình bậc bốn Ferrari rắc rối nhiều Nhà toán học Joseph Lagrange, người Ý, người đưa phương pháp chung để giải phương trình bậc ba bậc bốn Phương pháp Lagrange dưạ khái niệm giải thức mà xem xét kỹ lưỡng

Ruffini nghiên cứu phương pháp Lagrange nhận thấy khơng thể mở rộng cho phương trình có bậc năm bậc cao

Abel người đưa chứng minh chặt chẽ khẳng định phương trình bậc năm tổng quát giải thức Định lý Abel-Ruffini Galois, nhà toán học người Pháp, chứng minh cách độc lập Nhưng ông xa Abel đưa khái niệm có tính chất cách mạng, nhóm Galois

2 Về phát biểu toán

Bài toán ta quan tâm việc biểu diễn nghiệm phng trỡnh a thc

a0xn+a1xn1 +ă ă ă=0, (3.6)

dưới dạng biểu thức với biến số a0, a1, , mà ta

được quyền dùng bốn phép tốn thơng thường thức Để hiểu rõ biểu diễn dạng biểu thức thế, ta cần khái niệm trường mở rộng trường Ví dụ biểu thức với biến số a0, a1, ,an mà dùng bốn phép tốn thơng thường với hệ số hữu tỉ, trường sinh bởia0, a1, ,an

(18)

Tạp

chí

onl

ine

của

cộng

đồng

những

người

yêu

T

oán

phép ta biểu diễn nghiệm (3.1) Trong đó, cơng thức Tartaglia (3.5) dường cho ta sáu nghiệm khác phương trình bậc ba, rõ ràng khơng thể

Thực ta khơng có cách để chọn nnghiệm phương trình (3.6) Khái niệm nhóm Galois sinh ta để lượng hố mập mờ Ngược lại, ta phân tích, cấu trúc nhóm Galois định việc phương trình (3.6) giải thức hay khơng

3 Mở rộng bậc hai

Để giải phương trình bậc hai (3.1), ta thực phép đổi biến

y =x+ a2 Sau đổi biến, phương trình (3.1) để quy dạng đơn giản

y2 ´d=0 (3.7)

Ta coi mẹo để quy phương trình bậc hai tổng quát (3.1) phương trình bậc hai rút gọn (3.7)

Ta thay đổi quan điểm: Khơng quan tâm đến việc tìm dạng xác (3.2) nghiệm nữa, mà quan tâm đến việc nghiệm biểu diễn dạng biểu thức đại số ?d Lập luận phức tạp hơn, mở cho ta tầm nhìn

Để làm đơn giản vấn đề, giả sử hệ số a,b số hữu tỉ Ta biết C, phương trình (3.1) có hai nghiệm Ta ký hiệu α1 P C hai nghiệm Giả sử α1 R Q,

đó tập số phức có dạng

L=tm+nα1 |m, nPQu,

là không gian vector hai chiều trênQ Từ đẳng thức

α21 =´(aα1+b),

ta suy u, v P L uv P L Ta chứng minh uP L´ t0u, u´1 P L Như L trường

con C Nếu xem không gian vector Q, có chiều 2.Vì ta nói L mở rộng bậc hai Q Ta để ý thấy nghiệm lại, ký hiệu α2, đa thức

(19)

Tạp

chí

onl

ine

của

cộng

đồng

những

người

yêu

T

oán

cũng nằm trongL.Thật vậy, đa thức bậc haiP có nghiệm

α1 PL, nghiệm lạiα2 phải nằm L không

là số hữu tỉ Nói cách khác, mở rộng bậc hai sinh α1, trùng

với mở rộng bậc hai sinh bởiα2

tm+nα2 |m, nPQu

Suy từ (3.2) mở rộng bậc hai sinh α1 hay α2

trùng với mở rộng bậc hai sinh bậc hai biệt thức Q[

?

d] =tm+n?d|m, nPQu (3.8) Đây cách để diễn đạt việc α1 α2

viết dạng có dạng m+?d với m, nPQ

Định lý 3.1. Cho P P Q[x] là đa thức bậc hai bất khả quy,

L là mở rộng bậc hai củaQ sinh nghiệm củaP

Khi đóL=Q[?d]vớid=a2

´4b

Dễ thấy rằng, nếuL mở rộng bậc hai Q,khi phần tử α P L´Q nghiệm phương trình bất khả quy bậc hai Vì ta phát biểu lại định lý dạng cô đọng hơn:

Định lý 3.2. Mọi mở rộng bậc hai củaQ đều có dạng L =Q[?d]

vớidlà số hữu tỉ đó.

Mở rộng phương trình bậc cao hơn, ta định nghĩa rành rọt khái niệm phương trình giải thức

4 Phương trình giải thức

Từ trở đi, ta thay trường số hữu tỉ trường K

bất kỳ Thay cho trường số phức, ta cho trước trường đóng đại số chứaK.Xin nhắc lại trườngKđược gọi đóng đại số đa thức P P K[x] bậc n có n nghiệm K, ta đếm bội Ta xét tới mở rộng K

chứa K

Đa thức bậcn

(20)

Tạp

chí

onl

ine

của

cộng

đồng

những

người

yêu

T

ốn

được gọi làbất khả quynếu khơng thể phân tích thành tích hai đa thức có bậc nhỏ Giả sử P đa thức bậc nbất khả quy Với nghiệm αP Kca P,ta t

K[] =tm0+m1+ă ă ă+mn1n1 |m0, ,mn´1 PKu (3.9)

Sử dụng đẳng thứcαn=

(a1n1+ă ă ă+an),ta chng minh c rng nu u,v P K[α] uv P K[α] Ngồi ra, u P K[α]´ t0u

thì u´1

P K[α] Nói cách khác, K[α] trường K Sử dụng giả thiết P đa thức bất khả quy, ta chứng minh K[α], xem không gian vector trường K, có chiều n.Nói cách khác, K[α] mở rộng bậc ncủa K

Ta nói nghiệm α biểu diễn biểu thức đại số với thức tồn chuỗi mở rộng trường liên tiếp

K=K0 ĂK1 ĂK2 ă ă ă Kr, (3.10) cho vi mi i P t1, 2, ,ru, Ki mở rộng bậc ni

Ki´1 có dạng

Ki »Ki´1[x]/(xni ´βi), cho K[α]ĂKr

Khái niệm mở rộng trường cho phép ta phát biểu rành rọt câu hỏi liệu nghiệm α P biểu diễn dạng tổ hợp đại số thức hay khơng Nó cịn cho phép ta đặt câu hỏi khác, sâu sắc hơn, nghiệm đa thức

5 Phụ thuộc đại số nghiệm

Như trên, ta ký hiệu P P K[x] đa thức bất khả quy bậc n, vàα nghiệm P K,K[α] mở rộng bậc n

của Kbao gồm tổ hợp đại số αnhư (3.9)

Khác với trường hợp bậc 2,khi ně3,nếu α1 vàα2 hai nghiệm

khác P, trường conK[α1] K[α2] K,có thể

khác nhau, ta thấy ví dụ sau

Xét trường hợp K = Q đa thức P = x3 ´2 Nếu α P K nghiệm P hai nghiệm lại làjαvà j2α. Ở ta sử

dụng ký hiệu

j=cos2π

3 +isin 2π

(21)

Tạp

chí

onl

ine

của

cộng

đồng

những

người

yêu

T

oán

là bậc ba nguyên sơ đơn vị Dễ thấy Q[α]‰Q[jα]vì dấu xảy ta có j P Q[α] Mặt khác, mở rộng Q[j] mở rộng bậc2củaQvìjlà nghiệm đa thức bậc haix2+x+1, nằm mở rộng bậc ba Thật vậy, Q[j]ĂQ[α]thì Q[α] khơng gian vector trường Q[j], chiều khơng gian vector Q phải số chẵn Trong trường hợp này, mở rộng bậc ba ứng với nghiệm củaP =x3

´2là đôi khác nhau:

Q[α]‰Q[jα]‰Q[j2α] (3.12) Khi K=Q[j], P =x3

´2vẫn đa thức bậc ba bất khả quy

K[x] Nhưng mở rộng bậc ba K ứng với nghiệm P =x3´2 trùng nhau:

K[α] =K[jα] =K[j2α] (3.13) Ta nhận thấy trường hợp đầu,αvàjαkhông phụ thuộc đại số với so với trường sở K=Q Nói cách khác jαkhơng thể biểu diễn tổ hợp đại số α với hệ số hữu tỉ Tuy vậy, ta mở rộng trường sở thành K=Q[j], α vàjα trở nên phụ thuộc đại số

Ví dụ đưa ta đến với khái niệm trường phân rã đa thức bất khả quy Trường phân rã đa thức công cụ để đo phụ thuộc đại số nghiệm Trường phân rã lớn nghiệm có quan hệ đại số, trường phân rã nhỏ nghiệm có nhiều quan hệ đại số

Đa thức bất khả quy P P K[x] bậc n gọi tách được

nó có n nghiệm đơi khác K Trong trường hợp đặc số không, đa thức bất khả quy tách Trong trường hợp đặc số pą0, có đa thức bất khả quy

không tách Trong này, ta xét đến đa thức bất khả quy tách

Cho P P K[x] đa thức bất khả quy bậc n tách có hệ số đầu Ta ký hiệu α1, α2, ,αn P K nghiệm P, gọi trường phân rã K trường K sinh α1, α2, ,αn Trong vành đa thức L[x], đa thức P phân rã hoàn tồn

(22)

Tạp

chí

onl

ine

của

cộng

đồng

những

người

yêu

T

oán

Để làm rõ ý này, ta thực khảo sát Ký hiệuL trường phân rã P, L trường cực tiểu chứa tất trường K[α1], K[α2] , K[αn], gọi compositum

K[α1], K[α2] , K[αn].Ký hiệuLilà compositum củaK[α1], K[α2]

, K[αn],khi ta có chuỗi mở rộng trường liên tip

K=L0 L1 ă ă ă Ln=L

Ký hiệu li bậc mở rộng Li/Li´1 Trường phân rã L mở

rộng bậc l1l2 .ln K Các số nguyên l1, l2, , ln phản ánh mức độ phụ thuộc đại số nghiệm α1,α2, , αn Ta khảo sát chúng sau

• Vì P đa thức bất khả quy bậc n L1 mở

rộng bậc l1 =n củaL0

• Xét mở rộng L2/L1 Đa thức P xem phần tử

của L1[x] khơng cịn bất khả quy nữa, mà phân tích

được thành

P = (x´α1)Q

Thành phầnQcó thể đa thức bất khả quy, khơng

• Nếu Q đa thức bất khả quy, bậc n´1, L2

một mở rộng bậc l2 = n´1 L1 Trong trường hợp này,

α1 vàα2 khơng có quan hệ đại số với

• Nếu Q P L1[x] đa thức bất khả quy, ta

phân tích thành Q = Q2Q3 với Q2 đa thức bất khả

quy có nghiệm α2 Khi đóL2 mở rộng L1 có bậc l2

bằng với bậc đa thức Q2 Trong trường hợp α1

α2 có phụ thuộc đại số

• Tiếp tục với mở rộng L3/L2

Qua khảo sát ta thy l1 l2 ă ă ă l mt dãy số nguyên

giảm thật từ suy l1ă ă ăln n! Dóy s ny đo mức

độ phụ thuộc đại số nghiệm α1, α2, , αn P

6 Nhóm Galois đa thức

(23)

Tạp

chí

onl

ine

của

cộng

đồng

những

người

yêu

T

oán

Ta ký hiêu L trường phân rã, trường K sinh

α1, α1, , αn

Nhóm Galois ΓL/K nhóm tự đẳng cấu mở rộng L/K : Các tự đẳng cấu mở rộngL/Klà song ánh σ:LĐLbảo tồn cấu trúc vành cấu trúc không gian vector L trường

K Nếu khơng có nguy nhầm lẫn, ta viết giản lược số ngầm hiểu Γ =ΓL/K Để nhấn mạnh phụ thuộc vào P, gọi Γ nhóm Galois đa thứcP

Cho σPΓ vàαPL nghiệm P,khi σ(α)

nghiệm P Vì nhóm Galois Γ tác động lên tập hợp nghiệm P Với ký hiệu chọntα1, ,αnu, nghiệm

P đánh số, tác động củaΓ lên chúng cho đồng cấu nhóm

ρP :Γ ĐΥn vào nhóm hốn vị cấp n

Định lý 6.1. Đồng cấu ρP : Γ Ñ Υn là đơn ánh Tác động của Γ lên tập t1, 2, ,nu là tác động bắc cầu Số phần tử của Γ đúng bằng với bậc mở rộng L/K

Chứng minh khẳng định thứ khơng khó Nếuσnằm hạch ρP, σ(αi) = αi với nghiệm P Trong hoàn cảnh này, σ tác động tầm thường lên toàn L L sinh bởiα1, α1, , αn

Chứng minh khẳng định thứ hai thứ ba khó chút Trước hết ta chứng minh đồng cấu K- đại số ξ:LÑK

đều có ảnh L Thật đồng cấu bảo toàn tậptα1, α1, , αnu,mà tập sinh raL,cho nênξ(L) =L.Như ta chứng minh

AutK(L) =HomK L,K

(3.14) Để chứng minh khẳng định thứ ba, ta cần chứng minh

|HomK L,K

|=degK(L) (3.15) Một mở rộng hữu hạn L K gọi mở rộng tách được thoả mãn tính chất

Nếu L = K[x]/P với P P K[x] đa thức bất khả quy bậc n

(24)

Tạp

chí

onl

ine

của

cộng

đồng

những

người

yêu

T

oán

hợp này, cho đồng cấu σ : L ÑK tương đương với cho σ(x)

là nghiệm P K có n nghiệm khác

Có thể chứng minh compositum mở rộng tách K1, K2, , Kn với K Ă Ki Ă K mở rộng tách Vì thế, trường phân rã đa thức tách mở rộng tách

Ta quay lại chứng minh khẳng định thứ hai: Tác động Γ

lên tập nghiệm tα1, α2, , αnu tác động bắc cầu Ta chứng minh tồn σPAutK(L) cho σ(α1) =α2 Trước hết ta

có đẳng cấuσ:K[α1]ĐK[α2] cho bởiα1 ÞĐα2 α1 vàα2 có

đa thức cực tiểu P Ta cần chứng minh σ thác triển thành tự đẳng cấu σ:LÑL

Để kết thúc chứng minh, ta cần sử dụng thêm tính chất mở rộng tách được: NếuL/Klà mở rộng tách được, tồn β P L phần tử sinh L Nói cách khác tồn

βPL cho L=K[β].2

Mở rộng L/K[α1] mở rộng tách được, tồn

β1 PL cho L=K[α1][β1].Ta phát triển

σ:K[α1]ĐK[α2],

thành đồng cấu có dạng

σ:K[α1][β1]ĐK[α2][β2],

với β2 PK lựa chọn thích hợp Sử dụng (3.14), ta có

K[α2][β2] =L,

và σlà tự đẳng cấu Lnhư ta mong muốn

Ta tóm tắt thông tin mục sau Mỗi đa thức bất khả quyPứng nhóm GaloisΓ.Nếu nghiệm

P K đánh số coiΓ nhóm nhóm đối xứng cấp n, có tác động bắc cầu lên tập t1, 2, ,nu

Số phần tử Γ bậc trường phân rã

(25)

Tạp

chí

onl

ine

của

cộng

đồng

những

người

yêu

T

oán

7 Tương ứng Galois

Như ta thấy mục trước, L trường phân rã đa thức bất khả quy tách P PK[x], mở rộngL/K có tính chất số phần tử nhóm AutK(L) với bậc mở rộng Mở rộngL/Kđược gọi mở rộng Galois tính chất thoả mãn Trường phân rã đa thức bất khả quy luôn mở rộng Galois

Mở rộng L/K gọi mở rộng tách tồn đa thức bất khả quy P PK[x]tách choL»K[x]/(P),tất nhiên có

thể có nhiều đa thức P Dễ thấy L=K[x]/(P)là mở rộng tách mở rộng Galois, L trường phân rã P Nói cách khác mở rộng Galois trường phân rã đa thức Mặt khác, đồng thời trường phân rã nhiều đa thức khác

Không phải mở rộng mở rộng Galois Quay lại ví dụ (3.12) mở rộng bậc ba Q[α] Q, với α nghiệm

x3´2.Nếu σPAut

Q(Q[α]) σ(α) phải nghiệm

x3´2.Theo (3.12) σ tự đẳng cấu Q[α] trừ trường hợp σ=1

Nói cách khác Q[α]khơng phải biểu diễn Galois

Ngược lại, theo (3.13) K[α]/K mở rộng Galois với K = Q[j]

với j nguyên sơ bậc ba đơn vị (3.11) Tổng quát

Định lý 7.1. Nếu xn´a P K[x] là đa thức bất khả quy tách được, nếu K chứa nguyên sơ bậcn của đơn vị, với mọi nghiệm αP K của đa thức xn´a, mở rộng L = K[α] của K mở rộng Galois.

Các mở rộng trung gian K L phân loại dựa vào Γ Đây thường coi mệnh đề quan trọng lý thuyết Galois, gọi tương ứng Galois Vấn đề quan trọng ln ln bàn cãi

Định lý 7.2. Mỗi mở rộng trung gianKĂL1 ĂLứng với nhóm

con Γ1 của Γ bao gồm phần tử γ P Γ tác động lên L như một

ánh xạ L1-tuyến tính Tương ứng ngược lại cho bởiL1 =LΓ1

Hơn nữa,L1 là mở rộng Galois củaKkhi khi Γ1 là nhóm con

chuẩn tắc củaΓ.3

3Nhóm conΓ1 ĂΓ được gọi nhóm chuẩn tắc với mọi γP Γ,ta

(26)

Tạp

chí

onl

ine

của

cộng

đồng

những

người

yêu

T

ốn

Trong trường hợp đó, ta cóΓL1/K =Γ/Γ1

Trong mục 5, ta khảo sát trường phân rã L đa thức bất khả quy P PK[x] thông qua chuỗi mở rộng liên tiếp

K=L0 L1 L2 ă ă ă Ln=L,

với Li compositum K[α1], K[α2], , K[αi] Chuỗi mở rộng tương ứng với chuỗi nhúm ca

=0 ă ¨ ¨ ĄΓn=0 (3.16) Có thể chứng minh rằngΓi =ΓXΥn´ivới mọii =1, 2, , n, đóΥn´i nhóm phần tử củaΥncố định phần tử 1, 2, , i tậpt1, 2, , nu

Các nhóm Γ1, Γ2, nói chung khơng phải nhóm

chuẩn tắc Γ, L1, L2, mở rộng Galois

của K Trong mục sau, mục 8, khảo sát kỹ trường hợp chuỗi mở rộng Galois

8 Tiêu chuẩn để giải phương trình bằng thức

Xét ví dụ cho chuỗi mở rộng thức (3.10)

K=K0 K1 ă ă ă Kr, (3.17) vi KiằKi1[x]/(xnii).Gi thit K chứa nguyên sơ đơn vị bậc n1, n2, , nr Khi với i, Ki mở rộng Galois Ki´1, từ ta suy Kr mở rộng Galois K Ký hiệu Γi = ΓKr/Ki, ta có chuỗi nhóm chuẩn tắc sau:

=0 ă ă ă

với

Γi´1/Γi=ΓKi/Ki´1 »Z/niZ

Nói cách khác, nhóm Γ nhóm giải

(27)

Tạp

chí

onl

ine

của

cộng

đồng

những

người

yêu

T

oán

Cho αPK nghiệm P Như phân tích mục 4,

α biểu diễn dạng biểu thức đại số có thức tương đương với việc tồn chuỗi mở rộng liên tiếp (3.17), với Ki»Ki´1[x]/(xni´βi), K[α] ĂKr Vì trường phần rã L

P mở rộng Galois nhỏ chứa K[α]cho nên:

KĂLĂKr

Điều kéo theo ΓP = ΓL/K thương Γ Với điều kiện trường sởΓ chứa đủ nguyên sơ đơn vị, ta Γ nhóm giải Vì nhóm Galois ΓP đa thức

P nhóm giải được.ΓPlà nhóm giải điều kiện cần để α biểu diễn biểu thức đại số với thức Để chứng minh điều kiện điều kiện đủ, ta cần chứng minh nếuKlà trường có nghiệm nguyên sơ cấp

ncủa đơn vị L/Klà mở rộng Galois có nhóm Galois đẳng cấu với Z/nZ,thì tồn βP Kđể cho

L»K[x]/(xn´β)

Giả sử mở rộng L trường phân rã đa thức bất khả qui P P K[x] có bậc n Nếu α1, α1, , αn nghiệm P K L mở rộng Ksinh nghiệm

Chọn phần tử sinh σ nhóm Galois ΓL/K =Z/nZ Vì tác động ΓL/K lên tập α1, ,α1, , αn tác động bắc cầu, σtương ứng với hoán vị n - chu trình Ta giả sử

σ(α1) =α2, σ(α2) =α3, ,σ(αn) =α1

Chọn ζ P K nguyên sơ cấp n đơn vị thiết lập

giải thc Lagrange:

=1+2+ă ă ă+n1n

Lp lun nh trường hợp n = 3, ta thấy ξn = β phần tử củaK từ suy

L»K[x]/(xn´β)

(28)

Tạp

chí

onl

ine

của

cộng

đồng

những

người

yêu

T

ốn

hồn tồn khơng dễ Quy trình khảo sát trình bày mục5, cho ta số thơng tin nhóm Galois, có chuỗi nhóm (3.16), thường nhóm khơng chuẩn tắc Trong trường hợp đa thức tổng quát

P =xn+a1xn1+ă ă ă+anPK[x], (3.18) vi K = k(a1,a2, , an) trường phân thức với biến số

a1, a2, , an, lập luận 9, ta thấy ΓP = Υn Như phương trình tổng quát, ta cần tìm hiểu với n nhóm đối xứng Υn nhóm giải Ta nhómΥ3,Υ4 giải nhờ tìm cơng thức biểu

diễn nghiệm phương trình bậc ba bậc bốn Trong

Υn nhóm khơng giải với ně5

9 Phương trình bậc ba

Lời giải phương trình bậc ba Tartaglia phức tạp giống từ trời rơi xuống Lời giải phương trình bậc bốn Ferrari cịn phức tạp Sau này, Lagrange đưa phương pháp đẹp để tìm phương pháp chung cho lời giải Tartaglia, Cardano Ferrari Ơng sáng tạo cơng cụ mới, gọi resolvent, mà tạm chuyển ngữ thành “giải thức”

Để minh hoạ cho cơng dụng phương pháp Lagrange, ta trình bày lời giải phương trình bậc ba tổng quát theo phương pháp với chút hỗ trợ lý thuyết Galois Mặc dù phương pháp giải thức Lagrange diễn giải dễ dàng lý thuyết Galois, ta nên lưu ý nhóm Galois sinh sau phương pháp giải thức Lagrange

Trong đại số đại, ta gán cho chữ tổng quát nghĩa xác: Ta chọn trường sở

K=k(a,b,c)ĄR=k[a,b,c],

là trường biểu thức hữu tỉ với biến số a, b, c hệ số nằm trường knào Đa thức bậc ba tổng quát đa thức

(29)

Tạp

chí

onl

ine

của

cộng

đồng

những

người

yêu

T

oán

Ta giả sử trường k có nguyên sơ bậc ba đơn vị Nói cách khác đa thứcx3

´1P k[x] phân rã hoàn toàn thành:

x3´1= (x´1)(x´j)(x´j2),

với jPklà nguyên sơ bậc ba đơn vị Lấy ví dụ, ta chọn k=Q[j] với jnhư (3.11)

Cho K trường đóng đại số chứa K, gọi α1,α2,α3

nghiệm củaP K.Trường phân rã K:

L=k(α1,α2,α3)ĄS=k[α1,α2,α3],

là trường thương đa thức có biến số α1, α2, α3 thoả

´a = α1+α2 +α3

b = α1α2+α2α3+α3α1

´c = α1α2α3

Có thể chứng minh S module tự cấp sáu vành R vàLlà khơng gian vector có chiều sáu trường

K Như nhóm Galois Γ P có sáu phần tử tồn nhóm đối xứng Υ3 Để giải phương trình

bậc ba tổng quát, ta khảo sát nhóm đối xứng Υ3 Ta thể

hiện hoán vị cấp ba ví dụ sau: (2, 3, 1) hốn vị

1 Đ 2, Đ 3, Đ Các phần tử Υ3 phân loại

như sau

• Phần tử đơn vị (1, 2, 3)

• Phần tử có 2-chu trình (2, 1, 3), (1, 3, 2), (3, 2, 1) • Phần tử 3-chu trình (2, 3, 1), (3, 1, 2)

Đồng cấu dấu sgn : Υ3 Ñ t˘1u gán cho ba phần tử có 2-chu

trình dấu ´1 gán cho ba phần tử cịn lại dấu +1 Hạch đồng cấu dấu sgn : Υ3 Đ t˘1u nhóm ln phiên Θ3 Nhóm

này có phần tử:

Θ3 =t(1, 2, 3),(2, 3, 1),(3, 1, 2)u

và đẳng cấu với nhóm xíchZ/3Z Tóm lại ta có dãy khớp

(30)

Tạp

chí

onl

ine

của

cộng

đồng

những

người

yêu

T

ốn

với Θ3 »Z/3Z Nhóm Υ3 nhóm giải

Nhóm Θ3 tương ứng với mở rộng trung gian K+ = LΘ3 bao gồm phần tử L cố định tác động Θ3 Ta có

chuỗi mở rộng liên tiếp

KĂK+ ĂL

với K+/K mở rộng Galois cấp hai có nhóm Galois Z/2Z,

L/K+ mở rộng Galois cấp ba có nhóm Galois Z/3Z

Quá trình biểu diễn nghiệm phương trình bậc ba phổ biểu thức đại số có thức chia thành hai bước:

• Sử dụng dãy khớp (3.19) để xây dựng mở rộng trung gian

K Ă K+ Ă L với K+/K mở rộng Galois cấp hai có nhóm Galois Z/2Z, L/K+ mở rộng Galois cấp ba có nhóm Galois Z/3Z

• Biểu diễn mở rộng trung gian K+/K dạng

K+ =K[x]/(x2´β1),

vàL/K+ dạng L=K+[x]/(x3´β2)

Ta viết tường minh phần tử K+ :

δ= (α1´α2)(α2´α3)(α3´α1)

Dễ thấy δ ổn định tác động Θ3 Ta để ý

thấy δ2 =´d với

d=ź

i‰j

(αi´αj),

là biệt thức củaP Biệt thứcd phần tử Kvà ta có

K+ »K[x]/(x2+d) (3.20) Thật vậy, ta có đồng cấu trường:

K[x]/(x2+d)ĐK+,

(31)

Tạp

chí

onl

ine

của

cộng

đồng

những

người

yêu

T

oán

Nói cách khác, phần tử K+ biểu diễn cách dạng m+nδ với m, nPK

Để chứng tỏ phương trình tổng quát bậc ba giải thức, ta cần chứng minh mở rộng L/K+ biểu diễn dứới dạng phương trình

L=K+[x]/(x3´β),

với phần tử β P L+ Ký hiệu σ = (2, 3, 1) hoán vị tác động lên nghiệm P sau: σ(α1) =α2, σ(α2) = α3

và σ(α3) =α1.Giải thức Lagrange có dạng:

ξ=α1+jα2+j2α3

Ta nhận thấy rằng:

σ(ξ) =j´1ξ và σ2(ξ) =j´2ξ,

và từ suy

β=ξσ(ξ)σ2(ξ) =ξ3

Phần tử β biểu diễn tích ξσ(ξ)σ2(ξ) hiển nhiên có tính ổn

định tác động σ Vì vậyβPK+ Ta khẳng định

L»K+[x]/(x3´β) (3.21) Thật ta có đồng cấu vành

φ:K+[x]/(x3´β)ÑL,

xác định x ÞĐξ ξ3 = β. Vì σ(ξ)‰ ξ, cho nên ξR K, và ảnh

của đồng cấu φ mở rộng trung gian

KĂim(φ)ĂL

với K ‰ im(φ) Ta nhận thấy chiều im(φ) không gian vector Kphải ba mở rộng bậc ba L khơng thể chứa mở rộng bậc hai

Nói cách khác, đồng cấu φ tồn ánh Vì khơng gian nguồn đích có số chiều, φ đẳng cấu Như phần tử củaL,trong cóα1, α2, α3,có thể biểu diễn cách

dưới dạng

(32)

Tạp

chí

onl

ine

của

cộng

đồng

những

người

yêu

T

oán

Bản thân ξ thoả mã phương trình ξ3 =β với β P K+ Mọi phần tử K+ biểu diễn cách dạng

m+nδ với m, n PK δ2 =d. Chịu khó tường minh hố hệ

số m, n, m0,m1, m2, ta tìm lạo cơng thức Tartaglia (3.5)

cho nghiệm phương trình bậc ba tổng quát

x3+ax2+bx+c=0

10 Phương trình bậc bốn

Định lý 10.1. Nhóm Υ4 là nhóm giải được.

Ký hiệu S tập t1, 2, 3, 4u Ψ2(S) tập tập S có

đúng hai phần tử Tập có phần tử chia thành ba cặp tập đối nhau:

t1, 2u,t3, 4u t1, 3u,t2, 4u t1, 4u,t2, 3u

và gọi Φ2(S) tập cặp tập đối Ta có

ánh xạ 2-1

Ψ2(S)ĐΦ2(S) (3.22)

Nhóm Υ4 hốn vị S, tác động cách tương thích lên

Φ2(S) Ψ2(S) Tác động Υ4 lên Φ2(S) cho ta đồng cấu

Υ4 Ñ Υ3 Khảo sát kỹ tác động Υ4 lên Φ2(S) Ψ2(S) ta có

dãy khớp

0Đ(Z/2Z)2 ĐΥ4 ĐΥ3 Đ0 (3.23)

và từ suy nhóm Υ4 nhóm giải

Bạn đọc dùng giải thức Lagrange để tìm biểu thức thức cho nghiệm phương trình bậc bốn tổng quát, giống trường hợp phương trình bậc ba trình bày mục9

11 Phương trình bậc năm trở lên

Định lý 11.1. Với mọině5, Υn khơng phải nhóm giải được. Nhóm đối xứng Υn có đồng cấu dấu với hạch nhóm luân phiên Θn

(33)

Tạp

chí

onl

ine

của

cộng

đồng

những

người

yêu

T

ốn

Có thể chứng minh với n ě 5, nhóm luân phiên Θn nhóm đơn, tức nhóm khơng có nhóm chuẩn tắc, ngồi nhóm tầm thường

Nhóm chuẩn tắc hợp số lớp liên hợp số phần tử nhóm với tổng số phần tử số lớp liên hợp Mặt khác, tổng số phải ước số phần tử Θ5

Bằng cách liệt tất lớp liên hợp Θ5 lực lượng

chúng, ta nhận tổng lực lượng số lớp liên hợp Θ5 ước thực 60 Vì

nhóm Θ5 khơng có nhóm chuẩn tắc ngồi

nhóm tầm thường

(34)

Tạp

chí

onl

ine

của

cộng

đồng

những

người

yêu

T

(35)

T p c h í

online

cộng đồng

những n g i y ê u T o n

Tạp

chí

onl

ine

của

cộng

đồng

những

người

yêu

T

oán Évariste Galois

Người dịch: Lưu Trọng Luân

Đại học FPT

Évariste Galois sinh Bourg La Reine (gần Paris) ông

Nicholas Gabriel Galois bà Adelaide Marie Demante Cha mẹ Galois trí thức giáo dục kỹ triết học, văn học cổ điển tôn giáo Tuy nhiên, không gia đình Galois bộc lộ khả tốn học Mẹ Galois người thầy dạy dỗ ông năm 12 tuổi Bà dạy ông tiếng Hy Lạp, La tinh tôn giáo, lúc bà bắt đầu gieo tư tưởng hồi nghi cho Cha Galois người có ảnh hưởng cộng đồng năm 1815 bầu làm thị trưởng Bourg-la-Reine

Thời điểm bắt đầu kiện lịch sử đóng vai trị quan trọng đời Galois đánh chiếm nhà tù Bastille ngày 14/07/1789 Từ lúc đó, vương triều Louis16 bị lung lay đội đơng đảo người dân Pháp đồn kết lại nhằm lật đổ cai trị đặc quyền giáo hội nhà nước

Bất chấp nỗ lực thỏa hiệp, vua Louis 16 bị mang xét xử sau tìm cách trốn khỏi đất nước Sau nhà vua bị hành hình vào ngày 21/01/1973, nước Pháp rơi vào tình trạng kinh hồng với nhiều phiên xét xử trị Đến cuối năm1793,

có tới4595tù nhân trị bị giam giữ Paris Tuy nhiên, tình hình đất nước bắt đầu sáng sủa sau quân đội nước này, huy Napoleon giành hết chiến thắng đến chiến thắng khác

Napoleon trở thành Đệ Tổng tài vào năm 1800 lên ngơi Hồng đế vào năm 1804 Quân đội Pháp tiếp tục chinh phục châu Âu quyền lực Napoleon ngày củng cố Năm1811, Napoleon đạt đến đỉnh cao quyền lực Nhưng đến

(36)

Tạp

chí

onl

ine

của

cộng

đồng

những

người

yêu

T

oán

năm 1815thì quyền lực sụp đổ Cuộc cơng Nga năm 1812

thất bại kéo theo bại trận khác Quân liên minh tiến vào Paris ngày 31/03/1814 Napoleon thoái vị ngày 6/4 Louis XVIII liên minh đưa lên làm vua Năm 1815 chứng kiến 100 ngày tiếng Napoleon tiến vào Paris ngày 20/3, bị đánh bại Waterloo ngày 18/6 thoái vị lần thứ hai ngày

22/6.Louis XVIII khôi phục vua vào tháng

9/1824 Charles X trở thành vị vua

Galois lúc học Ông vào học lớp nội trú trường Lycée of Louis-le-Grand ngày06/10/1823 Trong học kỳ đầu tiên, loạn nhỏ nổ 40 học sinh bị đuổi học Galois không liên quan đến việc Năm học 1824´1825, ông đạt học lực giỏi nhận nhiều giải thưởng Tuy nhiên, năm 1826,

Galois phải học lại mơn hùng biện khơng đạt u cầu

Tháng2/1827 thời điểm mang tính bước ngoặc đời Galois Ơng vào lớp tốn mình, lớp M Vernier Ngay lập tức, ơng bị toán học hút giáo viên hướng dẫn nhận xét:

Niềm đam mê toán học chi phối cậu ấy, nghĩ tốt ba mẹ cậu nên cho phép cậu không học mơn khác ngoại trừ mơn Cậu lãng phí thời gian đây, làm phiền các giáo viên chuốc lấy nhiều hình phạt.

Học bạ Galois bắt đầu xuất từ cá biệt, kỳ dị, lập dị hướng nội Có lẽ nhà toán học lập dị thời đại Thầy giáo M Vernier nhận xét:

Thông minh, tiến đáng kể chưa đủ phương pháp.

(37)

Tạp

chí

onl

ine

của

cộng

đồng

những

người

yêu

T

oán

Cậu sinh viên thích tập trung vào vấn đề đỉnh cao của tốn học.

Tháng 4/1829 Galois cơng bố cơng trình tốn học liên phân số Annales de mathématiques Ngày 25/5 1/6

ông gửi viết phương pháp giải phương trình đại số cho Viện hàn lâm Khoa học Cauchy người phân cơng đánh giá cơng trình

Bi kịch ập đến với Galois vào ngày 02/07/1829 cha ông tự tử Linh mục xứ Bourg-la-Reine giả mạo tên thị trưởng Galois châm biếm nhằm vào người thân gia đình Galois Cha Galois người tốt vụ scandal vượt q sức chịu đựng ơng Ơng treo cổ tự sát hộ Paris, cách Louis-le-Grand, nơi học vài bước chân Galois bị tác động nghiêm trọng chết cha ảnh hưởng lớn đến hướng sau ông

Vài tuần sau chết cha mình, Galois tiếp tục đăng ký thi vào trường Bách khoa Paris lần thứ hai Lần ơng lại trượt, có lẽ phần rơi vào thời điểm tồi tệ sau chết cha ông, phần ông chưa giỏi việc diễn đạt ý tưởng tốn học sâu sắc Vì Galois đành phải vào học École Normale, nhánh trường Louis-le-Grand Để vào đây, ông phải dự kỳ thi tú tài mà vào trường Bách khoa Paris Galois khơng cần đến Ơng thi đậu nhận tốt nghiệp ngày

29/12/1829 Người đánh giá mơn tốn nhận xét:

Sinh viên đơi diễn đạt ý kiến khó hiểu cậu ta thông minh bộc lộ tinh thần nghiên cứu đặc biệt.

Giám khảo mơn văn nhận xét:

Đây sinh viên có kết tệ, cậu ta tuyệt đối chẳng biết Họ nói với tơi cậu ta có khả tốn phi thường Thật ngạc nhiên sau buổi thi này, tơi tin cậu ta có chút thơng minh.

(38)

Tạp

chí

onl

ine

của

cộng

đồng

những

người

yêu

T

oán

báo gửi cho Fourier, thư ký Viện hàn lâm Paris để xét duyệt Giải thưởng lớn toán học Fourier tháng4/1830 cơng trình Galois khơng tìm thấy Galois, sau đọc cơng trình Abel Jacobi bắt tay vào việc nghiên cứu lý thuyết hàm eliptic tích phân Abel Với hỗ trợ Jacques Sturm, ông công bố báo Bulletin de Férussac vào tháng 4/1830 Tuy nhiên, tháng6, ông biết giải thưởng học viện đồng trao cho Abel (sau mất) Jacobi cơng trình ơng không xem xét

Tháng 7/1830 nổ cách mạng Vua Charles X phải trốn khỏi nước Pháp Bạo động diễn đường phố Paris giám đốc trường École Normale, M Guigniault, đóng cổng trường để ngăn sinh viên tham gia bạo động Galois tìm cách trèo tường để tham gia không thành Tháng

12/1830 M Guigniault viết số báo trích sinh viên Galois viết thư đáp trả Gazette des Écoles, trích M Guigniault hành động giam sinh viên bên trường Vì thư mà Galois bị đuổi học gia nhập đội pháo binh Vệ binh quốc gia, nhánh dân quân tự vệ theo phe cộng hòa Ngày 31/12/1830, đội pháo binh Vệ binh quốc gia bị hồng gia lệnh giải tán vua Louis-Phillipe lo ngại mối đe dọa ngai vàng

Hai công bố nhỏ, tóm tắt đăng trên Annales de Gergonne (12/1830) thư việc giảng dạy khoa học Gazette des Écoles(2/1/1831)là ấn phẩm cuối đời ông Tháng 1/1831 Galois nỗ lực quay trở lại với tốn học Ơng tổ chức vài lớp đại số cao cấp thu hút 40 sinh viên đến dự buổi sau số nhanh chóng giảm xuống Ơng Poisson mời gửi phiên thứ ba cơng trình phương trình cho Viện hàn lâm ông thực ngày 17/1

Ngày18/4,Sophie Germain viết thư cho bạn cơ, nhà tốn học Libri kể tình hình Galois

(39)

Tạp

chí

onl

ine

của

cộng

đồng

những

người

yêu

T

oán

Cuối năm 1830, tất 19 sĩ quan thuộc đội pháo binh Vệ binh quốc gia bị bắt bị kết tội âm mưu lật đổ quyền Họ tuyên trắng án ngày 09/05/1831, có 200 người theo phe cộng hòa tập trung ăn tối mừng việc Trong bữa tiệc, Galois nâng ly tay cầm dao găm mở vẻ hăm dọa nhà vua, Louis-Phillipe Sau bữa ăn tối, Galois bị bắt giam nhà tù Sainte-Pélagie Ông thả sau phiên xử ngày 15/6

Ngày 14/7, ngày diễn công nhà tù Bastille Galois bị bắt trở lại mặc đồng phục đội pháo binh Vệ binh quốc gia vốn bị giải tán Lúc ơng mang người súng trường nạp đạn, vài súng ngắn dao găm Galois bị đưa trở lại nhà tù Sainte-Pélagie Thời gian này, ông nhận tin cơng trình bị bác bỏ Poisson nhận xét:

Lập luận Galois chưa đủ rõ ràng chưa phát triển đầy đủ phép chúng tơi đánh giá tính xác nó.

Tuy nhiên, ơng khuyến khích Galois cơng bố tóm tắt đầy đủ cơng trình Ở nhà tù Sainte-Pélagie, Galois cố dùng dao tự tử bạn tù khác ngăn ông lại Khi say rượu tù, ông bộc lộ tâm hồn mình:

Bạn có biết tơi thiếu khơng? Tơi tiết lộ với bạn thơi: Đó là người mà tơi u thương u thương tâm khảm. Tôi cha khơng thay ơng, bạn có hiểu không?

Tháng 3/1832, trận dịch tả quét qua Paris tù nhân, có Galois, chuyển đến trại Sieur Fault-rier Nơi đây, dường ông phải lòng Stephanie-Felice du Motel, gái bác sĩ địa phương Sau tù ngày 29/4,

Galois viết thư qua lại với Stephanie rõ ràng tìm cách né tránh tình

Cái tên Stephanie xuất nhiều lần bên lề ghi chép Galois Galois đấu súng với Perscheux d’Herbinville ngày

30/5 mà lý không rõ chắn việc có liên quan đến Stephanie Một số ghi chép có câu:

(40)

Tạp

chí

onl

ine

của

cộng

đồng

những

người

u

T

ốn

Chính điều dẫn đến huyền thoại ông dành đêm cuối viết lại tất ông biết lý thuyết nhóm Câu chuyện phóng đại lên

Galois bị thương đấu súng bị d’Herbinville người theo bỏ mặc nông dân tìm thấy Ơng bệnh viên Cochin ngày 31/5 đám tang tổ chức ngày 2/6 Nó trở thành tâm điểm loạn phe cộng hòa kéo dài nhiều ngày

Em trai Galois bạn ông, Chevalier, chép lại viết liên quan đến tốn học ơng gửi cho Gauss, Jacobi người khác Galois mơ ước Jacobi Gauss nhận xét cơng trình Người ta khơng tìm thấy nhận xét người Tuy nhiên, báo đến tay Liouville, người vào tháng9/1843công bố trước viện hàn lâm ơng tìm thấy báo Galois lời giải xác

vừa xác vừa sâu sắc cho tốn tuyệt vời sau: Cho một phương trình bất khả quy với bậc nguyên tố, xét xem có giải được thức không?

(41)

T p c h í

online

cộng đồng

những n g i y ê u T o n

Tạp

chí

onl

ine

của

cộng

đồng

những

người

yêu

T

oán NGHỊCH ĐẢO M ¨OBIUS

Ngô Quang Hưng (Đại học Buffalo, Mỹ)

Phộp nghch o Măobius nguyờn l mt cụng thc lý thuyết số Đến năm 1960 Giáo sư Gian-Carlo Rota cho thấy công thức lý thuyết số trường hợp đặc biệt công thức áp dụng tập thứ tự bỏn phn (poset) Cụng thc Măobius tng quỏt cú nhiu ứng dụng Tốn Máy Tính Trong ta ro qua chng minh ca phộp nghch o Măobius tập thứ tự bán phần vài ứng dụng

1 Ba ví dụ

1.1 Tốn tổ hợp

Cơng thứcinclusion-exclusion nói rằng, để đếm tổng số nhóc tì có Chí Phèo bố thị Nở mẹ, ta cộng số chí Phèo với số thị Nở trừ số chung Nói cách khác, cho n tập hp hu hn A1,ă ă ă,An thỡ ta cú th tính lực lượng hội chúng cơng thức:

ˇ ˇ ˇ ˇ

Ťn i=1Ai

ˇ ˇ ˇ ˇ

=řni=1|Ai| ´

ř

1ďiăjďn|AiXAj|+

1ijkn|AiXAjXAk| ă ă ă+ (1) n1

|A1X ă ă ă XAn| Cụng thc ny mt số sách nói Abraham de Moivre; xuất năm 1854 từ báo Daniel da Silva, lần năm 1883 báo Joseph Sylvester [11]

(42)

Tạp

chí

onl

ine

của

cộng

đồng

những

người

yêu

T

oán

1.2 Lý thuyết số

Trong lý thuyết số có cơng thức gọi làcơng thc nghch o Măobius [10], xinh hn hoa hu! Cụng thức phát biểu sau: Cho 2hàm số f,g miền số nguyên dương, ta có

f(n) = ÿ

d|n

g(d), @ně1

tương đương với

g(n) =ÿ

d|n

µ(d)f(n/d), @ně1

trong ú à(d) lhm Măobius nh ngha nh sau

µ(d) = $ ’ &

’ %

1 dlà tích số chẵn số nguyên tố khác

´1 dlà tích số lẻ số nguyên tố khác

0 dcó ước số bình phương số nguyên tố August Ferdinand Măobius l mt nh thiờn ngi c, trợ lý Gauss; ông tác gi ca cỏi bng Măobius

lng danh hỡnh học Tơ-pơ

1.3 Hình tơ-pơ

Cơng thức đa diện Euler phát biểu v´e+f= 2,

v,e,f tổng số đỉnh, cạnh, mặt khối đa diện ba chiều Euler khám phá công thức năm 1752, Descartes biết từ 1640 Trăm năm sau, năm 1852, Schlăafli phỏt biu cụng thc tng quỏt cho cỏc a diện lồi không giann-chiều, chứng minh phải chờ đến người khổng lồ Henry Poincaré (1893, [4])

Công thức Euler tổng quát, gọi làcông thức Euler-Poincaré, phát biểu sau Gọi Fi tổng số “mặt”i-chiều đa diện

n chiều (“mặt” 0-chiều đỉnh, mặt 1-chiều cạnh, vân vân) Ta quy ướcFn =1vàF´1 =1để viết cho tiện Thì ta có cơng thức

Euler-Poincaré

n

ÿ

i=´1

(43)

Tạp

chí

onl

ine

của

cộng

đồng

những

người

yêu

T

oán

1.4 Gian-Carlo Rota

Năm 1964, chuỗi báo kinh điển đặt móng cho lý thuyết tổ hợp đại số [5], Gian-Carlo Rota cho biết ba công thức chẳng qua trường hợp đặc bit ca phng phỏp tớnh nghch o Măobius trờn cỏc

tập hợp thứ tự phần (partially ordered set, hay poset) M phng phỏp nghch o Măobius trờn posets chẳng qua phát biểu sau đây: A ma trận vng khả nghịch, x=Ay tương đương với y=A´1x Đại số tuyến tính mn

năm! Rota có sách thú vị có nhiều giai thoại tiếng giới chuyên môn tên “Indiscrete Thoughts” [6]

Dưới duyệt qua phương pháp Rota, chứng minh ba công thức trên, chứng minh bổ đề Sauer-Shelah để tự thưởng cụng

2 Nghch o Măobius trờn posets

2.1 Tập hợp thứ tự bán phần (Poset)

Poset đại khái tập hợp mà ta so sánh lớn nhỏ số cặp phần tử không thiết so tất cặp Thứ tự lớn nhỏ có tính bắc cầu (transitive) không tạo thứ tự luẩn quẩn

Cụ thể hơn, poset (tập thứ tự bán phần) cặp (P,ĺ)

trong P tập hợp ĺ quan hệ nhị phân (hay quan hệ hai ngôi) phần tử P thỏa mãn tính chất x ĺy yĺz suy rax ĺz, với x,y,z P P (tính bắc cầu

– transitive)

2 xĺx,@xPP (tính phản xạ – reflexive)

3 x ĺyvà yĺx suy x=y(tính phản xứng – antisymmet-ric)

(44)

Tạp

chí

onl

ine

của

cộng

đồng

những

người

yêu

T

oán

H t2u

t1u t3u

t1, 3u

t1, 2u t2, 3u

t1, 2, 3u

Hình 5.1: Đại số Bool B3

Ví dụ 2.2. P =Dn tập tất ước số dương n, quan hệ nhị phân quan hệ “chia hết”, nghĩa iĺjnếu

i|j Ký hiệu i|j nghĩa j chia hết cho i (hay i chia hết j) Xem ví dụ Hình 5.2

1

2

4 10 15

12 20 30 60

Hình 5.2: Poset ước số của60

(45)

Tạp

chí

onl

ine

của

cộng

đồng

những

người

yêu

T

oán

bộ đa diện mặt với số chiều bằngn Poset gọi

face lattice polytope Xem ví dụ Hình5.2

a

b e

c

d

H

c e

a d b

eb ab ad

ea ec ed cb cd

ecb

eab ead ecd abcd

abcde

Hình 5.3: Face lattice hỡnh Pyramid

2.2 Hm Măobius ca poset

Nhng điều ta viết sau cho trườngKtùy hỉ posets vơ hạn (miễn hữu hạn địa phương1) Để cho đơn

giản, ta phát biểu kết với K=C posets hữu hạn

Gọi(P,ĺ)là poset hữu hạn Ta xét ma trậnαkích thước

|P| ˆ |P| cho α(x,y) = x ł y Khi x ĺ y α(x,y) P C tùy hỉ Tập ma trận gọi đại số kề (incidence algebra) P, ký hiệu làI(P) Trong đại số kề ma trận δ định nghĩa

δ(x,y) = #

1 x =y

0 x ‰y

là ma trận đơn vị

Định lý 2.4. Cho trước poset (P,ĺ) trong đó P hữu hạn Xét một ma trận α P I(P) tùy ý thì α khả nghịch nếu α(x,x) ‰ 0,@xPP.

1Nghĩa số thành viên nằm cặpxvà ylà hữu hạn với mọi

(46)

Tạp

chí

onl

ine

của

cộng

đồng

những

người

yêu

T

oán

Chứng minh. Nếu ta vẽ “đồ thị" củaP cách xem P tập đỉnh vẽ mũi tên từ x đến y x ĺ y (như Hình 5.1 5.2) ta có đồ thị có hướng khơng có vịng trịn (directed acyclic graph) Do đó, tồn cách liệt kê tất phần tử củaP từ trái sang phải cho tất mũi tên trỏ sang phải trỏ vào (loop đồ thị) Thứ tự gọi trật tự tô-pô (topological ordering) đồ thị, tập học thuật toán duyệt đồ thị Nếu ta viết ma trận α P I(P) mà hàng cột đánh số theo thứ tự ta có ma trận tam giác (upper-triangular) Do α khả nghịch α(x,x) ‰ 0,@x, nghĩa phần tử đường chéo khác không Lưu ý ma trận nghịch đảo ma trận tam giác trên, thuộc đại số kề

Một thành viên quan trọng đại số kề I(P) ma trận ζ, gọi hàm zetacủa P, định nghĩa

ζ(x,y) = #

1 x ĺy

0 x ły

Định nghĩa 2.5 (Hm Măobius ca mt poset). Hm Măobius ca poset (P,), ký hiệu µ, ma trận nghịch đảo hàm zeta ζ (Theo Định lý 2.4 ζ khả nghịch.)

Kế đến ta mô tả công thc quy tớnh hm Măobius ca mt poset Từ định nghĩa phép nhân ma trận, vớiα,βPI(P)

bất kỳ ta có

(αβ)(x,y) = ÿ

zPP

α(x,z)β(z,y) = ÿ

xĺzĺy

α(x,z)β(z,y),

tại x ł z α(x,z) =0, cịn z ł y β(z,y) =0 Do đó, từµζ =δ ta suy

δ(x,y) = ÿ

xĺzĺy

µ(x,z)ζ(z,y) = ÿ

xĺzĺy

µ(x,z)

Hay viết cụ thể với x,yPP ta có

ÿ

xĺzĺy

µ(x,y) = #

1 x=y

(47)

Tạp

chí

onl

ine

của

cộng

đồng

những

người

yêu

T

oán

Đẳng thức (5.1) suy cơng thức quy nạp để tínhµ(x,y):

µ(x,y) = $ ’ &

’ %

1 x=y

´řxĺzăyµ(x,z) xăy

0 xły

Từ cụng thc ny ta suy giỏ tr hm Măobius cho ba posets Hai đẳng thức đầu dễ (làm tập), thứ ba khó Nếu P =Bn tập tất tập [n] (đại số Bool),

thì

µ(A,B) = #

(´1)|B|´|A| A

ĎB

0 AĘB

2 Nếu P =Dn tập tất ước số n,

µ(x,y) = #

(´1)r y/x tích rsố nguyên tố khác

0

3 Nếu P face-lattice đa điện nchiều

µ(A,B) = #

(´1)dim(B)´dim(A) if A

ĎB

0 nu khụng (5.2)

2.3 Nghch o Măobius

Xột hai hàm số f,g:P ĐCbất kỳ Ta xem chúng hai vectors không gian C|P| Công thức nghch o Măobius trờn poset núi hai iu rt n giản:

f=ζgơg=µf, (5.3)

và, xoay ngang vectors

f=gζơg=fµ (5.4)

Để hiểu ý nghĩa tổ hợp tương đương này, ta viết rõ ràng chút ta biếtζ(x,y) vàµ(x,y) 0nếu x ły Quan hệ (5.3) nói rằng:

f(x) = ÿ

xĺy

g(y),@xPP ô g(x) = ÿ

xĺy

(48)

Tạp chí onl ine của cộng đồng những người yêu T oán

Đẳng thức ta hiểu sau Giả sử ta có hàm g gán số g(y) vào thành viên yPP, fgán vào xPP

con số tổng g(y) cho x ĺ y, vế phải (5.3) cho ta cách tính g dựa f

Đối ngẫu lại, quan hệ (5.4) nói rằng:

f(x) = ÿ

xľy

g(y),@xPP ô g(x) = ÿ

xľy

µ(y,x)f(y),@yPP (5.6)

Ví dụ 2.6. Để có cơng thức Euler-Poincaré, ta áp dụng (5.5) g(y) =1 với y=P g(y) =0 với mọiy cịn lại trongP Khi đó, rõ ràng tất f(x)đều Dùng (5.2), ta có

0=g(H) = ÿ mặtB

(´1)dim(B)´dim(H)f(B) = ÿ mặtB

(´1)dim(B)+1 =´

n

ÿ

i=´1

(´1)iFi

Ví dụ 2.7. Áp dụng (5.6) cho posetP =Dn, ta cú cụng thc nghch o Măobius cổ điển lý thuyết số

Ví dụ 2.8. Cịn cơng thức inclusion-exclusion sao? Cách hiểu sau hữu dụng nhiều trường hợp Giả s ta cú mt bi ve" U=A1Y ă ă ¨ YAn Mỗi viên bi có nhiều màu Các màu đánh số từ đến n Gọi Ai tập viên bi có màu

i Với XĎ[n] tùy ý, gọi g(X) tập tất viên bi có đúng

các màu X mà thơi Khi đó,

f(X) = ÿ

XĎY

g(Y)

chính số viên bi mà viên có ít nhất màu trongX, f(H) =|U| Do đó,

f(X) = ˇ ˇ ˇ ˇ ˇ č

iPX

Ai ˇ ˇ ˇ ˇ ˇ

Áp dụng (5.5) cho poset P =Bn ta kết luận

0=g(H) = ÿ

YĎ[n]

(´1)|Y| ˇ ˇ ˇ ˇ ˇ č

iPY

Ai ˇ ˇ ˇ ˇ ˇ

Chuyển f(H) = |U| sang vế ta có cơng thức

(49)

Tạp

chí

onl

ine

của

cộng

đồng

những

người

yêu

T

oán

3 Bổ đề Sauer–Shelah

Chúng ta tự thưởng công cách chứng minh bổ đề tổ hợp quan trọng gọi bổ đề Sauer-Shelah [7, 8] Bổ đề có ứng dụng sâu sắc lý thuyết học máy, cụ thể lý thuyết “chiều Vapnik-Chervonenkis” (VC dimension) [13,12]

Gọi F tập [n] Với S Ď [n] bất kỳ, định nghĩa “hình chiếu” Flên Slà tập

ΠF(S) =tFXS| FPFu

Ta nói F “băm nát” S ΠF(S) =2|S|.

Bổ đề 3.1 (Bổ đề Sauer-Shelah). Cho trước F là tập con của [n] Gọid là kích thước lớn tậpS Ď[n] bị F

băm nát, thì

|F| ďΦd(n) = d

ÿ

i=0

n i

Chứng minh. Ta chứng minh bổ đề “phương pháp chiều”(Đại Số tuyến tính van tuế!) Gọi ď[nd]là tập tất tập [n] với kích thước bé d Với F P F,

định nghĩa hàm số hF : [ďnd]

ÑRnhư sau:

hF(X) =

#

1 XĎF

0 XĘF

Các hàm hF vectors khơng gianRΦd(n) Có tất cả|F| vectors hF, chúng độc lập tuyến tính |F| ď Φd(n) Giả sử chúng khơng độc lập tuyến tính, nghĩa tồn hệ số αF cho

ÿ

FPF

αFhF =0 (5.7)

và hệ số không Để cho tiện, ta mở rộng định nghĩa gán αX =0 với XP2[n]zF

Từ (5.7), với X P ď[nd] ta có řFPFαFhF(X) = 0, hay nói

cách khác với X P [ďnd] tùy ý ta có řXĎYαY = Định nghĩa

βX=

ř

XĎYg(Y), ta vừa thấy βX =0,@XP ď[nd]

GọiY tập nhỏ của[n]sao choβY ‰0 (Nếu ta lấy tập

(50)

Tạp

chí

onl

ine

của

cộng

đồng

những

người

yêu

T

ốn

đó tồn tập Y nhỏ định nghĩa.) Dĩ nhiên |Y| ěd+1 Ta chứng minh Y bị F băm nát, từ dẫn đến điều vô lý Để chứng minh Y bị băm nát ta cần chứng minh, với ZĎ Y

tùy ý, tồn F P F cho FXY = Z Để chứng minh điều cần chứng minh

ÿ

AĎ[n],AXY=Z

αA ‰0

là xong, αA = 0,@A R F Đến ta xét poset Bm gồm tất tập Y ´Z (đặt m = |Y´Z|) Poset đại số

Bool bậc m Với phần tửW ĎY´Z, định nghĩa

g(W) = ÿ

X:XXY=ZYW

αX

Và định nghĩa, với V ĎY´Z,

f(V) = ÿ

VĎWĎY´Z

g(W)

(Lưu ý ta dùng dạng (5.5) nghịch đảo Măobius.) D thy rng

f(V) =ZYV, @V PBm

Do Y tập nhỏ với βY = 0, ta có f(V) = 0,@V ‰ Y´Z,

f(Y´Z) =Y Theo nghch o Măobius ta cú

AĎ[n],AXY=Z

αA =g(H) =

ÿ

VĂY´Z

(´1)|V|f(V) = (´1)|Y´Z|βY ‰0

4 Chú thích

(51)

Tạp

chí

onl

ine

của

cộng

đồng

những

người

yêu

T

oán

Tài liệu tham kho

[1] BJ ăORKLUND, A., HUSFELDT, T., KASKI, P., AND KOIVISTO,

M Trimmed moebius inversion and graphs of bounded degree Theory Comput Syst 47, (2010), 637–654

[2] DALVI, N N., AND SUCIU, D The dichotomy of probabilistic inference for unions of conjunctive queries J ACM 59, (2012), 30

[3] NEDERLOF, J Fast polynomial-space algorithms using

măobius inversion: Improving on steiner tree and related problems In Automata, Languages and Programming, 36th International Colloquium, ICALP 2009, Rhodes, Greece, July 5-12, 2009, Proceedings, Part I (2009), S Albers, A Marchetti-Spaccamela, Y Matias, S E Nikoletseas, and W Thomas, Eds., vol 5555 of Lecture Notes in Computer Science, Springer, pp 713–725

[4] POINCARÉ, H Sur la généralisation d’un théorème d’Euler relatif aux polyèdres Comptes rendus hebdomadaires de l’Académie des sciences de Paris 117 (1893), 144–145 [5] ROTA, G.-C On the foundations of combinatorial theory I

Theory of Măobius functions Z Wahrscheinlichkeitstheorie und Verw Gebiete 2 (1964), 340–368 (1964)

[6] ROTA, G C., AND PALOMBI, F Indiscrete thoughts

Birkhauser, 1996

[7] SAUER, N On the density of families of sets J

Combinato-rial Theory Ser A 13 (1972), 145–147

[8] SHEL AH, S A combinatorial problem; stability and order for models and theories in infinitary languages Pacific J. Math 41(1972), 247–261

[9] STANLEY, R P Enumerative combinatorics Vol 2, vol 62 of

(52)

Tạp

chí

onl

ine

của

cộng

đồng

những

người

yêu

T

oán

[10] STANLEY, R P Enumerative combinatorics Volume 1,

sec-ond ed., vol 49 of Cambridge Studies in Advanced Mathe-matics Cambridge University Press, Cambridge, 2012 [11] VAN LINT, J H., AND WIL SON, R M A course in

combina-torics, second ed Cambridge University Press, Cambridge, 2001

[12] VAPNIK, V N The Nature of Statistical Learning Theory

Springer-Verlag New York, Inc., New York, NY, USA, 1995 [13] VAPNIK, V N., ANDCHERVONENKIS, A Y Theory of uniform

(53)

T p c h í

online

cộng đồng

những n g i y ê u T o n

Tạp

chí

onl

ine

của

cộng

đồng

những

người

yêu

T

ốn CÁC BÀI TỐN ĐỘI NĨN

Đặng Nguyễn Đức Tiến (Đại học Trento, Italy)

Trong số Epsilon, giới thiệu với độc giả 15 tốn đội nón sưu tập 50 năm (từ 1961 đến 2013), phân bố mức độ khó thể loại khác Để tiếp tục hành trình, chuyên mục phần chúng tơi chọn lọc trình bày lời giải đẹp cho tốn trình bày Chúng giới thiệu với độc giả lời giải tổng quát ứng dụng thực tế cho vấn đề từ tốn giải trí

1 Bài tốn nón

Nhắc lại đề: Có người chơi, người đội ngẫu nhiên nón có màu đỏ xanh dương Họ nhìn thấy màu nón của2bạn khơng biết màu Mỗi người cần phải đốn màu nón mình, chọn bỏ qua khơng đốn Nếu người đốn màu nón người cịn lại khơng đốn sai, họ thắng trị chơi Họ thua có người đốn sai chọn bỏ qua Họ trao đổi chiến thuật với trước chơi tham gia khơng trao đổi thơng tin Tìm chiến thuật có xác suất thắng cao

Để bắt đầu hành trình, chúng tơi mời q độc giả quay lại nhóm tốn đội nón số 4, số 5, số

Chúng ta bắt đầu toán số đội nón số trước Theo luật chơi, ta thấy chọn ‘bỏ qua’ họ thua tình huống, vậy, cách trả lời dễ thấy phải có người gọi lên màu nón Xét trường hợp người chơi chọn màu, chọn ngẫu nhiên mà không ‘bỏ qua’ xác suất để thắng 1/8, tức 12.5% có tất khả khác cho người Một

(54)

Tạp

chí

onl

ine

của

cộng

đồng

những

người

yêu

T

oán

suất thắng 1/2 = 50% Liệu có phải chiến thuật tốt nhất?

Như gợi ý tốn đội nón số 6, chiến thuật chưa phải chiến thuật tốt Lời giải sau cho xác suất thắng lợi cao hơn: người thấy người đội nón khác màu nhau, họ chọn “bỏ qua" thấy người đội nón trùng màu nhau, chọn màu ngược lại

Với cách này, họ ln đốn 6/8 trường hợp sai 2/8 trường hợp nón màu (xem ví dụ Hình 6.1)

Hình 6.1: Mỗi người thấy nón khác màu chọn ’bỏ qua’ thấy nón màu, chọn màu ngược lại Trong ví dụ này, người đội nón đỏ đốn màu đỏ (do thấy nón xanh) người đội nón xanh chọn bỏ qua (do họ thấy nón khác màu nhau)

Lời giải cho khả thắng lợi 6/8 = 75% nón đội thật ngẫu nhiên Tuy nhiên, thực tế (như mơ tả tốn đội nón số 6), người dẫn trị nhanh chóng nhận chiến thuật họ đội nón cho họ màu với tần suất nhiều hơn, chí ln ln đội màu Khi khả chiến thắng từ 75% giảm xuống thành 0%! Vậy người chơi phải đối phó vớingười dẫn trị “xảo quyệt”?

(55)

Tạp

chí

onl

ine

của

cộng

đồng

những

người

yêu

T

oán

cuộc chiến người chơi người dẫn trị Trung bình chiến thắng 58% Liệu có cách làm tốt 58% để đối phó với người dẫn trò xảo quyệt?

Rất thú vị nâng cấp chiến thuật cũ để bảo toàn khả chiến thắng 75% Chiến thuật sau: Chúng ta có cách đội nón khác cách ghép thành cặp “đối xứng" nhau, đỏ xanh Ví dụ nón đỏ-đỏ-đỏ ĐĐĐ, nón xanh-xanh-xanh XXX, ĐXĐ XĐX Tưởng tượng hình lập phương với tọa độ đỉnh tương ứng với cách đội nón, đỉnh nối với có nón khác màu Các cấu hình đội nón “đối xứng" đỉnh đối xứng hình lập phương

ĐĐĐ

ĐXĐ ĐĐX

XĐĐ XĐX ĐXX

XXĐ

XXX

Chiến thuật chọn hi sinh cặp đỉnh đồng màu

ĐĐĐ

ĐXĐ

ĐĐX

XĐĐ

XĐX

ĐXX

XXĐ XXX

(56)

Tạp

chí

onl

ine

của

cộng

đồng

những

người

yêu

T

oán

thuật câu hỏi chọn cặp đỉnh XXX ĐĐĐ dễ dàng bị “bắt bài" người dẫn trị chơi (Hình 6.2) Nhưng ta chọn cặp đỉnh (XĐX ĐXĐ), hay (XXĐ ĐĐX) có xác suất thắng lợi 75% cho trường hợp đội nón ngẫu nhiên Lúc này, người dẫn chương trình khơng thể "bắt bài" nhóm người chơi linh động thay đổi cặp đỉnh “hi sinh" ngầm định, buộc người dẫn trò phải đội nón cách ngẫu nhiên cho họ, chiến thắng đến với xác suất 75%! Một chiến thuật ví dụ cho trường hợp chọn cặp XĐX ĐXĐ (xem Hình6.2 bên phải) sau:

• Người 1: gặp xanh - đỏ chọn xanh, điều tương ứng với cạnh ĐXĐ - XXĐ (màu đỏ hình), cặp đỉnh “hi sinh” có chứa ĐXĐ nên ta chọn xanh Nếu gặp đỏ - xanh chọn đỏ, điều tương ứng với cạnh màu xanh hình Người chọn bỏ qua thấy nón màu (vì cạnh nối đỉnh khơng thuộc tập “hi sinh")

• Người 2: gặp nón màu, chọn màu đó, ngược lại chọn bỏ qua

• Người 3: gặp xanh - đỏ chọn đỏ; gặp đỏ - xanh chọn xanh, bỏ qua màu

Xét cụ thể cách đội nón có, ta có kết sau:

Nón 1 Nón 2 Nón 3 Đốn 1 Đốn 2 Đốn 3 Kết quả

Xanh Xanh Xanh Bỏ qua Xanh Bỏ qua Đúng

Xanh Xanh Đỏ Xanh Bỏ qua Bỏ qua Đúng

Xanh Đỏ Xanh Đỏ Xanh Đỏ Sai

Xanh Đỏ Đỏ Bỏ qua Bỏ qua Đỏ Đúng

Đỏ Xanh Xanh Bỏ qua Bỏ qua Xanh Đúng

Đỏ Xanh Đỏ Xanh Đỏ Xanh Sai

Đỏ Đỏ Xanh Đỏ Bỏ qua Bỏ qua Đúng

Đỏ Đỏ Đỏ Bỏ qua Đỏ Bỏ qua Đúng

(57)

Tạp

chí

onl

ine

của

cộng

đồng

những

người

u

T

ốn

Tổng qt hóa cho 2n ´1 người chơi

Tiếp theo, xin mời độc giả xem lời giải mở rộng cho toán với trường hợp N=2n´1 người chơi, cũng câu trả lời cho tốn đội nón số thay n=4 Lưu ý lời giải đẹp tìm thấy trường hợp tổng quát đặc biệt này, tứcN=2n´1 Với trường hợp khác, lời giải bỏ ngõ Chúng tơi trình bày cách làm cụ thể cho trường hợp sau liên kết cách làm với mã Hamming, mã sửa lỗi tuyến tính tiếng sử dụng rộng rãi viễn thông

Cách làm sau: gán thứ tự cho người chơi từ đến N, đổi số sang nhị phân Chúng ta sử dụng phép sánh khác (XOR)1 trên chuỗi nhị phân Đặc tính quan trọng

của phép XOR với số k bất kỳ, k‘k = 0, k khớp với bit Gọi T tổng (XOR) tất người chơi có nón màu đỏ Chiến thuật người chơi đốn cho T

khác đó, họ thắng T thật khác (tất nhiên, họ sai T =0)

Cụ thể chiến thuật sau: Gọi tk tổng XOR người đội nón đỏ mà người thứ k thấy

• Nếu tk =0, đốn “đỏ" (vì đốn xanh,

T khơng cập nhật bằng0)

• Nếu tk =k, đốn “xanh” (vì đốn đỏ T =tk‘k=k‘k=0)

• Ngược lại, tk ‰0 vàtk ‰k, chọn bỏ qua

Lúc ta thấy, T thật 0, tất đoán sai Nếu T = k ‰ 0, người thứ k đoán tất người khác chọn bỏ qua, nên họ chiến thắng VìT có giá trị từ đến Nnên xác suất thắng lợi N/(N+1)

Liệu chiến thuật nâng cấp lên để đối phó với người dẫn trị “xảo quyệt"? Câu trả lời có với cách làm tương tự phần trình bày

1Phép sánh khác (thuật ngữ tiếng Anh “Exclusive OR", hay gọi là

(58)

Tạp

chí

onl

ine

của

cộng

đồng

những

người

yêu

T

ốn

MÃ HAMMING VÀ BÀI TỐN ĐỘI NĨN SỐ

Giả sử tơi muốn gửi thông điệp cho bạn thông điệp mã hóa thành bit nhị phân (ký số 1) Tuy nhiên đường truyền có kẻ can thiệp, thay đổi bit trước bạn nhận thơng điệp Vì vậy, tơi phải tìm cách thêm vào số thông tin bổ sung cho thơng điệp cho nhận được, bạn phát thông điệp bị thay đổi nữa, bạn khơi phục lại thơng điệp gốc Một phương pháp để phòng chống việc sửa đổi sử dụng “mã tái diễn”: gửi lần bit giống cho bit mà muốn gửi cho bạn: 000 111 Lúc này, dù kẻ can thiệp có thay đổi bit nào, khôi phục lại Cách làm đạt mục đích, hiệu khơng cao, nội dung thừa chiếm đến lần nội dung thông tin cần gửi Mã Hamminga đưa cách tốt cho việc sửa lỗi này, trong 2k–1 bit thơng tin gửi có k bit bổ sung, cịn lại

2k–k–1bit liệu Ví dụ với k = 4, gửi thơng điệp có chiều dài 15 bit 11 bit liệu (so với bit liệu phương pháp sử dụng mã tái diễn) Chi tiết cách hoạt động mã Hamming độc giả dễ dàng tìm thấy Wikipedia Ở đây, chỉ mối quan hệ Hamming tốn đội nón số

Một đặc tính đáng ý mã Hamming chuỗi (có chiều dài 2k´1) thông điệp đúng,

hoặc bị sai bit Điều khác biệt với mã tái diễn mã tái diễn có số thơng điệp khơng xuất Ví dụ, bạn khơng nhận mã 001011 có đến bit bị thay đổi Ở mã Hamming, chuỗi có khả xuất

(59)

Tạp

chí

onl

ine

của

cộng

đồng

những

người

yêu

T

oán

hiện KHƠNG PHẢI "thơng điệp" Cụ thể: người chơi thấy bit tương ứng với có giá trị biến chuỗi thành thông điệp đúng, chọn "đỏ" (ứng với bit 1), ngược lại Nếu thấy làm thành "thông điệp" đúng, chọn bỏ qua Chiến thuật gặp thất bại "thông điệp" ban đầu thành cơng "thơng điệp" sai Vì thơng điệp có tương ứngNthơng điệp sai, nên xác suất thắng lợi làN/(N+1), tương tự chiến thuật nêu tốn đội nón số

aMã Hamming đặt theo tên nhà toán học người Mỹ,Richard

Wesley Hamming(1915 - 1998)

2 Các tốn Konstantin Knop

Chúng tơi mời độc giả tiếp tục đến phần lời giải toán tuyệt đẹp Konstantin Knop đề xuất: tốn đội nón số tốn đội nón số

Như đề cập đến Epsilon số 1, nhóm tốn mà người chơi xếp thành hàng nên số lượng nón mà người chơi quan sát gặp giới hạn Điều khơng xảy nhóm tốn đội nón người chơi thấy nón người chơi khác (trừ nón mình) Tuy nhiên, nhóm tốn trước, người chơi khơng biết người khác đốn màu gì, cịn nhóm tốn này, thứ tự nêu lên màu nón thuộc chiến thuật chơi Để tiện theo dõi, chúng tơi nhắc lại đề tốn đội nón số cho trường hợp màu:

Có 100 người xếp thành hàng, người đội nón chọn ngẫu nhiên màu cho trước Mỗi người nhìn thấy màu nón người đứng trước mà khơng thấy nón người đứng sau Lần lượt người phải đốn màu nón hơ to cho người khác nghe Người đứng cuối (là người thấy màu nón tồn

(60)

Tạp

chí

onl

ine

của

cộng

đồng

những

người

yêu

T

oán

Ở đây, sau quen thuộc với nhóm tốn phần 1, ta thấy dạng mã sửa sai Cụ thể toán này, chiến thuật sử dụng modulo áp dụng Chúng tơi trình bày chiến thuật tổng quát cho toán với M người chơi Nmàu nón, chiến thuật sau:

• Gán màu tương ứng với số từ0 đến N´1 • Người tính tổng tất màu nón mà

quan sát được, lấy modulo theoNvà hơ màu nón tương ứng

• Những người tiếp theo, vào lời hô người trước đó, tổng người đứng trước đốn màu nón

Như vậy, cách làm có người có khả đốn sai, có N màu khác nhau, nên khả tất người đoán phụ thuộc vào khả người đầu đoán đúng, khả 1/N

Hãy thử xét ví dụ đơn giản với người chơi, màu nón người đội nón (1, 2, 0, 2, 1):

• Người đầu tiên, quan sát trước có nón

(2, 0, 2, 1), có tổng 2+0+2+1 ”2 (mod 3), người hơ có màu (và sai)

• Người tiếp theo, quan sát thấy trước có (0, 2, 1) vào lời đốn người trước 2, nên đốn

2´0´2´1”2 (mod 3)(đúng)

• Người thứ 3, quan sát thấy (2, 1) vào lời đoán người trước(2, 2), nên đoán 2´2´2´1”0 (mod 3)

(đúng)

• Người thứ 4, quan sát thấy (1) vào lời đoán người trước (2, 2, 0), nên đoán 2´2´0´1” (mod 3)

(đúng)

• Người cuối cùng, vào lời đoán người trước

(2, 2, 0, 2), nên đoán là2´2´0´2”1 (mod 3) (đúng) Kết người sau ln ln đốn người xác suất 1/3

(61)

Tạp

chí

onl

ine

của

cộng

đồng

những

người

u

T

ốn

Có 100người xếp thành hàng, người đội nón số 101 nón khác màu Mỗi người thấy nón người đứng trước khơng thấy nón người đứng sau Lần lượt người từ sau trước phải đốn màu nón hơ to cho người nghe Màu hô không hô lại Người chơi không trao đổi thông tin với Tìm chiến thuật cho khả tất đốn cao

Thoạt nhìn, toán giống nhau, thật giải thấy khác, đẹp riêng biệt Trước vào phân tích lời giải cho này, xin nhắc lại hai khác biệt tốn đội nón số so với tốn số 7:

• N người chơi, người đội nón số N+1 nón khác màu Điều có nghĩa màu nón người khác Ở số 7, ta khơng có ràng buộc số người chơi số màu nón, họ đội nón giống màu

• Màu hô không hô lại Ở số 7, điều kiện khơng có

Như phân tích trên, ta thấy áp dụng chiến thuật cũ với modulo 101, điều kiện không cho phép màu hô dùng lại nên chiến thuật gặp trở ngại! Vì người chơi (đứng cuối hàng) sau chọn màu nón (theo modulo), khả trùng với màu người đứng trước cao, lên đến99/101, nên nói gần có người biết rõ màu nón mình, khơng thể hô! Và gặp bế tắc đây!

(62)

Tạp

chí

onl

ine

của

cộng

đồng

những

người

yêu

T

oán

Như với chiến thuật này, không người phải hi sinh để cứu người khác Trong số này, "anh hùng", người cuối hàng (người đốn đầu tiên) biết màu nón với xác suất 1/2 Bằng cách thơng báo thấy, hi sinh hội Hai người lại "ra đi" trước họ phải đốn

Liệu phương án trình bày tối ưu? Câu trả lời chưa Bạn đọc hẵn đặt câu hỏi chúng tơi (và tác giả toán) bàn phương án này, chưa phải lời giải cần tìm? Đơn giản đẹp giải tổng quát với M > N nón

Để đến đến lời giải tối ưu cho toán này, xin mời độc giả rời xa modulo mà chuyển sang phương thức khác: sử dụng hoán vị!

Hãy tưởng tượng, ta có thêm người chơi "ảo", đội nón cịn lại đứng sau người cuối hàng Như vậy, tồn cách đội nón trở thành hoán vị 101 số!

Làm để người chơi (khơng ảo) truyền tải thơng tin cho người khác đốn hoán bọn họ? Trước mặt 99 người, nên có hốn vị có quan sát từ vị trí Với bit thơng tin (ứng với hốn vị) vậy, họ chiến thắng? Câu trả lời chọn màu cho hoán vị tương ứng hoán vị chẵn (even permutation)2 Căn vào người chơi đốn màu nón mình, nằm số màu mà họ chưa nghe thấy chưa quan sát thấy Nhờ vào hoán vị chẵn, họ chọn màu nón mình! Một lời giải đơn giản tuyệt đẹp!

Chúng ta xét ví dụ cụ thể sau: Giả sử có người chơi nón:

• Giả sử người đứng cuối hàng thấy nón người đứng người 4, nghĩa hốn vị có có dạng [* * 2] Cụ thể, có hốn vị có

2Hốn vị chẵn (even permutation) hốn vị có số lượng nghịch thế

(63)

Tạp

chí

onl

ine

của

cộng

đồng

những

người

yêu

T

oán

là [1 2] [3 2] Vì [1 2] hoán vị chẵn ([3 2] hoán vị lẻ), nên người hơ có màu (và gán cho người "ảo")

• Lúc này, người có thơng tin cho [* * 2], nên có hốn vị tương ứng [1 2] [4 2], nên chọn [1 2] [1 2] hoán vị chẵn [4 2] hốn vị lẻ (có nghịch thế) Vì vậy, hơ màu cho Và kết

• Người cuối cùng, lắng nghe thông tin từ người trước, nên có [* *] có hốn vị để chọn [1 2] [2 1] Ở lần chọn [1 2] hốn vị chẵn ([2 1] có nghịch thế, nên hốn vị lẻ) nên hơ màu cho

Vậy theo chiến thuật này, chắn N´1 người sau đoán màu nón người có khả để chọn nên khả chiến thắng cho tất người 1/2 Tiếp theo, để kết thúc cho chun mục "Các tốn đội nón", chúng tơi trình bày phần lời giải cho trường hợp khó với tốn vơ hạn nón, phần lớn phải sử dụng đến hỗ trợ máy tính

3 Bài tốn vơ hạn nón

Trong phần này, tham khảo gần dịch lại phần lớn ghi từ toán #1179 chuyên mục “Câu đố hàng tuần” nhà toán học Stan Wagon (đại học Macalester) Như phần trước, đăng lại đề để độc giả tiện theo dõi

Bài toán đề Lionel Levine (đại học Cornell) vào năm 2011 sau: Bốn người tham gia trị chơi đốn nón sau: Người dẫn trị đội vơ hạn nón có màu trắng đen lên đầu người với xác suất nón trắng đen 12 Các nón người đánh số

1, 2, Mỗi người chơi thấy toàn nón người khác nón họ khơng thấy

(64)

Tạp

chí

onl

ine

của

cộng

đồng

những

người

yêu

T

oán

ghi giấy người

Nếu cả4người đoán (tức là4người ghi số ứng với nón màu đen mình), họ thắng trị chơi, ngược lại, cần người đốn khơng đúng, họ thua Bốn người thảo luận trước chiến thuật trước chơi khơng có trao đổi sau Họ khơng biết thời điểm mà người khác đưa giấy cho người dẫn trị Hãy tìm chiến thuật để xác suất thắng cao

Ví dụ: họ ghi số2015vào mảnh giấy Khi đó, hội chiến thắng là1/16vì xác suất nón thứ2015của người là1/2

3.1 Chiến thuật New Zealand

Lời giải trình bày Stan Wagon gọi chiến thuật New Zealand (vì đội bóng bầu dục New Zealand có tên biểu tượng ALL-BLACKS) Chiến thuật sau:

Người thứ jsẽ tìm số #j nhỏ cho tồn nón thứ

#jcủa tất người quan sát có màu đen Người này đốn màu nón thứ #j của màu đen.

Chiến thuật thành cơng vị trí tất đội nón đen thất bại có người đội nón trắng Vì có n khả có người đội nón trắng khả tất đội nón đen, nên xác suất thành cơng chiến thuật 1/(n+1)

(65)

Tạp

chí

onl

ine

của

cộng

đồng

những

người

yêu

T

oán

là Đen-Trắng Trắng-Đen, họ thất bại Như xác suất để chiến thắng 1/3

Stan Wagon, sau nhận chiến thuật tìm số #j nhỏ có k nón trắng thành công với xác suất 1/(n+1) với k ăn Chiến thuật New Zealand với mở rộng đồng nghĩa với k = Tư tưởng gần với phần chiến thuật đối phó với người dẫn trị "xảo quyệt"

Đây có phải chiến thuật tốt nhất? Câu trả lời chưa phải, chiến thuật đơn giản có hiệu tốt Theo Stan Wagon, vớin=2vàn=3, hiệu chiến thuật so sánh với chiến thuật phức tạp Tuy nhiên với n tổng quát, chiến thuật đưa Peter Winkler thành công với xác suất xấp xỉ 1/log(n), n lớn vượt trội chiến thuật New Zealand

3.2 Chiến thuật Winkler (2011)

Chiến thuật sau: tất người chơi thống với sốtvà người đốn dựa tiêu chí sau đây:

Tiêu chí (A). Trong số t nón người có nón đen

Tiêu chí (B). Tổng số nón đen đầu người chia hết cho t

Người chơi thứ j quan sát thấy tồn n´1 nón đen

của người chơi khác nên có t áp dụng tiêu chí trên, nhanh chóng tính số nón đen Và số khoảng từ1đếnt

Xét tiêu chí (A) với n = 1000 người chơi giá trị t = 13, xác suất 13 nón đầu có nón đen xuất người là1´(1/2)13và xác suất cảnngười có nón đen

số t nón đầu là(1´(1/2)13)1000 =0.885, nghĩa xấp xỉ 88.5%.

Xác suất tiến dần đến xấp xỉ 100% t tăng

(66)

Tạp

chí

onl

ine

của

cộng

đồng

những

người

yêu

T

oán

Như vậy, chiến thuật Winkler đề thành công (A) (B) đúng, nghĩa xác suất thành công là:

2t

n

t

nếu ta xét (A) (B) biến cố độc lập (mặc dù thực tế (A) (B) khơng hồn tồn độc lập)

Để tìm cực trị cho(1´21t)n1t, ta xét đạo hàm:

B Bt

(1´21t)n

t

!

=(1´2

´t)n(ntlog(2)

´2t+1) (2t´1)t2

Vìtą0vàną0nên đạo hàm khi:

ntlog(2)´2t+1=0ô2t=ntlog(2) +1

Ta có khai triển MacLaurin của2tlà:

2t=

∞ ÿ

n=0

tnlogn(2) n!

Giá trịtthỏa mãn xấp xỉt«log2(2n´2)

Câu hỏi đặt với chiến thuật vậy, xác suất chiến thắng bao nhiêu? Và giá trị t

cần chọn bao nhiêu? Để chiến thuật Winkler thành công với xác suất cao nhất, cần tìm giá trịtsao cho xác suất (A) (B) đồng xảy đạt giá trị cực đại

Ở đây, tiêu chí (B) cần quan tâm cao có khả giá trị tối ưu đạt tổng số có giá trị khác (mod t), nghĩa xác suất tối ưu xảy tổng đồng dư với s(mod t) ‰0nào Chiến thuật Winkler trở thành tìm cặp giá trị (t,s) cho giá trị xác suất đạt cực đại với đầu vào n cho trước Ta tìm giá trị thơng qua thuật tốn sau:

Gọi F(n,t,s) số cách đội nón cho n người với t nón cho tiêu chí (A) (B) thỏa Khi đó, xác suất thành cơng F(n,t,s)/2tn

Fcó thể xây dựng đệ quy sau:

• F[1, t, 0] = 1, lưu ý, cần hiểu làs=t, với n=1

thì có khả nón vị trít nón có có màu đen

• F[1, t, s] =2t´s, cần nón vị trí s là nón đầu tiên màu đen, nón sau có màu tùy ý

(67)

Tạp

chí

onl

ine

của

cộng

đồng

những

người

yêu

T

oán

chỉ đệ quy dựa kết bước trước Một số kết tính tốn với chiến thuật Winkler:

• n = n = 3: chiến thuật Winkler không tốt chiến thuật New Zealand Cụ thể xác suất tối ưu cho n =

F(2, 2, 0)/16=5/16ă1/3 vàn=3 làF(3, 3, 0)/512 =121/512ă 1/4

• n = 4, chiến thuật Winkler nhắm tới s = không nhỉnh New Zealand, với kết tối ưu t =4 vàs =1, xác suất 421/2048, hay 0.2056, khoảng 3% nhiều 1/5

• Với n lớn, chiến thuật Winkler vượt trội chiến thuật New Zealand Với n = 1000, (t,s) tối ưu (13, 10) xác suất tính xấp xỉ 0.068, tức 68 lần tốt xác suất 1/1001 chiến thuật New Zealand

3.3 Những ý tưởng mới

Chiến thuật Winkler mặc kết tốt vớinlớn, nhiên, chưa phải chiến thuật tốt Năm 2014, Larry Carter, J-C Reyes, Joel Rosenberg (San Diego) đề xuất hai chiến thuật sau:

Chiến thuật “sửa sai”3 Ý tưởng chiến thuật sau:

nếu người nhận có đồng đội thất bại tuân theo chiến thuật Winkler, lúc người nhận họ thất bại cứng nhắc tuân theo chiến thuật gốc, thay áp dụng đoạn từ đến t chiến thuật Winkler, người sửa sai cách áp dụng chiến thuật đoạn từt+1đến2t Và vậy, người nhận áp dụng chiến thuật đoạn đến t, họ sửa sai cách nhảy lên đoạn Chiến thuật thất bại trường hợp có người chơi khơng thể tìm thấy lỗi Ví dụ người A có tồn t nón đầu màu trắng, lúc A khơng thể nhận cần phải sửa lỗi Kiểm tra máy tính, chiến thuật thật tốt chiến thuật Winkler, với(n=4,t=4,s=1), chiến thuật nâng xác suất thành công từ 0.205566 lên 0.210090

3Thuật ngữ gốc The “reset” strategy, gọi “sửa sai” dựa

(68)

Tạp

chí

onl

ine

của

cộng

đồng

những

người

yêu

T

oán

Chiến thuật hoán vị Ý tưởng thay tn theo chiến thuật (B) tính tổng tồn nón đen đồng đội họ tính tổng từ hốn vị khác Điều buộc phải sử dụng thêm giả định người chơi ngầm định người có thứ tự khác biệt Sử dụng trợ giúp máy tính, Stan Wagon cho kết với n =3, t =4 hoán vị (342), kết đạt 263/1024 = 0.256, cao 1/4 chiến thuật New Zealand 121/512 = 0.236 chiến thuật Winkler Với chiến thuật “sửa sai”, kết 0.259

J-C Reyes Joel Rosenberg tiếp tục đề xuất chiến thuật đạt kết tốt cho trường hợpn=3 Stan Wagon khơng giải thích phương pháp đạt hiệu cao vậy! Tuy nhiên, kiểm chứng máy tính cho thấy kết thật tốt Cụ thể xác suất với n = 0.2617, cao toàn chiến thuật giới thiệu tính đến

Phương pháp dựa ma trận kích thước8ˆ8được xây dựng trước:

2, 1, 4, 3, 5, 6, 8, 7, 1, 3, 2, 5, 4, 7, 6, 8, 3, 2, 1, 4, 6, 5, 7, 4, 5, 4, 6, 2, 1, 3, 1, 2, 4, 5, 7, 1, 3, 2, 8, 6, 7, 6, 5, 8, 2, 1, 4, 3, 6, 8, 3, 7, 1, 4, 1, 2, 8, 7, 2, 6, 5, 6, 2,

Ma trận A

2, 1, 3, 5, 4, 6, 7, 8, 1, 3, 2, 4, 6, 5, 8, 7, 3, 2, 1, 1, 5, 4, 6, 6, 5, 4, 1, 3, 2, 7, 6, 5, 4, 5, 6, 2, 1, 3, 7, 7, 7, 6, 4, 8, 3, 1, 2, 5, 6, 8, 5, 7, 2, 2, 3, 1, 8, 7, 8, 6, 1, 3, 1,

Ma trận B

2, 1, 3, 5, 4, 6, 7, 8, 1, 3, 2, 4, 6, 5, 8, 7, 4, 2, 1, 6, 5, 8, 8, 6, 4, 5, 4, 2, 1, 7, 6, 7, 5, 4, 6, 1, 2, 3, 6, 1, 6, 7, 5, 3, 8, 2, 1, 4, 8, 6, 7, 6, 3, 1, 4, 2, 7, 8, 4, 5, 2, 4, 2,

Ma trận C

(69)

Tạp

chí

onl

ine

của

cộng

đồng

những

người

yêu

T

oán

Chiến thuật Winkler với (n =3,t =3,s=0) tương ứng với chiến thuật với ma trận A = B = C

1 3 2

Để tổng kết phương pháp, sử dụng lại hình ảnh tóm tắt Stan Wagon trình bày lại Hình 6.3

Hình 6.3: Hiệu chiến thuật dùng để giải tốn vơ hạn nón Nguồn: [5]

3.4 Trường hợp n = 2

(70)

Tạp

chí

onl

ine

của

cộng

đồng

những

người

yêu

T

oán

Trắng-Trắng-Đen chọn A thấy Trắng-Đen-Đen Trắng-Đen-Trắng chọn Và B làm tương tự Chiến thuật này, Stan Wagon gọi chiến thuật 3-nón4

Nếu A thấy nón đầu B màu trắng, A biết chiến thuật không sử dụng được, A sửa sai cách di chuyển quan sát lên nón từ đến lại áp dụng luật

Kiểm tra vét cạn (Brute-force), không áp dụng sửa sai, chiến thuật cho kết 22/64 = 0.34375 với sửa sai, kết 22/63=0.3492064 Stan Wagon thử nghiệm với ma trận khác cho A B, kết không cao Các mở rộng với giá trị củatvớin=2vàn=3được khảo sát máy tính kỷ lục cho n=2,t =8 22937/65535 = 0.3499961 0.35 chặn cho trường hợp vơ hạn nón có người chơi

Tiefenbruck, Reyes, Carter, Rosenberg đưa cách lý luận đơn giản cho kết 0.35 sau: bắt đầu với chiến thuật 3-nón 11213321 với xác suất 22/64 Nếu A thấy nón B tồn trắng toàn đen, A bỏ qua xem xét nón tiếp tục B làm tương tự Nghĩa họ áp dụng chiến thuật 3-nón thấy nón có màu khác nhau, vậy, có 60 khả mà A B phải đốn (khơng bỏ qua) Và họ thắng lợi với xác suất 21 tổng số 60 trường hợp này, trường hợp so với chiến thuật 3-nón (là trường hợp nón người màu đen) Và vậy, xác suất chiến thắng là21/60=0.35

4 Với đen trắng, chiến thuật 3-nón tóm tắt sau:

(0, 0, 0)Đ1,

(0, 0, 1)Ñ1,

(0, 1, 0)Ñ2,

(0, 1, 1)Ñ1,

(1, 0, 0)Ñ3,

(1, 0, 1)Ñ3,

(71)

Tạp

chí

onl

ine

của

cộng

đồng

những

người

yêu

T

oán

3.5 Sáng tạo bất tận

Liệu nâng cao hiệu cho toán? Để giải đáp câu hỏi này, trước tiên cần phát biểu lại toán cách hệ thống Thử xét với trường hợp n=2:

GọiPlà tập tất chuỗi 1; Nlà tập số tự nhiên Khi đó, cách đội nón số phần tử PˆP Một chiến thuật hàm S : PˆP Đ N, khơng quan tâm hàm có khả dựng hay khơng (hay dễ hiểu không quan tâm đến phức tạp hàm này) Và với lý thuyết tập hợp dựa tiền đề chọn6, tồn một

chiến thuật mà người chơi sử dụng họ đưa tập PˆPvề chiến thắng với độ đo ngồi Điều có nghĩa ln tồn chiến thuật mà người chơi không thua! Hoặc, quay lại nghi ngờ kinh điển Toán học tiên đề chọn với tranh cãi vô tận thân vấn đề! Như vậy, tồn chiến thuật chiến thắng (Lưu ý: tiên đề chọn sử dụng), với trường hợp n =2, chiến thuật tốt ghi nhận 35% với ną2, kết

quả cịn thấp nhiều Điều có nghĩa tốn cịn để mở mong chờ đóng góp từ người u tốn học

4 Lời kết

Như với chun mục tốn học giải trí đăng tải số Epsilon, giới thiệu tương đối trọn vẹn "50 năm tốn đội nón" Thơng qua tốn này, hi vọng mang đến cho độc giả không

5Kết chứng minh Freiling với tên gọi định lý C Freiling,

có thể tham khảo liên kết cuối

6Tiên đề chọn, hay gọi tiên đề Zermelo-Fraenkel đưa bởi

Zermelo vào năm 1904, tiên đề khẳng định với họ tập hợp tùy ý không rỗng đôi không giao tồn tập hợp mà phần tử phần tử tập hợp họ tập hợp phần tử

Một cách phát biểu khác tiên đề chọn là: “Với họXbất kỳ tập

hợp không rỗng, tồn hàm chọnfđịnh nghĩa trênX” Đây

(72)

Tạp

chí

onl

ine

của

cộng

đồng

những

người

yêu

T

oán

chỉ tốn đố túy mà cịn từ cho thấy giá trị vẻ đẹp tốn học tiềm ẩn nơi

Để tiện tra cứu, độc giả truy tìm lại nguồn gốc thông qua tài liệu sau:

Phần 1: chúng tơi sử dụng chủ yếu tài liệu [1] [2] Phần tốn đội nón số 6, "người dẫn trò xảo quyệt" gặp Bài tốn theo ghi nhận chúng tơi xuất cách thức Việt Nam vào năm 2007, khóa học giáo sư Michel Waldschimidt, khoa Tốn đại học Pierre et Marie Curie, Paris Ông đến trường Đại học Khoa học Tự nhiên TP.HCM dịp giảng cho sinh viên học sinh phổ thông Năng Khiếu giảng thú vị, phần giảng này, may mắn thay cịn tìm thấy

Phần 2: Chúng sử dụng chủ yếu từ [3] tham khảo Blog Tanya Khovanova

Phần 3:Bạn đọc xem thêm [4] đặc biệt [5] (trang gốc tác giả Stan Wagon)

Tài liệu tham khảo

[1] E Brown and J Tanton, “A dozen hat problems,” Archives of Psychology, vol 16, no 4, pp 22–25, 2009

[2] A Zorn, “Colored hats and logic puzzles,” Berkeley Math Circle, 2013

[3] T Khovanova, “A line of sages,” The Mathematical Intelli-gencer, 2014

[4] C S Hardin and A D Taylor, “An introduction to infi-nite hat problems,” The Mathematical Intelligencer, vol 30, no 30, pp 20–25, 2008

(73)

T p c h í

online

cộng đồng

những n g i y ê u T o n

Tạp

chí

onl

ine

của

cộng

đồng

những

người

yêu

T

oán BÀI TOÁN FROBENIUS

VỀ NHỮNG ĐỒNG XU

Trần Nam Dũng (Đại học KHTN, TP Hồ Chí Minh)

Nguyễn Tất Thu (THPT Chuyên Lương Thế Vinh, Đồng Nai)

Bài toán Frobenius đồng xu toán kinh điển nhắc đến nhiều viết, nhiều cơng trình nghiên cứu Một lý để đề tài tiếp tục để ý toán số kỳ thi chọn học sinh giỏi toán quốc gia năm học 2014-2015 diễn vào tháng vừa qua

Tác giả thứ báo có nhiều duyên nợ với tốn Khi cịn học sinh, anh gặp (và may mắn, giải được) toán Sau anh lại gặp lại toán nhiều lần, lúc vai trò thầy giáo, giảng viên

1 Mở đầu

Chúng ta mở đầu toán vui sau đây: Chúng ta đọc truyện xem phim Tây Du Ký Câu chuyện sau rút từ chuyến kỳ vĩ thầy trò Đường Tăng đến Tây Trúc Vừa khỏi kiếp nạn Bạch cốt tinh, thầy trị Đường tăng lại vào vương quốc mới, gọi vương quốc Ngũ Bát Sở dĩ có tên Ngân hàng trung ương Vương quốc phát hành loại tiền quan (Ngũ) quan (Bát)

Vương quốc chưa phát triển nên người dân biết phép tính cộng, khơng biết phép tính trừ Vì thế, bán hàng, đưa thừa người ta khơng trả lại (cịn đưa thiếu người ta .không chịu - khôn lắm)

(74)

Tạp

chí

onl

ine

của

cộng

đồng

những

người

yêu

T

oán

Nhân viên bảo vệ chặn lại, xem chừng không muốn cho vào Trư bát giới xơng nói:

- Sao khơng cho vào?

Tay bảo vệ tay vào số 28 (nhị thập bát) nói: Ơng có thấy số không?

- 28à 28 tuổi vào Yên tâm em Anh

360 tuổi Cịn ơng anh gãi mơng 720 tuổi Cái gánh hàng 240 tuổi Ngay ngựa

130 tuổi Trẻ có lẽ sư phụ bọn anh, ông vừa làm sinh nhật lần thứ 30 Các có cần xem chứng minh nhân dân khơng, loại nhé, có tên bố mẹ

- Không, không, tuổi,

- Đây .Trư Bát giới thể công

- Đây siêu thị mà hàng từ 28 quan trở lên Tôi thấy ông nhà quê quá, sợ không đủ tiền nên không muốn cho vào

- Ấy, đừng nghĩ Bọn anh nhà có điều kiện nhé, tiền 5quan, quan bọn anh đổi cửa ních túi

- Vậy xin mời anh vào

Bài toán mở đầu: Chứng minh thầy trị Đường tăng mua (tức trả giá tiền) hàng siêu thị "Over 28"

Về mặt toán học, toán tương đương với mệnh đề sau: Chứng minh với số nguyên Ně28, tồn tạix,yngun

khơng âm cho N=5x+8y Bài tốn trường hợp riêng toán Frobenius đồng xu mà ta nói đến Có hai cách để giải tốn này, mang tính thuật tốn

(75)

Tạp

chí

onl

ine

của

cộng

đồng

những

người

yêu

T

oán

Cách 2: Cách khó nghĩ đến đẹp Cụ thể sau: Ta chứng minh N ě 28 biểu diễn N+1

cũng biểu diễn Nếu biểu diễn N có 3số 5thì ta thay 3số 5bằng2số 8được biểu diễn N+1 Nếu biểu diễn N có số 8thì ta thay 3số 8bằng số 5thì biểu diễn N+1 Một hai TH phải xảy vỡ nu khụng cú thỡ N2ă5+2ă8=26

Vớ d trích từ giảng chúng tơi dành cho sinh viên khoa Toán-Tin học trường ĐH KHTN Tp HCM, môn Số học logic, mục thuật toán Euclide định lý Bezout Với cách phát biểu thú vị hài hước, lôi kéo sinh viên vốn sợ Số học Logic tham gia vào thảo luận tìm kiếm lời giải Thực tế, lớp sinh viên tự tìm lời giải cho tốn theo cách

Bài tốn nói trường hợp đặc biệt toán tổng quát hơn: toán Frobenius đồng xu Bài toán Frobenius đồng xu toán xác định số tiền lớn trả sử dụng đồng xu có mệnh giá cố định Ví dụ với đồng xu mệnh giá

và đơn vị số tiền lớn không trả đơn vị Số lớn với số ta gọi số Frobenius

Một cách toán học, toán phát biểu sau:

Cho số nguyên dương a1,a2,ă ă ă ,an gcd(a1,ă ă ă ,an) = 1. Tìm số ngun lớn khơng biểu diễn di dng k1a1+

k2a2+ă ă ă+knan vik1,k2,ă ă ă ,kn là số nguyên không âm Số nguyên lớn gọi số Frobenius và thường được ký hiu l g(a1,a2,ă ă ă,an)

V bi vit nói số kết liên quan đến toán tiếng

2 Định lý Sylvester toán số trong đề thi VMO 2015

(76)

Tạp

chí

onl

ine

của

cộng

đồng

những

người

yêu

T

oán

Vớia,nnguyên dương, xét phương trìnha2x+6ay+36z=n, trong đó x,y,z là số tự nhiên

1 Tìm tất giá trị củaađể với mọi ně250, phương trình đã cho ln có nghiệm(x,y,z).

2 Biết rằng aą1và nguyên tố với6 Tìm giá trị lớn nhất của ntheo a để phương trình cho khơng có nghiệm

(x,y,z).

Để giải toán trên, xét toán sau đây:

Bài toán 1. Cho a, b số nguyên dương nguyên tố nhau,bą1 Chứng minh với số nguyênN, tồn

nhất cặp số nguyên x,ythỏa mãn điều kiện

N=ax+byvà 0ďx ăb

Lời giải. ‚ Chứng minh tồn

Vì (a,b) =1 nên theo định lí Bezout, tồn hai số nguyên u,v

sao cho

N=au+bv

Mặt khác u=b.q+rvới 0ďrďb´1 nên ta có

N=a(b.q+r) +bv =a.r+b(v+a.q)

Chọn x=r,y=v+a.qta N=ax+by với0ďxďb´1

‚ Chứng minh tính

Giả sử tồn hai bộ(x;y) (x1;y1)thỏa 0ďx,x1 ăbvà

N=ax+by=ax1+by1

Hay

a(x´x1) =b(y1

´y) (1)

Do (a,b) =1 nên từ (1)ta suy b|x´x1 Lại có 0

ďx,x1

ďb´1

nên suy x=x1 và y=y1.

Vậy toán chứng minh

Bài tốn 2. (Định lí Sylvester)Choa,blà số ngun dương nguyên tố Chứng minh N0 = ab–a–b số

nguyên lớn không biểu diễn dạng ax+by với x,

ylà số nguyên không âm Hơn nữa, với p,q nguyên với

p+q = N0, có hai số p, q biểu diễn

(77)

Tạp

chí

onl

ine

của

cộng

đồng

những

người

yêu

T

oán

Lời giải. Để giải toán, cần thực hai bước sau

Bước 1:Chứng minhN0không biểu diễn dạngax+by

với x,y số nguyên không âm

Giả sử tồn hai số nguyên không âm x,ysao cho

ax+by=N0

Hay

a(x+1) +b(y+1) =ab (2)

Vì (a,b) = nên từ (2), suy x +1 b hay x +1 ě b Tương tự

y+1ěa Khi

ax+byďa(b´1) +b(a´1) =2ab´a´bąab´a´b

Điều dẫn đến mâu thuẫn Do vậyN0 không biểu diễn

dưới dạngax+by với x,yPN

Bước 2:Chứng minh với số nguyên NąN0 N biểu diễn

được dạng ax+by với x,yPN

Theo kết Bài toán 1 ta suy ra: tồn cặp (x,y)

để

N=ax+by với 0ďxăb Ta cần chứng minh yě0 Thật

y= N´ax

b ą

ab´a´b´a(b´1)

b =´1

Mà ylà số nguyên nên ta có yě0 Vậy toán chứng minh

Trở lại tốn đề thiVMO´2015 Bài tốn phát biểu thành toán tổng quát sau

Bài toán 3. Cho số nguyên dươnga,bnguyên tố Khi đóN0 =a2b+b2a´a2´b2´ab số nguyên dương lớn

không biểu diễn dạng a2x+aby+b2z với x,y,z là các

số nguyên không âm

Lời giải. Tương tự toán toán 3, ta chứng minh toán qua hai bước

(78)

Tạp

chí

onl

ine

của

cộng

đồng

những

người

yêu

T

oán

aby+b2z

Giả sử tồn số tự nhiên x, y, z để

N0 =a2x+aby+b2z

Hay

a2(b´x´1) +b2(a´z´1) =ab(y+1)

Từ đây, suy hai số b´x´1 a´z´1 có

một số dương Khơng tính tổng qt, ta giả sử b´x´1ě0 Khi đó, (a,b) =1nên ta suy b´x´1 b, dẫn tớib´x´1ěb

hay xď ´1(vơ lí)

Do N0 không biểu diễn qua a2x+aby+b2z

Bước 2:Chứng minh với số nguyên NąN0 N biểu diễn

được dạng a2x+aby+b2z.

Do NąN0 nênN´ab2+a2+ab´aąab2´b2´a nên theo định

Sylvester ta suy tồn số tự nhiên u,z cho

N´ab2+a2+ab´a=ua+b2z (4)

Vìuě0 nênu+ab´a´b+1ąab´a´bnên tiếp tục áp dụng

định líSylvester suy tồn số tự nhiên x,ysao cho

u+ab´a´b+1=xa+yb (5)

Thay (5) vào (4) ta

N´ab2+a2+ab´a=a(ax+by´ab+a+b´1) +b2z

Hay

N=a2x+aby+b2z

Bài toán chứng minh

Tại kì thi IMO năm 1983 có toán sau

Bài toán 4. Cho số nguyên dương a, b, c đôi nguyên tố Chứng minh N0 = 2abc´ab ´bc´ca

số nguyên dương nhỏ không biểu diễn dạng

abx+bcy+caz với x, y,z số nguyên không âm

Lời giải. Tương tự cách chứng minh định líSylvester, tốn chứng minh qua hai bước

Bước 1:Chứng minh N0 không biểu diễn quaabx+bcy+

(79)

Tạp

chí

onl

ine

của

cộng

đồng

những

người

u

T

ốn

Sylvester

Bước 2: Chứng minh với số nguyên dương N ą N0 N

ln biểu diễn dạng abx+bcy+caz

Vì(a;bc) =1nên theobài tốn 1tồn tạiu,ysao choau+bcy=

N với 0ď yď a´1 Tương tự, tồn v,z cho N =vb+zac

với 0ďzďb´1

Từ đó, suy bcy+caz´N ab Hay tồn số nguyênx cho

abx+bcy+caz=N

Ta cần chứng minh x ě0 Thật

x= N´bcy´caz

ab ą

2abc´ab´bc´ca´bc(a´1)´ca(b´1)

ab =´1

Suy xě0 Bài toán chứng minh

Bài toán 5. (VN TST 2000) Cho ba số nguyên dương a, b, c

đôi nguyên tố nhau.Số nguyên dương n gọi

số bướng bỉnh n không biểu diễn dạng abx+

bxy+caz với x, y, z số nguyên dương Hỏi có số bướng bỉnh

Lời giải. Để giải toán này, ta chia làm hai bước

Bước 1: Chứng minh N0 = 2abc số nguyên dương lớn

không biểu diễn dạngabx+bcy+cazvới x,y,z số nguyên dương

Việc chứng minh N0 không biểu diễn qua abx+bcy+caz

với x, y, z số nguyên dương chứng minh tương tự toán

Ta chứng minh với số ngun dươngNąN0 thìNln biểu

diễn qua abx+bcy+caz vớix,y,z số nguyên dương Trước hết ta chứng minh nhận xét:

Với ď x ď b, ď y ďa ax+by lập thành hệ thặng dư đủ theo mô đun ab

Thật vậy: Dễ thấy có tất a.b tổng ax +by với ď x ď b,

1ďyďa

Giả sử tồn ď x,x1 ď b, ď y,y1 ď a cho ax+by ”

ax1+by1( mod ab).

Hay a(x´x1) ” b(y1 ´y)( mod ab) Từ đây, suy ra x´x1 .b Mà

1ď x,x1

ď bnên suy x =x1, dẫn đến y=y1 Do nhận xét

(80)

Tạp

chí

onl

ine

của

cộng

đồng

những

người

yêu

T

oán

Theo nhận xét kết hợp với a,b,c đôi nguyên tố nên với ď x ď b, ď y ď a c(ax +by) = acx+bcy lập thành

hệ thặng dư đủ theo mơ đun ab Do với số nguyên dương N ą N0, tồn ď z0 ď a, ď y0 ď b cho

acz0+bcy0 ”N( mod ab) hay tồn số nguyên x0 cho

acz0+bcy0+abx0 =N

Lại cóacz0+bcy0 ďacb+bca=2abcăN nên ta có x0 ą0

Vậy với mọiNąN0 thìNbiểu diễn dạngabx+bcy+caz

với x,y,z số nguyên dương

Bước 2:ĐặtBlà tập số nguyên dương không biểu diễn dạngabx+bcy+caz với x,y,z s nguyờn dng V A =t1, 2,ă ă ă ,ab+bc+ca1u Y tab+bc+ca,ab+bc+ca+ 1,ă ă ă, 2abcu

D thy t1, 2,ă ă ă,ab+bc+ca1u B

tC=tab+bc+ca,ab+bc+ca+1,ă ă ă, 2abcu Ta cần tìm|BXC|

Với nPCta xét hàm f(n) =2abc+ab+bc+ca´n Ta chứng

minh

nPBôf(n)R B (1)

Với n P B, theo chứng minh suy tồn số nguyên

1ďy0 ďb, 1ďz0 ďc vàx0 cho

n=abx0+bcy0+caz0

VìnP Bnên x0 ď0 Khi

f(n) =ab(1´x0) +bc(1+a´y0) +ca(1+b´z0)R B

Giả sử tồn n R B để f(n) R B Suy tồn số nguyên dương x, y, z,x1,y1,z1 sao cho

n=abx+bcy+caz vàf(n) =abx1+bcy1+caz1

Hay

2abc= (x+x1´1)ab+ (y+y1´1)bc+ (z+z1´1)ca

Suy 2abc RBvơ lí Vậy (1) chứng minh Từ chứng minh ta suy

|BXC|= 2|C|=

2abc´ab´bc´ca+1

2

Vậy số số bướng bỉnh là: abc+ab+bc+ca´1

(81)

Tạp

chí

onl

ine

của

cộng

đồng

những

người

yêu

T

oán

3 Một hướng tiếp cận khác: Hàm sinh

Trong phần ta đưa hướng tiếp cận khác cho trường hợp n = toán Frobenius Với hướng tiếp cận này, mở rộng cho trường hợp n ě thu kết

quả thú vị, xa so với tiếp cận truyền thống trình bày phần trước

Với hai số nguyên dương nguyên tố a,b số tự nhiên n ta đặt

pta,bu(n) = (k,l)PZ

2

|k,lě0; ak+bl=n(

Ta xét tích hai cấp số nhân

1 1´za

1

1´zb = 1+z

a+a2a+

1+zb+z2b+

Nếu ta nhân tất thừa số ta chuỗi lũy thừa tổ hợp tuyến tính (với hệ số tự nhiên) a b Và zn

sẽ có hệ số pta,bu(n) Vì

1 1´za

1 1´zb =

ÿ

k,lě0

zka.zlb =

ÿ

n=0

pta,bu(n)zn

Như hàm số hàm sinh dãypta,bu(n) Ý tưởng bây

giờ ta nghiên cứu tính chất hàm số vế trái

Ta muốn tìm cơng thức đẹp cho pta,bu(n) cách xem xét kỹ

lưỡng hàm số vế trái Để dễ tính tốn ta tìm hệ số tự dãy liên quan, cụ thể, pta,bu(n) số hạng tự

của

f(z) =

(1´za) (1´zb)zn =

ÿ

k=0

pta,buz

k´n

Chuỗi cuối chuỗi lũy thừa, có chứa số hạng với số mũ âm Chuỗi gọi chuỗi Laurent, theo tên Pierre Alphonse Laurent (1813–1854) Đối với chuỗi lũy thừa, để tính số hạng tự ta cần thay z =0

(82)

Tạp chí onl ine của cộng đồng những người u T ốn

Để tính số hạng tự này, ta khai triển f thành phân số sơ cấp Để hiểu rõ cách phân tích phân số thành phân số sơ cấp, trước hết ta trường hợp chiều Ký hiệu nguyên thủy bậc acủa đơn vị

ξa=cos

a +isin

2π a

thì 1,ξa,ξ2a, ,ξaa´1 tất bậc a đơn vị Trước hết ta tìm phân tích sơ cấp cho

1´za Vì cực hàm số nằm điểm bậc a đơn vị,

1 1´za =

a´1

ÿ

k=0

Ck

z´ξk a

Các hệ số Ck tính theo cơng thức

Ck = lim zÑξk

a

(z´ξka)

1 1´za

= lim

zÑξk a

1

´aza´1 =´

ξka a ,

trong đa sử dụng quy tắc L’Hopitale Như ta có khai triển

1

1´za =´

1

a

a´1

ÿ

k=0

ξka z´ξk

a

Trở lại với toán chúng ta, cực f nằm điểmz =0

với bậc n, z = với bậc đơn vị bậc a bậc

b khác với bậc1 a,bngun tố Do đó, khai triển fthành tổng phân số sơ cấp có dạng

f= A1

z + A2

z2 + +

An

zn +

B1

z´1+

B2

(z´1)2+

a´1

ÿ

k=1

Ck

z´ξka

+

b´1

ÿ

j=1

Dj

z´ξjb (1)

Bằng phương pháp gần giống trên, ta tính

Ck =´

1

a(1´ξkba )ξ k(n´1) a

, Dk =´

1

b(1´ξjab)ξ j(n´1) b

(2)

Để tính B2, ta nhân hai vế (1) cho(z´1)và tính giới hạn

khi z Ñ1 để thu

B2 = lim zÑ1

(z´1)2

(1´za)(1´zb)zn =

1

(83)

Tạp chí onl ine của cộng đồng những người yêu T oán

bằng cách sử dụng quy tắc L’Hopitale lần Với số hạng B1, ta

tính

B1 =lim zÑ1(z´1)

(z´1)2

(1´za)(1´zb)zn ´

1

ab(z´1)2 !

=

ab´

1 2a´

1 2b´

n ab,

với việc áp dụng quy tắc L’Hopitale lần

Ta khơng cần tính hệ sốA1,A2, ,Anvì chúng đóng góp số hạng với số mũ âm mà ta bỏ qua; số hạng khơng đóng góp cho số hạng tự f Vì ta có tất hệ số khác, số hạng tự f, ta nói trên, giá trị hàm số sau tính điểm0:

pta,bu(n) =

B1

z´1+

B2

(z´1)2 +

a´1

ÿ

k=1

Ck

z´ξka

+

b´1

ÿ

j=1

Dj

z´ξjb

!ˇ ˇ ˇ ˇ ˇ

z=0

=´B1+B2 ´ a´1

ÿ

k=1

Ck

ξk a

´

b´1

ÿ

j=1

Dj

ξjb

Từ

pta,bu(n) =

1 2a+

1 2b+

n ab+

1

a

a´1

ÿ

k=1

1 (1´ξkb

a )ξkna

+1

b

b´1

ÿ

j=1

1 (1´ξjbb)ξ

jn b

(3)

Bây ta tìm cách đơn giản tổng (3) để công thức thuận tiện

Ta nghiên cứu trường hợp đặc biệt b=1 Trường hợp tính dễ dàng pta,1u(n) tính cách số điểm nguyên

đoạn:

pta,1u(n) =|t(k,l)PZ

2

:k,lě0,ak+l=nu|=|tkPZ:0ďkď n

au|=

hn

a

i +1

trong [x] ký hiệu phần nguyên số thực x Thay b=1 vào (3), ta

1 2a +

1 + n a + a

a´1

ÿ

k=1

1 (1´ξk

a)ξkna

=pta,1u(n) =

hn a i +1 Từ a

a´1

ÿ

k=1

1 (1´ξk

a)ξkna

=´ !n a ) + 2´

(84)

Tạp chí onl ine của cộng đồng những người yêu T oán

Trong đóx phần lẻ số thựcx Ta chứng minh

1

a

a´1

ÿ

k=1

1 (1´ξkb

a )ξkna

=

a

a´1

ÿ

k=1

1 (1´ξk

a)ξb ´1kn

a

, (5)

Trong b´1 là số nguyên chob´1.b”1(moda) Từ(4)và(5)

suy

1

a

a´1

ÿ

k=1

1 (1´ξkb

a )ξkna

=´ "

b´1n

a

* +1

2 ´

2a (6)

Bây thay (6)và công thức tương tự vào (3), ta công thức đẹp Tiberiu Popovicu (1906´1975) tìm

Định lý (Popovicu) Nếuavàblà số nguyên dương nguyên tố

pta,bu(n) =

n ab ´

"

b´1n

a

* ´

"

a´1n

b

* +1,

trong b´1.b”1(mod a)và a´1.a”1(mod b).

Từ định lý Popovicu, dễ dàng chứng minh bổ đề sau

Bổ đề.Nếuavàblà số nguyên dương nguyên tố nhau,

nP[1;ab´1]và bội số a hoặcb,

pta,bu(n) +pta,bu(ab´n) =1

Chứng minh. Ta có

pta,bu(ab´n) =

ab´n ab ´

"

b´1(ab´n)

a

* ´

"

a´1(ab´n)

b

* +1

=2´ n

ab´

"

´b´1n

a

* ´

"

´a´1n

a

*

=´ n

ab+

"

´b´1n

a

* +

"

´a´1n

a

*

=1´pta,bu(n)

(85)

Tạp chí onl ine của cộng đồng những người yêu T oán

4 Bài toán Frobenius cho n ě 3

Quay trở lại với toán Frobenius dạng tổng quát, ta thấy rằng, với kết định lý Sylvester, trường hợp n = giải trọn vẹn, ta có cơng thức g(a,b) =ab–a–b Chính định lý đẹp đẽ khiến cho nhà tốn học hướng ý đến việc tìm cơng thức cho hàmg(a1,a2, ,an) với ną2 Tuy nhiên tính đến kết hướng

cịn xa đến hồn hảo Ngoại trừ hai kết đẹp đẽ dành cho trường hợp “n =2.5” (bài IMO 1983 VMO

2015), thu khiêm tốn

1 Định lý Schur khẳng định với điều kiện(a1,a2, ,a2) =

1thì số Frobenius tồn

2 Với n = 3, ta có định lý Davidson đánh giá chặn cho

g(a,b,c):

g(a,b,c)ě ?

3abc´a´b´c

3 Với n bất kỳ, tìm công thức hàm g cho cấp số cộng, cấp số nhân, số đánh giá chặn trên, chặn

4 Hướng sử dụng hàm sinh n=3 cho ta công thức sau

pta,b,cu(n) =

n2 abc + n ab + ac+ bc + + 12 a+ b+ c + a bc + b ca+ c ab + + a

a´1

ÿ

k=1

1

(1´ξkba )(1´ξkca )ξkna

+

b

b´1

ÿ

k=1

1 (1´ξka

b )(1´ξkcb )ξknb

+

+1

c

c´1

ÿ

k=1

1 (1´ξka

c )(1´ξkbc )ξknc

Như vớiną2và cácaibất kỳ tính tới chưa tìm cơng thức tường minh cho hàm g(a1,a2, ,an) hướng không khả thi

(86)

Tạp

chí

onl

ine

của

cộng

đồng

những

người

yêu

T

ốn

tốn để tính số Frobenius thời gian đa thức (tính theo logarith giá trị tiền xu có liệu vào) Hiện chưa có thuật toán thời gian đa thức theo số đồng xu, toán tổng quát giá trị đồng xu lớn tùy ý tốn NP-khó

Hiện xu hướng nghiên cứu toán Frobenius chia làm hai nhánh, nhánh công thức nhánh thuật tốn, nhánh thứ hai có tốc độ tiến triển tốt hẳn

Bài tốn Frobenius có nhiều ứng dụng thú vị đời sống, ví dụ toán McNuggets sau Henri Picciotto đưa vào năm 1980 ngồi ăn tối với trai nhà hàng Mc Donald’s Các hộp bánh McDonalds thiết kế có 6,

hoặc 20 miếng bánh Hỏi mua số lượng miếng bánh mà dùng hộp chẵn Định lý Schur khẳng định số số không mua danh sách là: 1, 2, 3, 4, 5, 7, 8, 10, 11, 13, 14, 16, 17, 19, 22, 23, 25, 28, 31, 34, 37,

43 Như 43 số lớn không biểu diễn dạng

6x+9y+20z với x,y,z nguyên không âm

Tác giả thứ báo lại vừa có dun nợ với tốn, lần khía cạnh ứng dụng Trong chuyến giảng Saudi Arabia vừa qua, tác giả phát số nhiều phiếu ăn trị giá 16SR và23SR Giá tiền bữa ăn có dao động từ 16SR đến 50SR Và tác giả phải nhiều lần vận dụng tốn Frobenius để có cách chọn lựa ăn phù hợp (chỉ phải trả thêm bỏ phí 1, 2SR)

Cũng có tốn nghiên cứu mạnh mẽ lại có ứng dụng gần gũi

5 Ghi Chỉ dẫn lịch sử

1 Bài toán Frobenius đặt theo tên nhà toán học Georg Frobenius, người nêu toán giảng Phần định lý Sylvester kết dân gian kinh điển mà thường gộp chung vào định lý Sylvester Thật ra, báo mình, Sylvester chứng minh phần2 định lý

(87)

Tạp

chí

onl

ine

của

cộng

đồng

những

người

yêu

T

ốn

tìm số ngun dương z nhỏ khụng biu din c di dng tng v1+v2 +ă ă ¨+vk k ď msố (không thiết phân biệt) thuộcV

3 Định lý Bezout định lý khẳng định a,b số nguyên dương nguyên tố tồn số nguyên x,y cho ax+by = Có thuật tốn hiệu để tìm số x,y, gọi thuật tốn Euclide mở rộng Thuật tốn Euclide mở rộng có nhiều ứng dụng lý thuyết mật mã, lý thuyết phụ hồi số hữu tỷ

4 Lý thuyết hàm sinh có lịch sử truyền thống lâu đời Ở ta sử dụng chúng công cụ Những độc giả muốn nghiên cứu sâu hàm sinh ứng dụng kỳ ảo chúng tìm hiểu Gen-eratingfunctionology Herb Wilf Combinatorial Problems and Exercises Laszlo Lovasz

5 Như ta thấy, trường hợp n ě khó nhiều so với trường hợp n = Tài liệu toán Frobenius nhiều tiếp tục bổ sung cơng trình Hiện có 40 tốn mở vấn đề Xem thêm [4]

Tài liệu tham khảo

1 James J Sylvester, On Subvariants, i.e Semi-Invariants to Binary Quantics of an Unlimited Order, American Journal of Mathematics 5(1882), 79–136

2 J.J Sylvester, Mathematical questions with their solutions, Educational Times 41 (1884), 21

3 P Erdos and E.L Graham, On a linear diophantine prob-lem of Frobenius, Acta Arithmetica (1972), 399–408

4 The Coin-Exchange Problem of Frobenius, Internet resource, www.springer.com/cda/ /9780387291390-c1.pdf? J.L Ramirez-Alfonsin, Complexity of the Frobenius

(88)

Tạp

chí

onl

ine

của

cộng

đồng

những

người

yêu

T

oán

6 J.L.Jamirez Alfonsin, The Diophantine Frobenius Problem, Oxford Lectures Series in Mathematics and its Applica-tions, No 30, Oxford University Press, 2005

7 Trần Nam Dũng, Tư thuật tốn tư cơng thức, Kỷ yếu Gặp gỡ Toán học 2014

8 Trần Nam Dũng, Số học qua định lý toán, Tài liệu Internet

(89)

T p c h í

online

cộng đồng

những n g i y ê u T o n

Tạp

chí

onl

ine

của

cộng

đồng

những

người

yêu

T

oán ĐỀ THI VIỆT NAM TST 2015

NHẬN XÉT VÀ GỢI Ý GIẢI

Trần Nam Dũng

Đại học KHTN, TP Hồ Chí Minh

Nguyễn Văn Lợi

Budapest, Hungary

Nguyễn Hùng Sơn

Đại học Warsaw, Ba Lan

Tóm tắt

Kỳ thi chọn đội tuyển quốc gia Việt Nam (thường gọi tắt TST) năm 2015 chọn thí sinh dự thi Olympic Tốn quốc tế Chiang Mai, Thái Lan vào đầu tháng Tham gia kỳ thi gồm có em đoạt giải Nhất, em đạt HCV kỳ thi IMO năm trước Nam Phi 46 em đoạt giải Nhì kỳ thi học sinh giỏi Quốc gia VMO hồi đầu tháng 01 năm 2015

Để đạt giải Nhì, em cần phải đạt từ 21,5 đến 32,5 điểm (trên thang điểm 40) tức từ khoảng 4,5 đến tổng số Các đánh giá phần dựa kết em thi VMO 2015 Kỳ thi diễn ngày ĐH Sư phạm Hà Nội:

(90)

Tạp

chí

onl

ine

của

cộng

đồng

những

người

yêu

T

oán

1 Đề Thi

1.1 Ngày thi thứ nhất

Bài toán 1. Gọi α nghiệm dương phương trình x2 +x = Giả sử n số nguyên dương số nguyên không âm

c0,c1,c2, ,cn thỏa mãn đẳng thức

c0 +c1α+c2α2+ .+cnαn=2015 (˚)

a) Chứng minh c0 +c1+c2+ .+cn”2 (mod 3)

b) Với n c0,c1,c2, ,cn thỏa mãn điều kiện

(˚)đã cho Tìm giá trị nhỏ tổngc0+c1+c2+ .+cn

Bài toán 2. Cho đường trịn (O) có dây cung BC cố định khơng phải đường kính Xét điểm A di chuyển cung lớn

BC cho tam giácABC nhọn ABăAC GọiI,H trung điểm cạnh BC trực tâm tam giác ABC Tia IH cắt lại đường tròn (O) K, đường thẳng AH cắt đường thẳng BC

D đường thẳng KD cắt lại đường tròn (O) M.Từ điểm M, kẻ đường thẳng vng góc với đường thẳng BC cắt AI điểm

N

a) Chứng minh điểm N ln thuộc đường trịn cố định A thay đổi

b) Đường tròn tiếp xúc với AKở Avà qua Ncắt AB,AClần lượt tạiP,Q.GọiJlà trung điểm củaPQ Chứng minh đường thẳngAJ qua điểm cố định

Bài toán 3. Một số nguyên dươngk có tính chất T(m) với số ngun dương a, tồn số nguyên dương nsao cho

1k+2k+3k+ .+nk ”a (modm)

(91)

Tạp

chí

onl

ine

của

cộng

đồng

những

người

yêu

T

oán

1.2 Ngày thi thứ hai

Bài tốn 4. Có 100 sinh viên tham dự thi vấn đáp Ban giám khảo gồm 25 thành viên Mỗi sinh viên hỏi thi giám khảo Biết sinh viên thích nhất10giám khảo số thành viên

a) Chứng minh chọn giám khảo mà thí sinh thích 1trong người

b) Chứng minh xếp lịch thi cho thí sinh giám khảo thích hỏi giám khảo hỏi khơng q 10thí sinh

Bài tốn 5. Cho tam giác ABC nhọn, không cân điểmP

nằm tam giác cho =APB = =APC = α với α ą 180˝ ´

=BAC Đường tròn ngoại tiếp tam giácAPB cắt đường thẳngAC

ở E, đường tròn ngoại tiếp tam giác APC cắt đường thẳng AB

ở F Gọi Q điểm nằm tam giác AEF cho =AQE =

=AQF=α Gọi D điểm đối xứng vớiQ qua EF, phân giác góc

EDFcắt AP T

a) Chứng minh =DET ==ABC,=DFT ==ACB

b) Đường thẳng PA cắt đường thẳng DE,DF

M,N Gọi I,J tâm đường tròn nội tiếp tam giác PEM,PFN K tâm đường tròn ngoại tiếp tam giác

DIJ Đường thẳng DT cắt (K) H Chứng minh HK

đi qua tâm đường tròn nội tiếp tam giácDMN

Bài tốn 6. Tìm số nguyên dương nnhỏ cho tồn n

số thực thỏa mãn đồng thời điều kiện sau:

i) Tổng chúng số dương

ii) Tổng lập phương chúng số âm

(92)

Tạp

chí

onl

ine

của

cộng

đồng

những

người

yêu

T

oán

2 Nhận xét chung

Cấu trúc đề thi Việt Nam TST năm gồm có:

• Bài 1: Đại số (đa thức có hệ số ngun nghiệm vơ tỷ)

• Bài 2: Hình học phẳng

• Bài 3: Số học

• Bài 4: Tổ hợp

• Bài 5: Hình học phẳng

• Bài 6: Bất đẳng thức

Như cấu trúc đề thi năm giống với cấu trúc đề thi IMO Theo đánh giá chủ quan chúng tơi, độ khó sau:

Bài 1bài 4 có độ khó trung bình (khoảng 50%-60% số học sinh làm trọn vẹn)

Bài 2, 5a bài 6 tương đối khó (khoảng 30%-50% làm được)

Bài 3 bài 5b khó (khoảng 10%-20% số học sinh đạt điểm dương)

Sau chúng tơi có số nhận xét cụ thể:

2.1 Về ngày thi thứ nhất

Chúng tơi thích thi ngày Bài không khó với học sinh đạt giải Nhất/Nhì VMO-2015

Bài (đại số) gồm hai câu a) b) Phương pháp giải dựa vào ý tưởng chia đa thức có hệ số nguyên cho tam thức bậc hai Ngồi câu a) làm quy nạp Câu b) khó ý tưởng xét hệ bất phương trình dễ bị nhầm lẫn

(93)

Tạp

chí

onl

ine

của

cộng

đồng

những

người

yêu

T

ốn

các kiến thức hồn tồn nhẹ nhàng Các em không thiết phải sử dụng định lý mà học sinh chuyên Toán làm quen Tuy nhiên, việc cho điểm N vào câu b) hồn tồn khơng cần thiết dễ làm thí sinh lạc hướng

Bài (số học) thực chất sử dụng số tính chất lý thuyết số thường dùng cho mật mã khái niệm: multiplicative order (hoặc modulo order) số nguyên định lý Trung Hoa số dư Đây bai toán "hàn lâm" bài, từ hình thức nội dung Bắt đầu việc xây dựng lý thuyết t(m) môt số kêt trường hợp cụ thể Có lẽ phải đọc t(m) Tờ-Mờ, số thí sinh chống với cách định nghĩa Một toán đưa vào tương đối táo bạo, vấn đề việc hiểu tiếp cận toán thời gian ngắn

Sau tiếp cận toán cảm giác tốn có liên quan đến định lý Trung Hoa phương trình Frobenius dấu hiệu báo trước khó khăn tốn

Bài toán liên quan đến vấn đề thời lý thuyết mật mã Theo hướng giống thông lệ chung IMO chọn thi từ nghiên cứu cịn nóng hổi

Như ngồi kiến thức số học muốn làm số 3, thí sinh cần phải làm nhanh số số Vì có lẽ vài em dạng "đặc biệt" biết làm Theo chúng tôi, số học sinh làm trọn vẹn số đếm bàn tay Hy vọng thực tốt cho vai trò phân loại

2.2 Về ngày thi thứ hai

Bài (tổ hợp) lại giống tổ hợp kì thi VMO 2015 có lẽ tác giả Tuy nhiên toán dễ hơn, em nghiên cứu kĩ lời giải kì thi VMO-2015 em khơng gặp khó khan Tơi cho mà nhờ phần lớn em với số điểm dương

(94)

Tạp

chí

onl

ine

của

cộng

đồng

những

người

yêu

T

ốn

nhiều lắp ghép không thực đẹp mắt Ban giám khảo đau đầu để chấm lời giải Ngồi ra, việc để hình túy lại giống (tam giác nội tiếp đường trịn) phí phạm cho quỹ đề Chúng tơi cho thay tổ hợp dạng khác hình học đa giác hình học tổ hợp (đang thưc trào lưu giới)

Bài (đại số) Đây có lẽ hay kỳ thi năm nhiều lý Nó vừa đẹp tự nhiên hiểu hy vọng tìm vũ khí để giải đủ bướng bỉnh để làm thất vọng thí sinh muốn đánh nhanh diệt gọn! Một tốn đủ khó lại khơng q khó kết cục người đạt đươc số điểm tối đa Nếu thí sinh kiên nhẫn khơng "sợ" xét nhiều trường hợp giải tốt tốn

Bên cạnh đó, tốn tìm nhiều cách tiếp cận làm giảng bồi dưỡng cho học sinh hệ sau Chúng hy vọng tìm lời giải đẹp ngắn gọn cho tốn

Nhìn chung, đề thi năm thỏa mãn yêu cầu đặt có nghĩa đảm bảo để chọn em xuất sắc đại diện cho học sinh nước tham dự kì thi IMO Thái lan vào tháng năm Tuy nhiên cảm thấy đề thi có chỗ để thêm toán lĩnh vực khác Toán học thuật tốn, tối ưu, lý thuyết trị chơi, nhằm góp phần cơng việc lưa chọn học sinh ưu tú đại diện tham dự kỳ thi IMO Theo kinh nghiệm chúng tôi, lựa chọn thi HSG quốc gia chọn đội tuyển TST khơng có ý nghĩa tức thời chọn học sinh giỏi mà định hướng để thầy em học sinh có chủ đề lên kế hoạch rèn luyện năm

(95)

Tạp

chí

onl

ine

của

cộng

đồng

những

người

yêu

T

oán

3 Gợi ý giải

Bài 1.

a) Chng minh rng a thc

cnxn+cn1xn1+ă ă ă+c02015,

chia hết cho đa thức x2+x–5.

b) Đầu tiên ta nhận xét c0, c1, , cn thỏa mãn điều kiện

c0+c1+c22+ă ă ă+cnn =2015,

vc0+c1+ă ă ă+cn nh nht có0ďci ď4.Từ đó, khai thác đẳng thức sau õy:

cnxn+cn1xn1+ă ă ă+c02015= (x2+x5)(an2xn2+ă ă ă+a1x+a0)

Bi 2.

a) Chỉ cần chứng minh B,C,H,N thuộc đường trịn Để thực điều này, ta gọi F giao điểm trung tuyến AM với đường tròn (O) N,F đối xứng qua trung trực BC

b. Gọi (S) đường tròn qua A, tiếp xúc với AK qua N Ta chứng minh DP, DQlà tiếp tuyến đường trịn (S), từ suy raAJ,AH đẳng giác tam giác ABC hayAJ quaO

cố định

Bài 3.

a) Có thể giải dễ dàng nhận xét số dư chia

Sk(n) =1k+2k+ă ă ă+nk theo 20l tun hon

b) Da vào kết câu a) ta dự đoán k=4là số nguyên dương nhỏ thỏa mãn yêu cầu toán Để chứng minhk=4thỏa mãn tính chất T(2015), ta sử dụng định lý phần dư Trung Hoa để quy modulo lũy thừa nguyên tố là3, 2nvà 5n

Sau dùng ý tưởng nâng lũy thừa để giải

(96)

Tạp

chí

onl

ine

của

cộng

đồng

những

người

yêu

T

oán

Ý tưởng chung hai câu dùng thuật toán, thuật toán ăn tham dựa vào nguyên lý Dirichlet Với ý b) ý số

25không quan trọng

Bài 5.

a)Chứng minhAP,AQliên hợp đẳng giác gócBACvàA,X

liên hợp đẳng giác tam giácEQFđể có X,Q,Athẳng hàng Từ suy T,Q liên hợp đẳng giác tam giácAEF

b) Chứng minh tứ giác DEPFngoại tiếp đường tròn tâm T GọiY

là tâm đường trịn ngoại tiếp tam giác DMNthì D,Y,I,Jnội tiếp nên YH đường kính đường trịn(DIJ) nên YHđi qua K

Bài 6.

Ta chứng minh n = cách chứng minh hai kết sau:

(a) Không tồn số thực thỏa mãn điều kiện đề (b) Chỉ số thực thỏa mãn điều kiện đề

Trong chứng minh ý (a), ta dùng phản chứng xét trường hợp khác dấu số, khó trường hợp

số dương,2 số âm Có thể dùng đại số dùng giải tích, dựa vào số nghiệm phương trình

ax+bx´cx´dx =0,

trong a, b, c, dlà số dương

Trong ý (b), ta tìm số thỏa mãn điều kiện dạng

a=2x, b=c=1, d=e=´(x+1)

Khi a+b+c+d+e=0.Ta chọnx cho

a3+b3+c3 +d3+e3 ă0,

a5+b5+c5 +d5+e5 ą0

(97)

T p c h í

online

cộng đồng

những n g i y ê u T o n

Tạp

chí

onl

ine

của

cộng

đồng

những

người

yêu

T

ốn LỜI GIẢI VÀ BÌNH LUẬN

HAI BÀI HÌNH THI

CHỌN ĐỘI TUYỂN VIỆT NAM 2015

Trần Quang Hùng (THPT Chuyên KHTN, ĐHQG Hà Nội)

Tóm tắt

Trong kỳ thi chọn đội tuyển Việt Nam năm 2015, có hai tốn hình học phẳng khó thú vị Dưới đây, chúng tơi trình bày lời giải chi tiết, phân tích vấn đề liên quan mở rộng chúng

1 Bài hình ngày thứ nhất

Bài tốn thứ có nội dung sau

Bài tốn 1. Cho đường trịn (O) có dây BC cố định khơng phải đường kính Xét điểm Adi chuyển cung lớn BCsao cho tam giác ABC nhọn AB ă AC Gọi I,H trung điểm cạnh BC trực tâm tam giác ABC Tia IH cắt lại đường tròn (O) K, đường thẳng AH cắt đường thẳng BC D đường thẳng KD cắt lại đường tròn (O) M Từ điểm M, kẻ đường thẳng vng góc với đường thẳng BCcắt AItại điểm N

a) Chứng minh điểm N ln thuộc đường trịn cố định A thay đổi

b) Đường tròn tiếp xúc với AKở Avà qua Ncắt AB,AClần lượt tạiP,Q.GọiJlà trung điểm củaPQ Chứng minh đường thẳngAJ qua điểm cố định

(98)

Tạp

chí

onl

ine

của

cộng

đồng

những

người

yêu

T

oán

cần đường trịn quaA tiếp xúcAKlà tốn đúng, mặt khác điểm cố định lại O xuất đề nên giảm nhiều thú vị tốn

Do đó, chúng tơi tách riêng hai ý toán giải phân tích Ta tốn sau:

Bài toán 2. Cho tam giác ABC nhọn nội tiếp đường tròn (O)

AD đường cao H trực tâm tam giác ABC M trung điểm BC Đường trịn đường kính AH cắt(O) G khác A GD

cắt (O) K khác A Đường thẳng qua K vng góc BC cắt

AMtại L Chứng minh bốn điểm B,C,L,H thuộc đường tròn

A

B C

O

H G

D M

E K

F

L S

Đầu tiên lời giải tác giả viết:

Lời giải 1. Do G nằm đường trịn đường kính AH nên gọi

GHcắt(O) tạiEkhácGthì AElà đường kính của(O) Từ đó, dễ thấy tứ giác HBEC hình bình hành nênHE quaM GọiKM

cắt(O)tại F khácK Chú ý tứ giác AGDM nội tiếp nên =AMB =180˝ ´=AGD ==AFM

Ta có, BCtiếp xúc đường trịn ngoại tiếp tam giác AFM Do đó,

OM K BC K SM với S tâm ngoại tiếp tam giác SFM S

(99)

Tạp

chí

onl

ine

của

cộng

đồng

những

người

yêu

T

ốn

Mặt khác, dễ dàng có SO KAF nên suy AFkBC Từ đó, ta có tam giác MAF cân nên

=LMB==MAF==MFA==BMK

Tam giác KLM cân nên K,L đối xứng qua BC Chú ý đối xứng H qua BC thuộc (O) nên H,L,B,C thuộc đường tròn đối xứng với (O) quaBC

Ta có điều phải chứng minh

Tiếp theo lời giải tác giả Nguyễn Chương Chí tham khảo ở [2]:

A

B C

O

M H

G

D

L

K E

P

Q

N

Lời giải 2. Do G nằm đường trịn đường kính AH nên gọi

GHcắt(O) tạiEkhácGthì AElà đường kính của(O) Từ đó, dễ thấy tứ giácHBEC hình bình hành nênHE quaM

Gọi AM cắt (O) N khác A Từ tứ giác AGDM AGKN nội tiếp suy

=GDM =180˝

´=AGD==GKN

suy raKNkBC Do đó, tứ giácBCNK hình thang cân mà

(100)

Tạp

chí

onl

ine

của

cộng

đồng

những

người

yêu

T

oán

Dễ thấy tam giác LKN vuông K nên suy M trung điểm

LNhayLđối xứngKquaBC Chú ý điểm đối xứng củaHquaBC

cũng thuộc (O) nên H,L,B,C thuộc đường trịn đối xứng với (O)qua BC

Ta có điều phải chứng minh

Nhận xét. Bài toán hiển nhiên bao hàm câu a) toán Chú ý có số tính chất hệ điểm L thuộc đường trịn đường kính AH AK đường đối trung tam giác ABC Thực chất tính chất mơ hình quen thuộc Sau xin đưa hai mở rộng khác cho toán này:

Bài tốn 3. Cho tam giácABC nội tiếp đường trịn(O) Điểm P

thuộc cungBCkhông chứa A.Qđối xứngP quaBC.QB,QCcắt

CA,AB E,F Đường tròn ngoại tiếp tam giác AEF cắt (O)

Gkhác A AP cắtBCtạiD.GDcắt(O)tại KkhácG.M trung điểm BC Đường thẳng quaK vng gócBC cắtAMtại L a) Chứng minh rằngB,C,L,Qcùng thuộc đường tròn b) Chứng minh rằngL nằm đường tròn ngoại tiếp tam giác

AEF

A

B C

O

P Q F

E

M G

D

K L

N

(101)

Tạp

chí

onl

ine

của

cộng

đồng

những

người

yêu

T

oán

Lời giải. GọiGQcắt(O)tạiRkhácG Ta thấy=CQR==GQF=

=GAB = =GRB Từ CQ k BR Tương tự BQ k CR Vậy tứ giác

BRCQ hình bình hành hay QRđi qua M Từ dễ thấy

PR k BC Ta có =EBD = =BCR = =CBP = =DAC Suy tứ giác ABDE nội tiếp Suy =BDA = =BEA = 180˝

´AGQ hay

=ADM = 180˝ ´=BDA = =AGQ Từ tứ giác AGDM nội tiếp.

Gọi KM cắt (O) N khác K Vì tứ giác AGDM nội tiếp nên =AMB =180˝

´=AGD ==ANM Từ BC tiếp xúc đường tròn ngoại tiếp tam giác ANM VậyOMKBCKSMvớiSlà tâm ngoại tiếp tam giác SNMdo Sthuộc OM Mặt khác dễ cóSOKAN

nên suy raANkBC Như vậy, ta có tam giácMAF cân suy =LMB= =MAF ==MFA ==BMK Từ tam giác KLM cân nên K,L đối xứng qua BC Vậy B,C,L,Q thuộc đường tròn đối xứng với (O) qua B,C

b) Gọi AM cắt (O) P khác A Từ tứ giác AGDM nội tiếp ta có =AMD = 180˝

´=AGD = =APK suy PK k DM =CAK = =BAM Từ =LQC = =LBC = =KBC = =BCR =

=CAK==BAM Vậy tứ giácAFQLnội tiếp hayLnằm đường tròn ngoại tiếp tam giácAEF

Nhận xét. Bài toán AP K BC Q trực tâm ta thu tốn Chú ý ta hồn tồn có số yếu tố cố định P thay đổi điểm KvàL Từ ta đề xuất tốn sau có ý nghĩa

Bài toán 4. Cho tam giácABC nội tiếp đường trịn(O) Điểm P

thuộc cungBCkhơng chứa A.Qđối xứngP quaBC.QB,QCcắt

CA,ABtại E,F

a) Chứng minh đường tròn ngoại tiếp tam giácAEFluôn qua điểm cố định khácA P thay đổi

b) Chứng minh tâm đường trịn Euler tam giác AEF

ln thuộc đường thẳng cố định P thay đổi Ta tiếp tục tới mở rộng khác cho toán 2:

Bài toán 5. Cho tam giác ABC nội tiếp đường tròn (O) Một đường tròn (K) qua B,C cắt CA,AB E,F khác B,C BE cắt

CFtại H Đường tròn ngoại tiếp tam giác AEFcắt(O) tạiGkhác

A AH cắt BC D GD cắt (O) N khác G Gọi M trung điểm BC Chứng minh đường thẳng qua N song song AD

(102)

Tạp

chí

onl

ine

của

cộng

đồng

những

người

yêu

T

oán

A

B C

K E

F

H O G

D

N

M L

P

R

Lời giải. Theo tính chất trục đẳng phương dễ thấy AG,EF,BC

đồng quy P Gọi MN cắt (O) R khác M ta có (BC,DP) = ´1 suy G(BC,DP) = ´1 chiếu lên đường tròn (O) suy

(BC,AN) =´1suy chùmR(BC,AN) =´1nhưng doRNđi qua

trung điểm BC suy RA k BC Từ =RAE = =ACB = =AFE

suy raAR tiếp xúc đường tròn ngoại tiếp tam giácAEF Vậy gọi AM cắt đường tròn ngoại tiếp tam giác AEF L suy =GNM = 180˝ ´=GAR ==GLA Suy tứ giác GLMN nội tiếp.

Mặt khác với ý(BC,DP) =´1ta có PG.PA=PB.PC=PD.PM

suy tứ giác GAMD nội tiếp Từ =GDA = =GMA ==GNL

suy NLkADvậy ta có điều phải chứng minh

Nhận xét. Nếu(K)là đường trịn đường kínhBCthì ta thu tốn Từ tính chất hàng điều hòa đường tròn ta chứng minh đượcANlà đường đối trung tam giácABC Với ý tưởng từ hai lời giải trên, tiếp tục với mở rộng khác sau

Bài toán 6. Cho tam giác ABC nội tiếp đường tròn (O) D

thuộc đoạn BC AD cắt (O) P khác A Q thuộc (O) cho

PQ k BC R thuộc (O) cho QR k AP Một đường tròn qua

(103)

Tạp

chí

onl

ine

của

cộng

đồng

những

người

yêu

T

oán

và đường thẳng qua K song song AD cắt đường tròn ngoại tiếp AGH

Tuy toán tổng quát thực tốn có lời giải đơn giản khơng phải dựng thêm hình phụ, ý tưởng tổng qt xuất phát từ lời giải toán 2:

A

B C

P Q

D

R

M

K G

H

L

Lời giải. TừPQkBCvà góc nội tiếp

=AGM ==APQ ==ADM

suy tứ giác AGDM nội tiếp Từ đó, ý đường trịn ngoại tiếp tam giác ARM tiếp xúc BCnên

=ARM==AMD=180˝

´=AGD==ARK

Do đó, RMđi qua K Từ APkQR, ta có =RAH==APQ ==AGH

(104)

Tạp

chí

onl

ine

của

cộng

đồng

những

người

yêu

T

oán

Giả sử đường tròn ngoại tiếp tam giác AGH cắtAMtại Lthì

=GKR=180˝

´=GAR ==GLA

nên suy tứ giác GLMK nội tiếp Từ =GDA = =GMA =

=GKL hayKLkAD

Ta có điều phải chứng minh

Ta quay lại ý b) tốn Ta tách thành tốn sau:

Bài toán 7. Cho tam giác ABC nội tiếp đường trịn (O) trực tâm H Đường trịn đường kính AH cắt (O) G khác A Một đường tròn tiếp xúc AG A cắt CA,AB E,F khác

A Chứng minh AO chia đôi EF

Lời giải. Gọi GH cắt (O) D khác A AD đường kính

của (O) Từ tứ giác HBDC hình bình hành nên HD qua trung điểm M củaBC

Gọi ADcắt đường tròn ngoại tiếp tam giác AEF P khácA

AP cắt EFtại N Ta dễ thấy

=FPN==FAE==GAB==BDM

=PFE==PAE ==DBM

Từ tam giác DBM PFM đồng dạng Tương tự tam giác

DCM PENđồng dạng

(105)

Tạp

chí

onl

ine

của

cộng

đồng

những

người

yêu

T

oán

A

B C

O H

G

F

E N

D M

P

Nhận xét. Việc bốn điểm thẳng hàng G,H,M,D đóng vai trị quan trọng lời giải Một cách dùng kỹ thuật góc đồng dạng hồn tồn tương tự ta có tốn sau bạn làm luyện tập:

Bài toán 8. Cho tam giác ABC nội tiếp đường trịn (O) Điểm

P thuộc BC khơng chứa A Q đối xứng P qua BC QB,QC cắt

CA,AB E,F Đường tròn ngoại tiếp tam giác AEF cắt (O)

GkhácA Một đường tròn tiếp xúc AGtạiAcắtCA,ABlần lượt M,N R thuộc (O) cho PR k BC Chứng minh AR

chia đôi MN

Bài toán 9. Cho tam giác ABC với Gthuộc(O) Một đường tròn tiếp xúc AG A cắt CA,AB E,F khác A D thuộc

BC.GD cắt(O)tại L khác G AL cắtEF K Chứng minh trung điểm BF,CE vàDK thẳng hàng

Bài tốn 10. (Đề châu Á Thái Bình Dương 2012) Cho tam giác

ABCnhọn GọiDlà chân đường cao kẻ từAđếnBC, Mlà trung điểm BC H trực tâm tam giác ABC Giả sử E giao điểm đường trònΓ ngoại tiếp tam giácABC với tiaMH.GọiF

là giao điểm đường thẳngEDvà đường tròn Γ Chứng minh BF

(106)

Tạp

chí

onl

ine

của

cộng

đồng

những

người

yêu

T

oán

2 Bài hình ngày thứ hai

Cũng đề chọn đội tuyển Việt Nam ngày thứ 2, có hình học sau:

Bài tốn 11. Cho tam giác ABC nhọn khơng cân có điểm P

nằm tam giác cho =APB ==APC =α ą180˝

´=BAC

Đường tròn ngoại tiếp tam giác APB cắt AC Ekhác A Đường tròn ngoại tiếp tam giác APC cắt AB F khác A Gọi Q điểm nằm tam giác AEF cho =AQE = =AQF = α Gọi D điểm đối xứng vớiQ quaEF, phân giác góc =EDFcắtAP tạiT

a) Chứng minh =DET ==ABC,=DFT ==ACB

b) Đường thẳng PA cắt đường thẳng DE,DF

M,N Gọi I,J tâm đường tròn nội tiếp tam giác

PEM,PFNvàKlà tâm đường tròn ngoại tiếp tam giácDIJ Đường thẳng DT cắt (K) H Chứng minh đường thẳng HK qua tâm đường tròn nội tiếp tam giác DMN

Nhận xét.Đây toán hay Câu a) dùng gợi ý hướng giải cho câu b) Nếu để ý kỹ thực chất toán ghép nối hai tốn khác Sau chúng tơi xin giới thiệu lại với bạn hai tốn với nguồn gốc

Trước hết ta nhắc lại hai toán quan trọng tứ giác ngoại tiếp

Bài toán 12. Cho tứ giác ABCD

a) Tứ giác ABCD ngoại tiếp AB+CD = AD+BC Tứ giác ABCD gọi ngoại tiếp tức tồn đường tròn (I) tiếp xúc cạnh AB,BC,CD,DA

b) Tứ giácABCDbàng tiếp gócA,Ckhi khiAB+AD=CB+

CD Tứ giácABCD gọi bàng tiếp tức tồn đường trịn

(I)chứa gócAhoặcCtiếp xúc với cạnhAB,BC,CD,DA

kéo dài

(107)

Tạp

chí

onl

ine

của

cộng

đồng

những

người

yêu

T

oán

I

D B

A

C

I

D B

A

C

Chúng ta toán tham khảo [3] đề chọn đội tuyển Indonesia năm 2007:

Bài toán 13. Cho tứ giác ABCD nội tiếp đường tròn (O) Hai đường chéo AC BD cắt S Đường tròn ngoại tiếp tam giác SAD,SBC cắt tại T khác S Dựng tứ giác tam giác ABR đồng dạng với DCT Chứng minh tứ giác

AT BR tứ giác ngoại tiếp

Đây tốn hay có nhiều cách tiếp cận khác Tuy chọn lời giải sau gần ngắn gọn sử dụng hướng toán 11

A

D C

O B

S

T R

(108)

Tạp

chí

onl

ine

của

cộng

đồng

những

người

yêu

T

oán

Lời giải. GọiK đối xứng R quaBC Ta thấy

=KAS==BAS´=BAK==BDC´=RAB

==BDC´=T DC==SDT ==SAT

Từ đó, AS phân giác =KAT Tương tự, BS phân giác =KBT Dễ thấy T Slà phân giác =AT B

Từ đó, đường trịn (S) tiếp xúc với KA,KB,T A,T B suy AK´

AT =BK´BT Theo tính đối xứng, suy raAR´AT =BR´BT hay

AR+BT =BR+AT suy tứ giácARBT ngoại tiếp Ta có điều phải chứng minh

Nhận xét. Trong [3] có lời giải khác ngồi tốn giải phép nghịch đảo tính chất phương tích Ta dễ chứng minh tâm nội tiếp tứ giác AT BR

là đẳng giác K tam giácSABđó nội dung tốn 11 phần a) Để tiếp tục xin nhắc lại chứng minh toán G8 IMO Shortlist 2009 [4] sau:

Bài toán 14. Cho tứ giác ABCD ngoại tiếp đường tròn (I) Một đường thẳng qua A cắt đoạn thẳng BC cắt tia đối tia CD

tại N Gọi J,K,L tâm nội tiếp tam giác CNM,MAB NAD Chứng minh trực tâm tam giác JKL nằm trênMN

Đây kết đẹp, bao hàm nhiều ý tưởng Chúng xin giới thiệu lời giải sử dụng toán 12 sau:

I A

C D

B

M

N K

L

(109)

Tạp

chí

onl

ine

của

cộng

đồng

những

người

yêu

T

oán

Lời giải. Gọi tiếp tuyến đường tròn (L) nội tiếp tam giác

NADcắt AM,ABtại P,Q Như tứ giácAPCD nội tiếp Kết hợp ABCD nội tiếp suy

PA´PC=DA´DC=BA´BC

Từ tứ giác APCB bàng tiếp hay tứ giác BQPM ngoại tiếp

CP tiếp xúc (K) Từ

2=LCK==BCD ==CMN+=CNM=2=JMN+2=JNM=2=MJL

Từ đó, tứ giác CJKL nội tiếp Theo định lý đường thẳng Steiner đường thẳng nối đối xứng C qua JK,JL qua trực tâm tam giác JKL

Chú ý theo tính chất phân giác đường thẳng

MN Vậy MNđi qua trực tâm tam giác JKL Ta có điều phải chứng minh

Nhận xét. Bài tốn có kết cấu chặt chẽ ý tưởng hay Chú ý điều kiện N thuộc tia đối tia CD cần thiết để toán Để ý CI K CJ nên CI cắt đường tròn (X) ngoại tiếp tứ giác CJKLtại Y JY đường kính của(X)

Phần nhận xét kết câu b) toán 11 Nếu để ý kỹ thi mơ hình tốn IMO shortlist xuất lần đề thi VMO năm 2011 xem [5]

I A

C D

B

M

N K

L

J P Q

(110)

Tạp

chí

onl

ine

của

cộng

đồng

những

người

yêu

T

oán

Trở lại toán 11, ta ý điều kiện chặt chẽ toán Shortlist nên ta cần bổ sung thêm điều kiện cho toán 11 M,N phải ln phía với P đường thẳng AP Từ ý tứ giác BCEF nội tiếp Bài toán kết hợp cách học toán 13 toán 14

Chú ý ta hồn tồn thay thế AP thành đường thẳng qua P Và đó, ta phát biểu lại toán này đẹp sau:

Bài toán 15. Cho tam giác ABC có điểm P nằm tam giác cho =APB ==APC =α.Đường tròn ngoại tiếp tam giác

PAB,PAC cắt CA,AB E,E khác A Điểm Q điểm nằm tam giác AEF cho =AQE = =AQF = α Gọi D điểm đối xứng vớiQ quaEF

Một đường thẳng qua P cắt đường thẳng DE,DF M,N cho M,N phía với P Gọi I,J tâm đường tròn nội tiếp tam giác PEM,PFN K tâm đường tròn ngoại tiếp tam giácDIJ Phân giác =EDF cắt(K) tạiH Chứng minh đường thẳng HK qua tâm đường tròn nội tiếp tam giác DMN

Nhận xét. Do toán cách kết hợp học hai tốn nên giải phân tích rõ hai tốn tốn kết hợp khơng cịn mang nhiều ý nghĩa Tuy việc sử dụng hai toán lớn tốn làm độ khó tốn thi tăng nhiều lần đánh giá cao phân loại tốt học sinh

Tác giả Nguyễn Văn Linh phát toán G8 IMO Shortlist 2009 có mở rộng có diễn đàn AoPS trước xem [6,7] Cũng thú vị tác giả Trần Quang Hùng đề nghị độc lập vấn đề tương tự thi kỷ niệm 45 tạp chí Tốn học Tuổi trẻ, xem [8] lời giải tác giả đăng [9]

Bài tốn có phát biểu sau:

Bài toán 16. Cho tứ giácABCD ngoại tiếp điểmP nằm tứ giác cho đường thẳngAP,DP tương ứng cắt đoạnBC

(111)

Tạp

chí

onl

ine

của

cộng

đồng

những

người

yêu

T

oán

B

D C

A

P R S

I1

I2

X

Y I4

I3 M N Z

U

V K

L

T

Lời giải. Gọi(I1),(I2),(I3),(I4)theo thứ tự đường tròn nội tiếp

của tam giác ABS,DCR, PAD,PSR Gọi X,Y tiếp điểm tiếp tuyến chung khác BC (I1),(I2) với (I),(I) Gọi M,N

giao XY với PA,PD Gọi Z,K,U tiếp điểm (I1) với

AS,AB,SB

Gọi T,L,V tiếp điểm (I2) với DR,DC,RC Dễ thấy

(1)BU=BK,CV =CL,

(2)XY =UV,AZ=AK,DT =DL, (3)MX=MZ,NY =NT

Ta có biến đổi tương đương sau cạnh: Tứ giác ABCD

ngoại tiếp

CB+AD=AB+CD

ôBU+UV +VC+AD=AK+KB+DL+LC

ôUV +AD=AK+DL, theo(1) ôXY+AD=AZ+DT, theo (2)

ơXM+MN+NY+AD=AM+MZ+DN+NT ơMN+AD=AM+DN, theo(3)

Từ suy AMND ngoại tiếp đường trịn nội tiếp AMDN

chính (I3) Do đó,

=MI3N=180˝´=AI3D=180˝´(90˝´

1

2=APD) =90

˝

´1

(112)

Tạp

chí

onl

ine

của

cộng

đồng

những

người

yêu

T

oán

nên

=I1I3I2 =180˝´(90˝ ´

1

2=SPR) =180

˝

´=I1I4I2

Vậy I1I2I3I4 nội tiếp, ta có điều phải chứng minh

Chúng ta tiếp tục với mở rộng thú vị sau cho toán 13.

Bài toán 17. Cho tứ giác ABCD nội tiếp đường tròn (O) Các điểm E,F thuộc cạnh CB,AD cho EF k AB DE cắt CF

S Đường tròn ngoại tiếp tam giác ASD BSC cắt T

khác S Dựng điểm R tứ giác ABCD cho =RAB =

=ASF+=T DC =RBA = =BSE+=T CD Chứng minh tứ giác AT BR ngoại tiếp

O A

B

C D

F

E

S

T Q R

Lời giải. Ta dễ thấy tứ giácCDFE nội tiếp nên =ST A==SDF==SCE==ST B

Từ đó, T S phân giác =AT B Ta lại có

=QAS==SAB´=QAB==DAB´=DAS´=RAB

==DFE´(=DFS´=FSA)´(=ASF+=T DC)

(113)

Tạp

chí

onl

ine

của

cộng

đồng

những

người

u

T

ốn

Do đó, AS phân giác =QAT Tương tự, ta có BS phân giác =QBT Từ đó, suy đường trịn (S) tiếp xúc với

QA,QB,T A,T Bhay AQ´AT =BQ´BT

Theo tính đối xứng suy AR´AT = BR´BT hay AR+BT =

BR+AT, ta suy tứ giác ARBT ngoại tiếp Ta có điều phải chứng minh

Nhận xét. Việc phát biểu tốn dùng góc hình học nhiều trường hợp chưa thật xác xong phần giúp cho tốn nhìn đẹp hơn, việc phát biểu cách chặt chẽ khái niệm góc có hướng bạn tự tìm hiểu thêm Khi

EF trùngAB ta thu toán 13 Nếu việc kết hợp toán 13 toán 14 cho ta tốn TST việc kết hợp toán 17 toán 14 cho ta mở rộng toán dùng kỳ thi TST, rõ ràng việc kết hợp cách học bạn thấy khơng mang nhiều ý nghĩa

Tuy vậy, để thú vị xin đưa ý tưởng sau từ đề toán 11 Những điểm P nằm tam giác ABC

sao cho =APB = =APC điểm P có đặc biệt hay nói cách khác, quỹ tích P Bài tốn sau giải đáp thắc mắc đó, khơng cố gắng tìm quỹ tíchP mà tìm quỹ tích điểm đẳng giác P Đó quỹ tích đẹp chứng minh đơn giản, bạn làm luyện tập:

Bài tốn 18. Cho tam giác ABC có đường trịn Apollonius ứng với A (K) P điểm thuộc (K) Q đẳng giác với P tam giác ABC Chứng minh =AQB ==AQC

Tài liệu tham khảo

[1] Đề thi chọn đội tuyển Việt Nam thi IMO năm 2015 [2] Facebook Bài toán hay - Lời giải đẹp

https://www.facebook.com/BÀI TOÁN HAY - LỜI GIẢI ĐẸP - ĐAM MÊ TOÁN HỌC

[3] Indonesia IMO 2007 TST, Stage 2, Test 5, Problem

(114)

Tạp

chí

onl

ine

của

cộng

đồng

những

người

yêu

T

oán

[4] IMO Shortlist 2009 - Problem G8

http://www.artofproblemsolving.com/community/ c6h355795p1932940

[5] A, M, N, P are concyclic iff d passes through I- [VMO 2011]

http://www.artofproblemsolving.com/community/ c6h386167p2144390

[6] Vietnam TST 2015 Problem

https://nguyenvanlinh.wordpress.com/2015/03/28/ vietnam-tst-2015-problem-5/

[7] Tangential quadrangle

http://www.artofproblemsolving.com/community/c6h247411

(115)

T p c h í

online

cộng đồng

những n g i y ê u T o n

Tạp

chí

onl

ine

của

cộng

đồng

những

người

yêu

T

oán CÁC VẤN ĐỀ

CỔ ĐIỂN VÀ HIỆN ĐẠI

Trần Nam Dũng

Đại học KHTN, TP Hồ Chí Minh

Chuyên mục dành cho vấn đề cổ điển đại trình bày dạng tốn xâu chuỗi Đó chuỗi để giải tốn đẳng chu, chứng minh đẳng thức Euler kỳ hiệu1+

22+

1

32+ă ă ă=

2

6 ,một chuỗi toán vận trù Cách

trình bày xuất phát từ vấn đề đơn giản, dễ hiểu, khái niệm định nghĩa ln để đọc tương đối độc lập Và chuỗi nêu vấn đề định, giải toán kinh điển hay nêu giả thuyết mới, vấn đề

Ban biên tập khuyến khích độc giả gửi lời giải (tồn phần phần) cho Ban biên tập Các lời giải hay chọn đăng số (N+3) Thư điện tử xin gửi theo địa trannamdung@yahoo.com, tiêu đề [Epsilon] [Lời giải]

Trong số chúng tơi chọn đăng chuỗi tốn Bất đẳng thức Schapiro A.Khabrov đề nghị cho Hội nghị mùa hè Cuộc thi toán thành phố, năm2004và tốn mạng lưới xa lộ trích từ đề thi Olympic toán Pháp năm 2014 Đề bài.Bốn thành phố Alenc¸on, Bélanc¸on, Célanc¸on Délanc¸on nằm đỉnh hình vng có cạnh100km Bộ giao thơng vận tải muốn nối liền thành phố với mạng lưới xa lộ có tổng chiều dài ngắn có thể.

1 Phần A

(116)

Tạp

chí

onl

ine

của

cộng

đồng

những

người

yêu

T

ốn

Hay là, ta xây hai xa lộ chéo nhau: Một từ Alenc¸on đến Célanc¸on xa lộ cịn lại từ Délanc¸on đến Bélanc¸on” - Trợ lý thứ hai đề xuất

Và lại khơng dựng xa lộ dạng nửa đường trịn, được bổ sung hai đoạn xa lộ thẳng ?” - Trợ lý thứ ba góp ý kiến

1 Phương án ba phương án ngắn ? Một nhà toán học đề xuất phương án khác: “Ta

có thể nối Alenc¸on Délanc¸on tam giác cân (tam giácAEDtrong hình 4), sau Bélanc¸on Célanc¸on bằng một tam giác cân đồng dạng (tam giácBFC)và nối hai đỉnh

(117)

Tạp

chí

onl

ine

của

cộng

đồng

những

người

yêu

T

oán

2 Phần B

Trong phần này, muốn chứng minh mạng lưới xa lộ ngắn thực mơ hình nhà tốn học đề xuất Do ta tìm chiều dài đoạn EF để đạt mạng lưới ngắn

Nhắc nhở hình học: Nếu A, B, Clà ba đỉnh tam giác thì

AB+BCěAC,dấu xảy khi Bthuộc đoạn AC

Chúng ta biết rằng, vẽ đường cong giữa A

B, chiều dài đường cong lớn hay chiều dài đoạn (đường thẳng đường ngắn nhất).

Bài toán 1. Quay trở lại với mạng lưới xa lộ Khơng tính tổng quát ta giả sử mạng lưới tạo thành bở hai đường nối đỉnh đối diện (một nối A với C nối B với D), hai đường cong nằm bên hình vng cạnh 100 km hình vẽ

Xét mạng lưới tạo hai đường cong hình Ta xét xa lộ Alenc¸on Célanc¸on, Alenc¸on Gọi E0

là giao điểm mà xa lộ gặp với xa lộ nối Délanc¸on Bélanc¸on F0 giao điểm cuối (hai điểm

(118)

Tạp

chí

onl

ine

của

cộng

đồng

những

người

yêu

T

oán

Bài toán 2. Xét đường thẳng ∆E ∆F, qua E0 F0 tương

ứng song song vớiAD (xem hình đây)

(a) Xác định điểm E ∆E cho tổng khoảng cách

(119)

Tạp

chí

onl

ine

của

cộng

đồng

những

người

yêu

T

oán

(b) Chứng minh EFďE0F0

(c) Từ lý luận suy mạng lưới cần tìm bắt buộc phải có dạng sau, E F nằm trung trực đoạn AD(hình 8)

Bài tốn 3. Ta công nhận mạng lưới tối ưu cần tìm, điểm Evà F phải nằm trung trực củaAB

(a) Hãy đưa lý luận chứng tỏ mạng lưới tối ưu cần tìm phải đối xứng qua trung trực AB

(b) Sau tất điều nói trên, mạng lưới cần tìm phải có dạng nhà tốn học đề xuất (hình 4) Bạn giúp xác định chiều dài EF cho mạng lưới xa lộ dạng có tổng chiều dài ngắn ?

(120)

Tạp

chí

onl

ine

của

cộng

đồng

những

người

yêu

T

(121)

T p c h í

online

cộng đồng

những n g i y ê u T o n

Tạp

chí

onl

ine

của

cộng

đồng

những

người

yêu

T

oán BẤT ĐẲNG THỨC SHAPIRO1

1 Bất đẳng thức Shapiro

Tháng 10/1954, Tạp chí “American Mathematical Monthly” xuất toán nhà toán học Mỹ Harold Shapiro:Với các số dương x1, x2, , xn,hãy chứng minh bất đẳng thức

x1

x2+x3

+ x2 x3+x4

+ă ă ă+ xn1 xn+x1

+ xn x1+x2

ě n

2, (1)

với dấu xảy tất mẫu số nhau. Trong tạp chí “Monthly”, khác với tạp chí khác, ví dụ Kvant hay Tốn học tuổi trẻ, cho phép đăng tốn mà chưa có giải được, điều không báo trước cho độc giả Và lần Tác giả toán gửi có lời giải cho n=3 và4

Trong tốn đề nghị đây, thay địi hỏi tính dương tất xk, ta yêu cầu số xk không âm tất mẫu số khác 0.Nếu bất đẳng thức cho số dương từ suy bất đẳng thức cho số không âm làm cho mẫu số khác 0.Đặt:

f(x1,x2, , xn) =

x1

x2+x3

+ x2

x3+x4

+ă ă ă+ xn1

xn+x1

+ xn

x1+x2

1. Chứng minh bất đẳng thức (1) cho n=3, 4, 5,

2. Chứng minh bất đẳng thức (1) không đúng:

a) chon=20; b) chon=14; c) chon=25

1Trích đề tốn từ Cuộc thi toán thành phố, năm 2004 Được đề

(122)

Tạp

chí

onl

ine

của

cộng

đồng

những

người

yêu

T

oán

3. Chứng minh bất đẳng thức (1) cho dãy đơn điệu

4. Chứng minh bất đẳng thức (1) khơng với

n=m không cho n=m+2

5. Chứng minh bất đẳng thức (1) không với

n = m, m số tự nhiên lẻ, khơng với nlớn m

6. Chứng minh bất đẳng thức (1) với n =8, 10, 12

vàn=7, 9, 11, 13, 15, 17, 19, 21, 23.Theo mệnh đề4, để thực điều này, ta cần chứng minh cho hai trường hợp

n=12 vàn=23

7. Chứng minh f(x1, x2, , xn) +f(xn, xn´1, , x1)ěn

8. Giả sử điểma1, , aną0,hàm sốf(x1, x2, , xn) đạt cực tiểu địa phương

a) Nếun chẵn, chứng minh f(a1, a2, , an) = n2

b) Chứng minh mệnh đề tương tự chon lẻ (˚) c) Sử dụng a), b), chứng minh bất đẳng thức (1) chon=7

vàn=8

9. Chứng minh bất đẳng thứcf(x1, x2, xn)ěcncho giá trị sau sốc:

a) c= 14; b) c=?2´1; c) c= 125

Khi thi diễn ra, thí sinh giải tốt toán đề nghị mục 1, Ban tổ chức định bổ sung hai toán sau, thực chất chứng minh kết mạnh V Drinfeld bất đẳng thức Shapiro (kết mạnh cho đánh giá dạng9)

10. a) Với số nguyên dương n, tồn số qn ą1,sao cho với số thực x1, x2, , xn thuộc đoạn

1

qn, qn

bất đẳng thức (1)

b) ‹Tồn hay không sốq

ą1,sao cho với số nguyên dươngnvà với xiP

1

q, q

(123)

Tạp

chí

onl

ine

của

cộng

đồng

những

người

yêu

T

oán

11. GọiS=f(x1, x2, , xn)là vế trái bất đẳng thức Shapiro Ký hiệua1, a2, , an số xx21, xx32, , xnxn

´1,

x1

xn xếp theo thứ tự tăng dần

a) Chứng minh

a1(1+an)

+

a2(1+an1)

+ă ă ă+

an(1+a1)

;

b) Đặt:

bk =

$ ’ ’ &

’ ’ %

1

akan+1´k

, akan+1´k ě1

2

akan+1´k+

?

akan+1´k

, akan+1´k ă1

Chứng minh rng2Sb1+b2 +ă ă ă+bn

c) Gigl hm li ln không vượt hàme´x và2 ex+ex2´1.Chứng minh

2Sg(lna1an) +g(lna2an1) +ă ă ă+g(lnana1)năg(0)

d) Chng minh rng vi mi 12ăg(0),tn ti s nguyờn dngnv cỏc s dươngx1, x2, , xn,sao choSďλn

2 Các bất đẳng thức hữu dụng liên quan

Chứng minh bất đẳng thức với điều kiện tất số xk dương Hãy kiểm tra số in đậm thay số lớn (với n)

1 Bất đẳng thức Mordell.

a) Với số không âmx1, x2, , xn,ta có bất đẳng thức

n ÿ

k=1

xk

2

ěmin

!n

2,

)

ă

n

k=1

xk(xk+1+xk+2)

b) Hãy xác định với giá trị không âm

(124)

Tạp chí onl ine của cộng đồng những người yêu T oán

2. Với số thực không âm x1, x2, , xn, chứng minh bất đẳng thức sau:

n ÿ

k=1

xk min " n 3, * ă n

k=1

xk(xk+1+xk+2+xk+3)

3. a) Vớinď8,hãy chứng minh bất đẳng thức

x1

x2+x3+x4

+ x2

x3+x4+x5

+ă ă ă+ xn

x1+x2+x3

ě n b) ‹Bất đẳng thức với giá trị khác của

nhay không?

4. (x1+x2 +ă ă ă+xn)2 4(x1x2 +x2x3+ă ă ă+xnx1), @n4

5.

n

ÿ

k=1

xk

xk+1+xk+2

ě

n

ÿ

k=1

xk+1

xk+xk+1

6. x1

xn+x2

+ x2

x1+x3

+ă ă ă+ xn´1

xn´2+xn

+ xn

xn´1+x1

ě2, @ně4

7. x1+x2

x1+x3

+x2+x3

x2+x4

+ă ă ă+xn1+xn

xn1+x1

+xn+x1

xn+x2

ě4, @ně4 8. x1

xn+x3

+ x2

x1+x4

+ă ă ă+ xn1

xn2+x1

+ xn

xn´1+x2

ě3,ně6

9. x2+x3

x1+x4

+x3+x4

x2+x5

+ă ă ă+ xn+x1

xn´1+x2

+ x1+x2

xn+x3

ě6, @ně6 10. x1+x2

x1+x4

+x2+x3

x2+x5

+ă ¨ ¨+x2004+x1

x2004+x3

ě6

11. x1

xn+x4

+ x2

x1+x5

+ă ă ă+ xn

xn´1+x3

ě4, với ną5,n chẵn

12.

n

ÿ

k=1

x2k x2

k+1´xk+1xk+2+x2k+2

ě

n+1

(125)

T p c h í

online

cộng đồng

những n g i y ê u T o n

Tạp

chí

onl

ine

của

cộng

đồng

những

người

yêu

T

oán PHƯƠNG PHÁP

MỞ RỘNG VÀ SÁNG TẠO

CÁC ĐỊNH LÝ HÌNH HỌC CỔ ĐIỂN

Đào Thanh Oai (Hà Nội)

Tóm tắt

Đam mê môi trường nghiên cứu hai yếu tố quan trọng giúp cho thành công nhà Toán học Đam mê cộng với phương pháp nghiên cứu tạo kết nghiên cứu Môi trường nghiên cứu giúp cho kết nghiên cứu đánh giá cách xác giới chun mơn cộng đồng Ngồi ra, mơi trường nghiên cứu nơi để nhà nghiên cứu học hỏi trao đổi chia sẻ vấn đề họ quan tâm Khi thành nghiên cứu cộng đồng cơng nhận nguồn động lực để nhà nghiên cứu trì niềm đam mê Làm tạo vấn đề Tốn học hay tốn cách nhanh chóng, kiểm chứng kết cách dễ dàng hai vấn đề mà nhà nghiên cứu nên quan tâm

Xuất phát từ kinh nghiệm nghiên cứu Toán học việc tạo vấn đề hình học, tác giả viết báo nhằm chia sẻ kinh nghiệm đến bạn học sinh sinh viên tất có niềm đam mê Tốn học, đặc biệt mơn hình học phẳng

Trình bàyphương pháp mở rộng sáng tạo vấn đề hình học cổ điểnvà số vấn đề liên quan từ việc kiểm tra ý tưởng hay sai, hay cũ, mở rộng ý tưởng và cơng bố đến cộng đồng minh họa thơng qua nhiều ví dụ sinh động.

(126)

Tạp

chí

onl

ine

của

cộng

đồng

những

người

yêu

T

oán

1 Vai trò đối tượng bản

Đối với nghiên cứu Toán học, điều trước tiên ta cần tìm hiểu tính chất vốn có (hay cịn gọi tính chất tự nhiên đối tượng), mối kết hợp đối tượng, lớp đối tượng thuộc lĩnh vực ta muốn nghiên cứu mà đông đảo cộng đồng thừa nhận Tiếp theo nghiên cứu cấu trúc tính chất (vấn đề)

Nếu ta khơng biết biết đến ta khó biết Do việc hiểu đối tượng, tính chất, khái niệm, cấu trúc có vai trị Nếu khơng nắm bắt hiểu đối tượng, tính chất, khái niệm, cấu trúc đối tượng, cấu trúc kết hợp đối tượng khơng tìm cách phát biểu cho việc đưa giả thiết, hình thành, ý tưởng Vì thế, muốn nghiên cứu lĩnh vực phải có khái niệm lĩnh vực Nói cách khác, lĩnh vực ta quan tâm có đối tượng nào? Các đối tượng hiểu nào? Các tính chất đối tượng gì?

Đối với lĩnh vực hình học cổ điển phải biết đối tượng hình học điểm, đường thẳng, tam giác, tứ giác, đường tròn, Nghiên cứu điểm phải biết tính chất, khái niệm liên quan đến điểm ví dụ:

• Xác định đường thẳng qua hai điểm phân biệt

• Ba điểm gọi thẳng hàng điểm thứ ba nằm đường thẳng nối hai điểm thứ thứ hai Khi nghiên cứu đường thẳng ta lại cần biết khái niệm song song, vuông góc, góc hai đường thẳng, đường thẳng giao nhau, ba đường thẳng đồng quy,

• Nghiên cứu góc ta phải biết góc nhọn, góc tù, góc bẹt, liên hợp đẳng giác,

(127)

Tạp

chí

onl

ine

của

cộng

đồng

những

người

u

T

ốn

• Mở rộng chút sang đến đường cong, mà đường cong dễ xây dựng đường tròn, từ đường tròn ta mở rộng để tìm hiểu đường conic

Tóm lại:Các định lý hình học xây dựng cách nêu nên một giả thiết cấu trúc số đối tượng đối tượng hình học, sau suy mối quan hệ tính chất từ cấu trúc đó. Vì vậy, vai trị đối tượng tính chất đối tượng quan trọng.

2 Nắm bắt xu hướng nghiên cứu

Sau nắm vững khái niệm đối tượng , tính chất bản, Mới bắt đầu biết cần nghiên cứu gì? Cái đáng quan tâm? Cách nhanh để đến việc phát kết đáng ý phải nắm bắt xu hướng nghiên cứu Hay nói cách khác hòa vào dòng nghiên cứu chủ đạo đối tượng lĩnh vực ta muốn nghiên cứu Việc thật không đơn giản với người bắt đầu Vì bắt đầu cịn mờ mịt đến khái niệm tính chất chưa nắm bắt hết hiểu hết dòng xu hướng nghiên cứu?

Câu trả lời muốn biết xu hướng quan tâm lĩnh vực, cần phải ý đến kết luận bật nhất, bật chút, việc xem báo, viết thành tựu Toán học bật mà quan tâm Từ đó, ta hiểu vấn đề quan tâm có ý nghĩa, cịn thiếu cần bổ sung sau có kết ta dễ để truy tìm vấn đề cũ hay mới, ý nghĩa hay khơng

Hình học lĩnh vực thu hút ý thời gian qua, bắt đầu quan tâm đến hình học, tơi khơng biết hay, tơi hay theo dõi trang web Cut the Knot (http: //www.cut-the-knot.org/- trang phổ biến Toán học Tiến sĩ Mỹ) Người quản lý trang đồng thời tạo trang thông tin facebook nhằm chia sẻ, trao đổi với người vấn đề hình học

(128)

Tạp

chí

onl

ine

của

cộng

đồng

những

người

yêu

T

oán

giác quan tâm đến Tuy nhiên có lần trao đổi với người bạn nước ngồi, họ nói tam giác đẹp khơng đẹp tính chất ba điểm thẳng hàng đồng quy Tôi nhận hình học Euclid khơng có riêng tam giác cần nghiên cứu Từ hướng suy nghĩ đến kết luận để đến cấu trúc đồng quy thằng hàng Tôi muốn tạo cấu trúc đồng quy thẳng hàng, sau người bạn lại chia sẻ trân facebook rằng: Anh ta ước ao tìm đường trịn đường trịn Lester, tơi lại quan tâm đến đường tròn Lester muốn nghiên cứu đường tròn mới.

Cứ vậy, nghiên cứu nhiều cấu trúc hình học Euclid nhận dịng nghiên cứu chủ đạo hình học Euclid tạo cấu trúc để đến kết quả:

Ba điểm thẳng hàng: Các kết tiếng cấu trúc tạo ba điểm thẳng hàng định lý Menelaus, định lý Pascal, định lý Desages, định lý Gauss-Newton, định lý Droz-Frany,

Ba đường thẳng đồng quy: Các kết tiếng cấu trúc tạo ba đường thẳng đồng quy định lý Ceva, định lý Shiffler, điểm Fermat, định lý Jacobi, định lý Cartnot, định lý Kariya, định lý Kiepert,

Bốn điểm nằm đường tròn:Các kết tiếng cấu trúc tạo bốn điểm năm đường tròn định lý Lester, định lý van Lamoen,

Một đa giác đa giác đều: Tạo cấu trúc để tạo tam giác đều, hình vng, lục giác ví dụ kinh điển xu hướng định lý Napoleon, định lý Morley, định lý van Aubel,

Các hệ thức tốn học: Giữa góc, độ dài đoạn thẳng, chẳng hạn định lý Pythagoras, định lý cos, định lý sin, định lý Stewart,

(129)

Tạp

chí

onl

ine

của

cộng

đồng

những

người

yêu

T

oán

Các bạn muốn sáng tạo hình học Euclid nên tìm hiểu định lý vừa giới thiệu Các bạn nên ý hình học Euclid khơng cịn lĩnh vực tốn học mũi nhọn Nhưng việc nghiên cứu hình học Euclid lứa tuổi học sinh đặc biệt bạn từ lớp đến lớp 12 phù hợp Nó tạo hành trang vững để mai sau nghiên cứu sang lĩnh vực Toán cao cấp Ở lĩnh vực Toán cao cấp, đối tượng, cấu trúc xây dựng từ đối tượng phức tạp hơn, khó khăn có tính ứng dụng cao phương pháp nghiên cứu cách làm theo cá nhân tơi phần giúp tìm kết hay ý nghĩa

Một điều quan cần bổ sung việc tìm hiểu lịch sử vấn đề toán học giúp cho nắm bắt dịng chảy nghiên cứu tốn học

3 Vai trị cộng đồng diễn đàn toán học

Khi nghiên cứu Tốn học, bạn cịn gặp nhiều khó khăn việc học để nắm bắt đối tượng nghiên cứu nghiên cứu nên thời gian đầu có lẽ chưa thể đạt thành điều khiến cho bạn chán nản Bản thân bạn chán nản giả sử thành bạn lại không thừa nhận khiến bạn động lực sáng tạo (điều với công việc khác) Thực tế chứng minh ghi nhận, làm việc có suất có hiệu khuyến khích người ta chủ động tích cực hăng say làm việc

Do đó, vai trị quan trọng cộng đồng việc khuyến khích bạn học tập nghiên cứu nên ghi nhận đóng góp thành bạn, thành bạn khơng ghi nhận khơng khuyến khích bạn hăng say tiếp công việc làm

(130)

Tạp

chí

onl

ine

của

cộng

đồng

những

người

yêu

T

oán

lực để khiến tơi thích nghiên cứu hình học Bài tốn mà đăng trang Cut The Knot [1], mở rộng định lý Reim:

Định lý Reim:Cho điểm E,B,Dthẳng hàng, điểmF,C,G

thẳng hàng thỏa mãn điều kiện hai tứ giác EBCF,DBCG nội tiếp thì hai đường thẳngEF DGsong song.

Hình 12.1: Định lý Reim mở rộng

Tôi phát cần E,B,E,G,C,F nằm đường conic hai tứ giác EBCF,DBCG nội tiếp hai đường thẳng EF

DGsong song

Mặc dù sau biết mở rộng định lý Reim tơi cũ năm 1974, tác giả Evelyn, C J A.; Money-Coutts, G B.; and Tyrrell, J A. tác giả tìm mở rộng trược hợp đặc biệt định lý ba đường conic Tuy nhiên, thời điểm trang Cut the Knot không công bố phát khơng khuyến khích tơi tìm hiểu có phát lĩnh vực hình học

Vạn khởi đầu nan Các kết nghiên cứu thời gian đầu thường khơng có, xấu xí, hiển nhiên, lặp lại kết người khác, lúc ban đầu, thành nghiên cứu bạn chưa cộng đồng ghi nhận đừng chán nản Các bạn nghiên cứu đi, có ngày bạn ghi nhận (nếu phát bạn hay), phần thưởng to lớn thưởng tiền bạc Có người bạn xe chia sẻ với rằng: Chỉ cần có nhỏ nhắn xinh xắn mình(khơng cần phải đao to búa lớn)

(131)

Tạp

chí

onl

ine

của

cộng

đồng

những

người

u

T

ốn

học khơng có nghĩa ghi nhận cách lăng xê cá nhân bạn Vì vậy, mong muốn tơi thành lập tạp chí phổ biến Tốn học nước

Tôi nghĩ để bạn nghiên cứu có thành quả, bạn nên tham gia diễn đàn Tốn học thật nơi mà có nhà nghiên cứu trao đổi với Ở đó, bạn học hỏi trao đổi nhiều điều ý tưởng bạn dễ dàng để đến giới chuyên môn, để thẩm định hay cũ, dễ dàng việc chia sẻ ý tưởng bạn đến cộng đồng Tham gia diễn đàn nghĩa bạn nhà chuyên môn biết đến

Tơi thấy bên Mỹ có diễn đàn tạp chí lập để thảo luận riêng biệt lĩnh vực hình học, lĩnh vực số học, lĩnh vực khác lại chủ trì nhà nghiên cứu thật Thiết nghĩ, Việt Nam nên học theo mơ hình nghiên cứu

4 Phương pháp mở rộng định lý trong hình học phẳng

4.1 Phương pháp mở rộng giả thiết

Phương pháp sáng tạo vấn đề mới, mở rộng định lý hình học dựa tảng định lý có mà tác giả thường xuyên áp dụng sau:

Cho đối tượngAB Trong đóAcó đầy đủ thuộc tính củaB(hoặc thuộc tính củaBtương tự với A) Nếu định lý với các giả thiết xây dựng từ đối tượngA thì kiểm tra định lý với giả thiết xây dựng từ đối tượng B.

Để minh họa cho phương pháp xem số ví dụ sau đây:

4.1.1 Ví dụ tính chất cho tam giác trung bình mở rộng cho tam giác Cevian

(132)

Tạp

chí

onl

ine

của

cộng

đồng

những

người

yêu

T

oán

Trung điểm đường chéo tứ giác toàn phần nằm trên đường thẳng.

Hình 12.2: Định lý đường thẳng Gauss-Newton

Trong hình vẽ trên, đường thẳng cắt ba cạnh điểm

A1,B1,C1 vàA2,B2,C2 trung điểm ba đoạn thẳngAA1,BB1,CC1

thì A2,B2,C2 thẳng hàng Tuy nhiên, ta thấy ba điểmA2,B2,C2

trên nằm ba cạnh tương ứng tam giác trung bình tam giácABC ta phát biểu lại định lý sau:

Cho đường thẳng (L) cắt ba cạnh tam giác BC,CA,AB tại

A1,B1,C1 ĐặtA2,B2,C2là giao điểm đường thẳngAA1,BB1,CC1 lần lượt với ba cạnh tương ứng tam giác trung bình tam giácABC thìA2,B2,C2 thẳng hàng.

Ta mở rộng định lý thay tam giác trung bình tam giác mở rộng tam giác trung bình không? Tam giác Cevian tam giác tổng quát tam giác trung bình Thật tam giác A1B1C1 gọi tam giác Cevian tam giác

ABC ứng với điểm P AP,BP,CP giao với ba cạnh

BC,CA,AB điểm A1,B1,C1 Khi tam giác Cevian ứng với

trọng tâm tam giác tam giác trung bình tam giác ABC Như vậy, rõ ràng tam giác Cevian tổng quát tam giác trung bình

Từ sở ta đến mở rộng định lý Gass-Newton [3] sau:

Vấn đề 1: Cho đường thẳng(L) cắt ba cạnh BC,CA,ABcủa tam giác tại A1,B1,C1 Đặt A2,B2,C2 là giao điểm đường thẳng

(133)

Tạp

chí

onl

ine

của

cộng

đồng

những

người

u

T

ốn

Hình 12.3: Mở rộng định lý đường thẳng Gauss-Newton

4.1.2 Ví dụ tính chất cho tam giác Napoleon và Morley mở rộng hai tam giác Jacobi bất kỳ.

Tam giác Napoleon tam giác Morley tam giác tiếng hình học sơ cấp, khơng tìm hiểu lại khái niệm Chúng ta tìm hiểu khái niệm tam giác Jacobi, khái niệm sau:

Cho tam giác ABC, dựng tam giácBA0C,CB0A,AC0Bcùng ra ngoài (hoặc vào trong) ba cạnh tam giác ABC, cho =C0AB=

=B0AC,=C0BA==A0BC,=A0CB==B0CAkhi tam giácA0B0C0 gọi tam giác Jacobi tam giác ABC.

Đối chiếu với khái niệm trên, ta thấy tam giác Napoleon tam giác Morley trường hợp đặc biệt tam giác Jacobi Thầy Trần Quang Hùng (giáo viên THPT chuyên Tổng Hợp -ĐHQGHN) phát tính chất sau:

Cho tam giác ABC, tam giác Napoleon tam giác Morley tương ứng làMAMBMC, NANBNC gọiA1 là giao điểm củaMANA và BC, định nghĩa B1,C1 tương tự đó AA1,BB1,CC1 đồng quy

[4]

Trước đó, nhà nghiên cứu người Phần Lan tên van Lam-oen tìm tính chất sau:

Nếu gọi A1,B1,C1 là chân đường cao ba đỉnh A0,B0,C0 trên ba cạnh tam giác thìAA1,BB1,CC1 đồng quy [5]

Ta thấy đường cao kẻ từA1,B1,C1lần lượt xuốngBC,CA,AB

chính đường thẳng nối đỉnhA1,B1,C1 với tam

(134)

Tạp

chí

onl

ine

của

cộng

đồng

những

người

yêu

T

oán

vậy, ta ta quan sát kết van Lamoen Trần Quang Hùng chúng trường hợp đặc biệt vấn đề sau đây:

Vấn đề 2: Gọi JaJbJc Ja1Jb1Jc1 là hai tam giác Jacobi của ABC; gọiA0B0C0 là giao điểm đường thẳngJaJa1 vớiBC;JbJb1 với

AC; JcJc1 vớiABkhi đó AA0,BB0,CC0 đồng quy [6]

Hình 12.4: Vấn đề hai tam giác Jacobi

4.1.3 Ví dụ định lý cho giao điểm tứ giác có hai đường chéo vng góc cho điểm thỏa mãn tính chất đặc biệt tứ giác.

Mục tìm hiểu mở rộng định lý Brahmagupta tiếng tứ giác nội tiếp có hai đường chéo vng góc Brahmagupta(598-670) nhà tốn học tiếng Ấn Độ Nội dung định lý:

Cho tứ giác nội tiếp có hai đường chéo vng góc, đó đường thẳng nối trung điểm cạnh với giao điểm hai đường chéo vng góc với cạnh đối diện [7]

Ta tìm hiểu chút giả thiết kết luận định lý Cho tứ giác ABCD nội tiếp gọiF trung điểm AB, Elà giao điểm hai đường chéo, theo định lý Brahmagupta ta có FE

(135)

Tạp

chí

onl

ine

của

cộng

đồng

những

người

yêu

T

ốn

Hình 12.5: Định lý Brahmagupta

ra, tứ giác ABCD nội tiếp nên theo tớnh cht phng tớch ta cú EBăED = EAăEC T nhận xét ta tìm cách mở rộng giả thiết định lý Brahmagupta xem có giữ kết luận đúng, kết luận tương tự với kết luận định lý Brahmagupta giả thiết hay không?

Bây không đưa giả thiết góc=DEA==BEC= 90˝ nữa giữ tổng hai góc bằng 180˝, điểm

B,E,D khơng cịn thẳng hàng điểm A,E,C khơng cịn thẳng hàng, tứ giác ABCD khơng cịn nội tiếp na nhng gi iu kin EAăEC= EBăED ú có kết luận tương tự kết luận định lý Brahmagupta Tuy nhiên, việc xây dựng điểm thỏa E thỏa mãn tính chất giả thiết từ tứ giác ABCDcho trước khó khăn nên ta xây dựng tốn từ nhận xét xuất phát từ toán tam giác sau

Vấn đề 3: Dựng cạnh AB, AC của tam giác ABC

hai tam giác BAD CAE, cho =DAB + =EAC = 180˝

(136)

Tạp

chí

onl

ine

của

cộng

đồng

những

người

yêu

T

ốn

Hình 12.6: Mở rộng định lý Brahmagupta

4.1.4 Một ví dụ định lý cho hai cặp đường thẳng thì cho đường conic

Nhắc lại định lý Reim: ChoE,B,D thẳng hàng và F,C,G thẳng hàng hai tứ giác EBCF, DBCG nội tiếp hai đường thẳng EF

DGsong song Như phần ta thấy cầnE,B,E,G,C,F

nằm đường conic hai tứ giác EBCF, DBCG nội tiếp hai đường thẳng EFvà DGsong song

Như ta thấy cấu trúc định lý Reim thay hai đường thẳngEBD, vàECG đường conic qua sáu điểm

E,B,D,G,C,F kết luận định lý [9]

Đối với định lý Pascal: Cho lục giác có đỉnh nằm trên một đường conic giao điểm ba cặp cạnh đối diện thẳng hàng.

Hình 12.7: Định lý Pascal, định lý Pappus

(137)

Tạp

chí

onl

ine

của

cộng

đồng

những

người

yêu

T

oán

conic định lý Pascal hai đường thẳng với ba điểm

A,B,Cnằm đường thẳng thứ vàD,E,Fbằm đường thẳng thứ hai quan sát kết thấy G,H,I thẳng hàng [10][11], kết nội dung định lý Pappus

Qua hai ví dụ trên, ta thấy tính chất phát biểu cho cặp đường thẳng kiểm tra tính chất ta thay chúng đường conic

Định lý bướm ví dụ minh họa trường hợp Lời giải thích đường conic suy biến thành cặp đường thẳng

4.2 Tìm kết luận tương tự định lý

4.2.1 Tìm cấu trúc để tạo tam giác đều-vấn đề Dual mở rộng định lý Napoleon

Với điều kiện tam giác Hofstadter tam giác đều, câu trả lời cho câu hỏi thơng qua định lý góc chia ba Morley Với điều kiện tam giác Kiepert tam giác đều? Câu trả lời hiểu thông qua định lý Napoleon

Ta biết rằng, cho tam giác ABC điểm P, tam giác bàn đạp điểm P tam giác tạo điểm hình chiếu củaP

trên ba cạnh ABC Tam giác Cevian điểm P tam giác tạo điểm giao điểm đường thẳng từ đỉnh qua P

với cạnh đối diện Tại thời điểm bắt đầu nghiên cứu hình học phẳng, tác giả có câu hỏi tìm điểm cho tam giác bàn đạp tam giác đều, tam giác Cevian tam giác

Người ta tìm hai điểm thỏa mãn tính chất tam giác bàn đạp điểm tam giác đều, hai điểm biết đến với tên điểm Isodynamic thứ thứ hai, điểm điểm liên hợp đẳng giác điểm Fermat

(138)

Tạp

chí

onl

ine

của

cộng

đồng

những

người

yêu

T

oán

Như câu hỏi tìm điều kiện để tam giác bạn đạp tam giác đều, hay tìm điều kiện để tam giác Cevian tam giác quan tâm làm rõ Tác giả lại đặt câu hỏi tìm điểm cho đối xứng điểm qua ba cạnh tam giác ABC cho trước tam giác đều, tác giả tìm điểm lại hai điểm Isodynamic, việc chứng minh điều thật dễ dàng tam giác vị tự tam giác bàn đạp hai điểm

Nếu dừng lại kết trở nên bình thường khơng có đáng nói hồn tồn vấn đề tìm điểm cho tam giác bàn đạp tam giác Nhưng điều đặc biệt đường thẳng nối đỉnh tương ứng tam giác với tam giác ABC lại đồng quy điểm Fermat Do đỉnh tam giác xác định đối xứng điểm Isodynamic với ba cạnh tam giác điều có suy đường thẳng nối điểm Isodynamic với ba đỉnh tam giác vng góc với ba cạnh tương ứng tam giác

ABC

Sau nghiên cứu đưa mở rộng định lý Napoleon tác giả nhớ lại kết nghiên cứu đánh dấu in nghiêng đoạn tác giả phát vấn đề mở rộng vấn đề đối xứng điểm qua ba cạnh tam giác tạo thành tam giác sau

Vấn đề 4:Cho tam giácABC,Flà điểm Fermat thứ thứ hai,Ilà điểm liên hợp đẳng giác củaF,P là điểm nằm đường thẳngFI.A0 là giao điểm đường thẳng quaP và vng góc với cạnh BC và đường thẳng AF Định nghĩa B0,C0 tương tự Chứng minhA0B0C0 là tam giác đều [13]

Trường hợp đặc biệt xảy khiP điểm Symmedian Với trường hợp ta có hai tam giác Dual với tam giác Napeleon Hai tam giác có nhiều tính chất thú vị Một tính chất là:

Vấn đề 5: Bốn điểm gồm hai điểm Fermat trọng tâm hai tam giác vừa đề cập nằm đường tròn

(139)

Tạp

chí

onl

ine

của

cộng

đồng

những

người

u

T

ốn

Hình 12.8: Vấn đề dual định lý Napoleon

Cho tứ giác ABCD ngoại tiếp đường tròn, gọiE là giao điểm của hai đường chéo Khi tâm bốn đường trịn nội tiếp tứ giácEAB,EBC, ECD, EDAnằm đường trịn [14]

Hình 12.9: Định lý Christopher Bradley

Câu hỏi tương tự: Cho tứ giác ABCD ngoại tiếp đường trịn, tìm một điểm P trên mặt phẳng cho tâm nội tiếp tam giác

PAB, PBC, PCD, PDA nằm đường tròn. Từ câu hỏi tác giả tìm đươc số vấn đề [15] sau đây:

(140)

Tạp

chí

onl

ine

của

cộng

đồng

những

người

yêu

T

oán

Hình 12.10: Vấn đề

Tác giả tìm hai điểm nữa thỏa mãn tính chất bốn tâm nội tiếp nằm đường tròn Hai đề hiểu thông qua khái niệm tứ giác trung trực tứ giác ngoại tiếpsau

Cho tứ giác lồiABCDngoại tiếp đường tròn, bốn đường trung trực bốn cạnh tứ giác hợp với thành tạo thành tứ giác ngoại tiếp.

Tính chất biết đến sách Mike de Villiers, chứng minh đưa Jordan Tabov [16] Nhưng trước xuất tài liệu bên Nga [17] Hiện chưa thống tên gọi riêng cho tứ giác nên tạm gọi tứ giác với tên tứ giác trung trực tứ giác ngoại tiếp.

Vấn đề 7:Gọi Olà tâm đường tròn nội tiếp tứ giác trung trực của tứ giác ngoại tiếp ABCD khi tâm bốn đường tròn nội tiếp các tam giácOAB, OBC, OCD, ODAnằm đường tròn.

Vấn đề 8: Gọi L giao điểm hai đường chéo tứ giác trung trực tứ giác ngoại tiếp ABCD khi tâm bốn đường tròn nội tiếp tam giácLAB, LBC,LCD,LDAnằm đường tròn.

(141)

Tạp

chí

onl

ine

của

cộng

đồng

những

người

yêu

T

ốn

Hình 12.11: Tứ giác trung trực tứ giác ngoại tiếp

Hình 12.12: Vấn đề

Hình 12.13: Vấn đề

4.3 Dịng chảy mở rộng định lý Napoleon

(142)

Tạp

chí

onl

ine

của

cộng

đồng

những

người

yêu

T

oán

được phát biểu sau:

Cho tam giác ABC, dựng ba tam giác đều BA0C, CB0A, AC0B ra ngoài (hoặc vào trong) ba cạnh tam giác ABC; đó, trọng tâm tam giác đỉnh tam giác đều.

Tam giác gọi tam giác Napoleon ngồi (hoặc trong) tam giácABC [18]

Hình 12.14: Định lý Napoleon

Định lý Napoleon định lý đẹp hình học cổ điển, thu hút quan tâm nghiên cứu nhiều người từ nhà toán học, giáo viên học sinh đến người nghiên cứu nghiệp dư Các nghiên đưa nhiều cách chứng minh khác cho định lý, chứng minh dựa vào lượng giác, số phức tọa độ

Một số định lý tiếng khác lấy cảm hứng từ định lý Napoleon, thay cấu trúc dựng cạnh tam giác tam giác định lý van Aubel nội dung dựng ngoài một tứ giác hình vng đoạn thẳng nối tâm hai hình vng hai cạnh đối diện vng góc nhau [20], hay định lý Thebault với nội dung dựng ngồi hình bình hành hình vng tâm các hình vng là các đỉnh hình vng

(143)

Tạp

chí

onl

ine

của

cộng

đồng

những

người

u

T

ốn

Hình 12.15: Định lý van Auble định lý Thebault

tam giác tam giác đều, Kiepert dựng ba cạnh tam giác tam giác cân đồng dạng ông quan sát thấy kết thú vị thể quađịnh lý Kiepert sau:

Cho tam giác ABC, dựng ba tam giác cân đồng dạng BA0C,

CB0A, AC0Bra (hoặc vào trong) ba cạnh tam giác ABC khi đó AA0,BB0,CC0 đồng quy [21]

Điểm đồng quy có tên chung điểm Kiepert Giả sử góc đáy tam giác cân θthì ta có điểm đồng quy tương ứng với góc θ ký hiệu K(θ) Tuy nhiên số trường hợp đặc biệt điểm Kiepert có tên riêng, góc θ = 45˝ thì điểm Kiepert gọi với tên riêng là

điểm Vecten, θ=60˝ thì điểm Kiepert gọi với

(144)

Tạp

chí

onl

ine

của

cộng

đồng

những

người

yêu

T

oán

Napoleon vng góc với ba cạnh tam giác đồng quy tâm đường tròn ngoại tiếp Nếu ta vẽ hình quan sát thấy: Điểm Fermat thứ nhất, điểm Napoleon tâm đường tròn ngoại tiếp thẳng hàng Tương tự ba điểm: Điểm Fermat thứ hai, điểm Napoleon tâm đường trịn ngoại tiếp thẳng hàng. [22]

Hình 12.16: Điểm Fermat, điểm Napoleon, O thẳng hàng Từ quan sát đánh dấu chữ nghiêng tác giả tìm vấn đề mở rộng định lý Napoleon sau

Vấn đề 1: Cho tam giácABC, Flà điểm Fermat ngoài,

Klà điểm Kiepert,P là điểm nằm đường thẳngFK Định nghĩa

A0 là giao điểm của AK và đường thẳng qua P và vng góc với

BC Định nghĩa B0,C0 tương tự Khi tam giác A0B0C0 là tam giác đều [23]

(145)

Tạp

chí

onl

ine

của

cộng

đồng

những

người

yêu

T

oán

Hình 12.17: Mở rộng định lý Napoleon với hyperbol Kiepert Nhà hình học Kariya đưa ý tưởng tương tự (với cấu trúc định lý Kiepert) áp dụng với trường hợp ba đường vng góc với ba cạnh đồng quy tâm nội tiếp(thay tâm ngoại tiếp), đoạn thẳng thay tỉ lệ với ba cạnh đoạn thẳng biến thành định lý Kariyanhư sau:

Gọi I là tâm đường tròn nội tiếp tam giácABC,dựng ba tam giác

BA0C, CB0A, AC0B cùng (hoặc vào trong) ba cạnh tam giácABC,sao cho IA0 =IB0 =IC0 Ikhi đóAA0,BB0,CC0 đồng quy [24].

Khi nghiên cứu định lý Kariya tác giả đặt câu hỏi định lý có cịn đúng ta thay tâm nội tiếp tâm bàng tiếp, câu trả lời mong nhận từ bạn đọc

Tiếp theo quan sát thấy định lý Kiepert định lý lý Kariya, có điểm chung góc=C0AB==B0AC,

=C0BA = =A0BC, =A0CB = =B0CA Và ta thử tạo toán với

phát biểu: Cho tam giác ABC, dựng tam giác BA0C, CB0A,

AC0B cùng (hoặc vào trong) ba cạnh tam giác ABC, sao cho =C0AB = =B0AC, =C0BA = =A0BC, =A0CB = =B0CA, nếu ta nối đường thẳng AA0,BB0,CC0 ta quan sát thấy ba đường thẳng đồng quy. Kết nội dung định lý Jacobi, tam giác A0B0C0 gọi tam giác Jacobi

(146)

Tạp

chí

onl

ine

của

cộng

đồng

những

người

yêu

T

ốn

Hình 12.18: Định lý Jacobi

đã đưa cấu trúc đặc biệt góc =C0AB = =B0AC = k.=A,

=C0BA = =A0BC = k.=B, =A0CB = =B0CA = k ă=C, i vi

trng hợp tam giácA0B0C0 gọi tam giác Hofstadter

[25]

Tóm lại: Định lý với điểm kiểm tra với điểm khác, tìm quỹ tích điểm thỏa mãn Định lý với đường thẳng song song kiểm tra với đường thẳng cắt Định lý đúng với tam giác kiểm tra với tam giác cân, với tam giác Định lý với tứ giác đặc biệt kiểm tra nó với tứ giác đặc biệt

5 Xử lý ý tưởng

(147)

Tạp

chí

onl

ine

của

cộng

đồng

những

người

u

T

ốn

tìm kiếm, tra cứu kết quả, nghiên cứu phát triển hình thành ý tưởng

Đối với lĩnh vực hình học hai từ điển sau dùng để tra cứu có giá trị:

1 Từ điển bách khoa điểm đường đặc biệt tam giác, trang web: http://faculty.evansville.edu/ck6/ encyclopedia/ETC.html nói tâm tam giác(các điểm đặc biệt) xây dựng Clack Kimberling giáo sư trường University of Evansville Các tâm tam giác đánh số theo mục theo thứ tựX(1),X(2), ,X(n), chẳng hạn, tâm nội tiếp X(1), trọng tâm làX(2), tâm ngoại tiếp làX(3), trực tâm là(4), Và liệt kê gần đầy đủ tính chất quan trọng từ lớn đến nhỏ tâm tam giác Hiện nay, từ điển liệt kê khoảng 6400

điểm đường đặc biệt tam giác

2 Một từ điển hàn lâm khác đáng tham khảo bách khoa thư đường bậc ba, từ điển ta sử dụng để tìm quỹ tích điểm thỏa mãn tính chất đó: http://bernard.gibert.pagesperso-orange.fr/ctc html Bách khoa thư mơ tả đường cubic tam giác, đường cubic qua tâm tam giác Ta thử lấy ví dụ để nêu cách áp dụng từ điển cho việc tra cứu

Ví dụ tra cứu:Xuất phát từvấn đề IV Thebault:H trực tâm tam giác ABC, vàHa,Hb,Hc là chân ba hình chiếu củaHnên ba cạnh tam giácABC, đường thẳng Euler của các tam giácAHbHc, BHcHa CHaHbđồng quy.

Khi thành thạo phương pháp mở trình bày trên, nhanh chóng, tơi đặt câu hỏi tìm điểmP mặt phẳng tam giác ABC, cho vớiPa,Pb,Pc hình chiếu P ba cạnh tam giác ABC, đường thẳng Euler tam giácAPbPc,BPcPavàCPaPb đồng quy

(148)

Tạp

chí

onl

ine

của

cộng

đồng

những

người

yêu

T

ốn

khác ví dụ trực tâm, trọng tâm, hai điểm Fermat, tâm ngoại tiếp trình kiểm tra ta thấy tính chất điểm P tâm đường trịn chín điểm X(5) Tiếp theo ta tra Catalogue thấy:

K005 NAPOLEON CUBIC, FEUERBACH CUBIC, pK(X6, X5), D(0) X(1), X(3), X(4), X(5), X(17), X(18), X(54), X(61), X(62), X(195), X(627), X(628), X(2120), X(2121), X(3336), X(3460), X(3461), X(3462), X(3463), X(3467), X(3468), X(3469), X(3470), X(3471), X(3489), X(3490)

Ta thấy đường Napoleon cubic đường Cubic mà ta thấy qua điểm tâm đường tròn nội tiếp X(1), trực tâm

X(4), tâm đường trịn chín điểm X(5) Ta dự đốn tính chất nều P nằm đường Napoleon Cubic

Tiếp theo, ta click vào thấy tính chất nêu tính chất đường Napoleon cubic Tuy nhiên, giả sử chưa nêu từ điển ta tiếp tực kiểm tra với điểm X(17),X(18), ta thấy Và dự đoạn ta tính chất P

nằm đường Napoleon Cubic Tất nhiên trường hợp xảy ta có nghiên cứu tính chất đường Napoleon cubic(tuy nhiên điều giả tưởng thực tế nghiên cứu)

Nếu kết có lẽ đến bước quan trọng tìm cách chứng minh cơng bố kết Cơng bố kết có nhiều cách, ví dụ đưa lên blog cá nhân, diễn đàn đề xuất lên tạp chí gửi đến số nhà tốn học uy tín, tìm cách chứng minh viết báo độc lập cộng tác với người khác nghiên cứu viết báo

Hiện hình học cổ điển tơi khun cáo bạn cơng bố tạp chí Crux hội tốn học Canada, tạp chí AMM hội tốn học Mỹ, tạp chí Forum Geometricorum khoa tốn đại học Florida Atlantic

(149)

Tạp

chí

onl

ine

của

cộng

đồng

những

người

yêu

T

oán

2 Tạp chí AMM: Một tạp chí tốn hàng tháng hội Toán học Mỹ, xuất báo vấn đề hình học cổ điển http://www.maa.org

3 Tạp chí Forum Geometricorum: Đây tạp chí điện tử, miễn phí, hàn lâm lĩnh vực hình học cổ điển Tạp chí khoa Tốn đại học Florida Atlantic, Mỹ Tạp chí nhận đăng báo hoàn chỉnh: http: //forumgeom.fau.edu/

6 Bài tập thực hành

Bài toán 1. Cho tam giác ABC, F điểm Fermat, ba đường thẳng qua G vng góc với FA,FB,FC cắt ba cạnh tam giác A0,B0,C0 ta thấy A0,B0,C0 thẳng hàng Hãy kiểm tra tính

chất thay điểm trọng tâm G điểm Vecten, điểm Napoleon sau kiểm tra với điểm nằm quỹ tích thỏa mãn tính chất này, đến kết luận tổng quát

Bài toán 2. Gọi O tâm đường tròn ngoại tiếp tam giác

ABC, đường thẳng Euler tam giácOBC,OCA,OAB

đồng quy Hãy kiểm tra tính chất ta thay điểm O điểm tâm đường tròn nội tiếp, trực tâm, tâm đường trịn chín điểm Tìm quỹ tích điểm thỏa mãn tính chất

Bài toán 3. Cho tam giác ABC, đường thẳng Lcắt ba cạnh

BC,CA,AB tam giác A0,B0,C0 S điểm bất

kỳ đường thẳng L Gọi A1,B1,C1 điểm đối xứng

A0,B0,C0 quaSthìAA1,BB1,CC1 đồng quy Hãy kiểm tra kết

bài tốn ta thaySbởi đường trịn phép đối xứng qua điểm phép nghịch đảo qua đường trịn

Bài tốn 4. Cho hình vng ABCD dựng cạnh AB

BC hai tam giác AMB, BNC tam giác DMN tam giác Hãy kiểm tra tính chất thay hình vng

ABCD hình bình hành ABCD

Bài tốn 5. Dựng ba cạnh tam giácABCcùng (hoặc vào trong) tam giác BA0C, CB0A, AC0B ta thấy

AA0,BB0,CC0 đồng quy, kiểm tra tính chất thay ba

(150)

Tạp

chí

onl

ine

của

cộng

đồng

những

người

yêu

T

oán

Bài tốn 6. Định lý bảy đường trịn phát biểu rằng: Cho sáu đường tròn mặt phẳng cho đường tròn tiếp xúc với hai đường tròn liền kề Sáu đường trịn tiếp xúc với đường trịn thứ bảy Hãy thử tìm tính chất toán xây dựng từ giả thiết: Cho sáu đường tròn, đường tròn tiếp xúc với hai đường tròn liền kề cho điểm tiếp xúc nằm đường tròn

Tài liệu tham khảo

[1] A Bogomolny, Two Circles, Ellipse, and Parallel Lines, available at http://www.cut-the-knot.org/pythagoras/ CircleCircleEllipse.shtml

[2] R A Johnson, A Modern Geometry: An Elementary Treatise on the Geometry of the Triangle and the Circle, Houghton Mif-flin, Boston, 1929, pp.152–153

[3] O T Dao, Advanced Plane Geometry, message 1283, May 9, 2014

[4] H Q Tran, Advanced Plane Geometry, message 1736, September 19, 2014

[5] Darij Grinberg, Math Forum, Synthetic Proofs of Jacobi and Lamoen Theorems, March 21, 2003

[6] O T Dao, Advanced Plane Geometry, message 1765, September 19, 2014

[7] R Honsberger, Episodes in Nineteenth and Twentieth Cen-tury Euclidean Geometry, MAA, 1995

[8] O T Dao, Quadri-Figures-Group, message 241, September 23, 2013

[9] Evelyn, C J A.; Money-Coutts, G B.; and Tyrrell, J A The Seven Circles Theorem and Other New Theorems London: Stacey International, pp 11-18, 1974

(151)

Tạp

chí

onl

ine

của

cộng

đồng

những

người

yêu

T

oán

[11] Johnson, R A Modern Geometry: An Elementary Treatise on the Geometry of the Triangle and the Circle Boston, MA: Houghton Mifflin, pp 237-238, 1929

[12] Jiang Huanxin and David Goering, Problem 10358* and Solution, Equilateral cevian triangles, American Mathemati-cal Monthly 104 (1997) 567-570

[13] O T Dao, Advanced Plane Geometry, message 2267, Jan-uary 25, 2015

[14] Darij Grinberg, CTK Exchange, Synthetic proof of Christo-pher Bradley’s Conjecture, January 11, 2004

[15] O T Dao, Quadri-Figures-Group, message 409, January 16, 2014

[16] M de Villiers, Some Adventures in Euclidean Geometry, Univ of Durban-Westville, 1994 (revised 1996), pp 192-193 [17] Titanium, Tangential quadrilateral well known properties?

http://artofproblemsolving.com,January16,2014

[18] Coxeter, H S M and Greitzer, S L Geometry Revisited Washington, DC: Math Assoc Amer., pp 60-65, 1967 [19] van Aubel, H H Note concernant les centres de carrés

con-struits sur les côtés d’un polygon quelconque.Nouv Corresp Math 4, 40-44, 1878

[20] Casey, J.A Treatise on the Analytical Geometry of the Point, Line, Circle, and Conic Sections, Containing an Account of Its Most Recent Extensions with Numerous Examples, 2nd rev enl ed Dublin: Hodges, Figgis, & Co., pp 442-448, 1893 [21] Kimberling.C, Encyclopedia of Triangle Centers, http://

faculty.evansville.edu/ck6/encyclopedia/ETC.html

[22] O T Dao, Advanced Plane Geometry, message 2261, Jan-uary 24, 2015

[23] D Grinberg, Hyacinthos message 10504, September 20, 2004

(152)

Tạp

chí

onl

ine

của

cộng

đồng

những

người

yêu

T

(153)

T p c h í

online

cộng đồng

những n g i y ê u T o n

Tạp

chí

onl

ine

của

cộng

đồng

những

người

yêu

T

oán A JOURNEY INTOINEQUALITIES

FIBONACCI & LUCAS NUMBERS

Vandanjav Adiyasuren

National University of Mongolia

Bold Sanchir

Mongolian University of Life Sciences

1 Fibonacci and Lucas numbers

The Fibonacci numbers may this be defined as follows:

F0 =0,F1 =1and Fn =Fn´1+Fn´1, ně2 (˚)

The sequence of Fibonacci numbers is:

0, 1, 1, 2, 3, 5, 8, 13, 21, 34, 55,

The Lucas numbers may this be defined as follows:

L0 =2,L1 =1 and Ln=Ln´1+Ln´2, ně2

The sequence of Lucas numbers is:

2, 1, 3, 4, 7, 11, 18, 29, 47, 76, 123,

Both Fibonacci and Lucas numbers satisfy numbers identities that have been discovered over the centuries In this part we explore several of these fundamental identities

Proposition 1.1.

n

ÿ

k=1

(154)

Tạp

chí

onl

ine

của

cộng

đồng

những

người

yêu

T

oán

Proof. Using the Fibonacci recurrence relation, we have

F1 =F3´F2

F2 =F4´F3

Fn´1 =Fn+1´Fn

Fn=Fn+2´Fn+1

Adding these equations, we get n

ÿ

k=1

Fk =Fn+2´F2 =Fn+2´1

Proposition 1.2.

a)

n

ÿ

k=1

F2k´1 =F2n (2)

b)

n

ÿ

k=1

F2k =F2k+1´1 (3)

Proof. a) Using the Fibonacci recurrence relation, we have

F1 =F2 ´F1

F3 =F4 ´F2

F2n´3 =F2n´2´F2n´4

F2n´1 =F2n´F2n´2

Adding these equations, we get n

ÿ

k=1

F2k´1 =F2n´F0 =F2n

b)

n

ÿ

k=1

F2k= 2n

ÿ

k=1

Fk´ n

ÿ

k=1

F2k´1

( by Proposition 1.1.1 and 1.1.2 (a)) = (F2n+2´1)´F2n

(155)

Tạp

chí

onl

ine

của

cộng

đồng

những

người

yêu

T

oán

Proposition 1.3.

Fn´1Fn+1´F2n = (´1) n

(4)

whereně1

Proof. (By PMI) Since F0F2F21 =0ă11=1= (1)1, the given

statement is clearly true when n = Now we assume it is true for an arbitrary positive integer n:Fn´1Fn+1´F2n = (´1)n Then

FnFn+2´Fn2+1 = (Fn+1´Fn´1)(Fn+Fn+1)´F2n+1

=FnFn+1+F2n+1´Fn´1Fn´Fn´1Fn+1´F2n+1

(by the IH:)

=FnFn+1´Fn´1Fn´F2n´(´1) n

=FnFn+1´Fn(Fn´1 +Fn) + (´1)n+1

=FnFn+1´FnFn+1+ (´1)n+1 = (´1)n+1

Thus the formula works for (n+1).So, by PMI, the statement is true for every integer ně1

Proposition 1.4.

n

ÿ

k=1

F2k =FnFn+1 (5)

Proof. (by PMI) When n=1, the

LHS=

1

ÿ

k=1

F2k =F21 =1 =1ă1=F1ăF2 =RHS

So the result is true when n= Assume it is true for an arbi-trary positive integer n:

n

ř

k=1

F2k =FnFn+1 Then n+1

ÿ

k=1

F2k =

n

ÿ

k=1

F2k+F2k+1

(by the IH:)

=FnFn+1+F2n+1 =Fn+1(Fn+Fn+1) =Fn+1Fn+2

(156)

Tạp chí onl ine của cộng đồng những người yêu T oán

Interestingly enough, Identities (1)´(5) have analogous results for Lucas number also:

n

ÿ

k=1

Lk =Ln+2´3 (6)

n

ÿ

k=1

L2k´1 =L2n´2 (7) n

ÿ

k=1

L2k =L2n+1 (8)

Ln1Ln+1L2n=5ă(1) n1

(9)

n

ÿ

k=1

L2k =LnLn+1´2 (10)

These identities can be established using PMI

Proposition 1.5. (Binet’s Formula)

Fn=

1 ?

1+?5

n ´

1´?5

n! ,

whereně1

Proof. This can be established easily using PMI

1+?5

n =

1+?5

Fn+Fn´1,ně1

1´?5

n =

1´?5

Fn+Fn´1,ně1

Let Un = ?15

h

1+?5

´

1´?5

ni

,where ně1

ThenU1 = ?15

1+?5

2 ´

1´?5

=1andU2 = ?15

1+?5

2 ´

1´?5

2 = 1.Suppose ně3.Then

Un´1 +Un´2 =

1 ?

1+?5

n´1 ´

1´?5

n´1! +

+?1

1+?5

n´2 ´

1´?5

n´2!

= ?1

1+?5

n ´

1´?5

(157)

Tạp

chí

onl

ine

của

cộng

đồng

những

người

yêu

T

oán

This, Un satisfies the FRR(˚) and the two initial conditions This gives us an explicit formula for Fn:

Fn=Un=

1 ?

1+?5

n ´

1´?5

n!

Hệ 1.1.

F2n+1+F2n=F2n+1 (11)

F2n+1´F2n´1 =F2n (12) Corresponding to Binet’s formula forFn, there is one for Lnalso

Proposition 1.6. Letně1Then

Ln=

1+?5

n +

1´?5

n

Hệ 1.2.

F2n =FnLn (13)

Fn´1+Fn+1 =Ln (14)

Fn+2´Fn´2 =Ln (15)

Ln´1 +Ln+1 =5Fn (16)

Proposition 1.7.

n

ÿ

k=1

FkFk+2 =

1

2(FnFn+1+F

2

n+2´1) (17)

Proof. (by PMI) When n=1, the

(LHS) =F1F3 =2=

1

2(F1F3+F

2

3´1) = (RHS)

So the result is true when n=1

Assume it is true for an arbitrary positive integern:

n

ÿ

k=1

FkFk+2 =

1

2(FnFn+1+F

2

(158)

Tạp chí onl ine của cộng đồng những người yêu T oán

Then, by the IH and by the Fibonacci recurrence relation: n+1

ÿ

k=1

FkFk+2 = n

ÿ

k=1

FkFk+2+Fn+1Fn+3

=

2(FnFn+1+F

2

n+2´1) +Fn+1Fn+3

=

2 Fn+1(Fn+Fn+1) +2Fn+1Fn+3+ (F

2

n+2´F

n+1)´1

=

2(Fn+1Fn+2+2Fn+1Fn+3+FnFn+3´1) =

2(Fn+1Fn+2+ (2Fn+1+Fn)Fn+3´1) =

2(Fn+1Fn+2+F

2

n+3´1)

So the statement is true when (n+1) Thus it is true for every positive integer n

Proposition 1.8.

n

ÿ

k=1

n k

Fk =F2n (18)

Proof. Let gn=

n

ř

k=1 n k

Fk, and lethn= n

ř

k=0 n k

Fk+1.We have

gn+1´gn= n+1

ÿ

k=1

n+1

k

Fk´ n

ÿ

k=1

n k

Fk

=Fn+1+ n

ÿ

k=1

n+1

k ´ n k Fk

=Fn+1+ n

ÿ

k=1

n k´1

Fk = n+1

ÿ

k=1

n k´1

Fk

=

n

ÿ

k=0

n k

(159)

Tạp chí onl ine của cộng đồng những người yêu T oán and

hn+1´2hn=hn+1´hn´hn

=

n

ÿ

k=0

n k

Fk+2´ n

ÿ

k=0

n k

Fk+1

=

n

ÿ

k=0

n k

(Fk+2´Fk+1)

=

n

ÿ

k=0

n k

Fk =gn From these results we get

gn+1´gn =hn

hn+1´2hn =gn

*

ñgn=3gn´1´gn´2

Thus we have

Fn=Fn´1+Fn´2 = (Fn´2+Fn´3) +Fn´2 =2Fn´2+Fn´3,

which implies, viaFn´3 =Fn´2´Fn´4,thatFn=3Fn´2´Fn´4.Since

g1 = = F2 and g2 = = F4, the sequence tgnu has the same initial conditions and satisfies the same recurrence relation as the even Fibonacci numbers Thus F2n =gn =

n

ř

k=1 n k

Fk

2 Inequalities including Fibonacci and Lu-cas numbers

Problem 2.1. Let nbe a positive integer Prove that n

ÿ

k=1

L2 k

Fk

ě (Ln+2´3)

2

Fn+2´1

(Juan Jose Egozcue)

Proof. Using Cauchy-Schwarz Inequality we get

(Ln+2´3)2 = n

ÿ

k=1

Lk

!2 =

n

ÿ

k=1

Lk ? Fk ă a Fk !2 n ÿ

k=1

L2 k

Fk

! n ÿ

k=1

Fk

!

=

n

ÿ

k=1

L2 k

Fk

!

(160)

Tạp chí onl ine của cộng đồng những người yêu T oán

Here we used n

ř

k=1

Lk =Ln+2´3, n

ř

k=1

Fk=Fn+2´1.Equality occurs

only iff n=1

Problem 2.2. Let nbe a positive integer Prove that n

ÿ

k=1

Fk

1+Lk

2

ě

FnFn+1 n

ÿ

k=1

F2 k

1+Lk

!2

ěF3nF n+1

n

ÿ

k=1

F2k(1+Lk)

!´2

(Jose Luis Diaz-Barrero)

Proof. Using Cauchy-Schwarz Inequality and the identity

n

ÿ

k=1

F2k=

FnFn+1, we get n

ÿ

k=1

F2k

1+Lk

!2 =

n

ÿ

k=1

Fkă

Fk

1+Lk

!2 ď

n

ÿ

k=1

F2k

! n ÿ

k=1

Fk

1+Lk

2

=FnFn+1 n

ÿ

k=1

Fk

1+Lk

2 ,

and

F2nF2n+1 =

n

ÿ

k=1

F2k

!2 =

n

ÿ

k=1

Fk

? 1+Lk

ăFk

a 1+Lk

!2

ď

n

ÿ

k=1

F2k

1+Lk

! n ÿ

k=1

F2k(1+Lk)

!

Therefore the desired inequalities proved

Problem 2.3. Let Tn be the nth triangular number defined by

Tn= n+21

for allně1.Prove that

1

n2 n

ÿ

k=1

Tk Fk ě T n+1

9FnFn+1

(Jose Luis Diaz-Barrero)

Proof. Note that

nTn+1

3 =

n(n+1)(n+2)

6 =

n+2

=

n

ÿ

k=1

(161)

Tạp chí onl ine của cộng đồng những người yêu T oán

Using Cauchy-Schwarz Inequality, we get n

ÿ

k=1

Tk = n

ÿ

k=1

Tk

Fk

Fk ď

g f f e n ÿ

k=1

Tk Fk g f f e n ÿ

k=1

F2 k

which is equivalent to the desired inequality, after using follow-ing well known identity,

F21+F22+ă ă ă+F2n =FnFn+1

Problem 2.4. Let n be a positive integer and lbe a whole num-ber Then

Fl1+Fl2+ă ă ă+Fln

1

Fl´4

+

Fl´4

+ă ă ă+

Fl4 n

ěF2nF n+1

(P.G.Popescu, J.L.Diaz-Barrero)

Proof. Taking account that F2

1+F22+ă ă ă+Fn2 =FnFn+1, as is well

known, and the fact that the function f: (0,∞)ÑR, defined by

f(x) = x1 is convex, then by Jensen’s Inequality we get

1

Fl

1

Fn+1 +

Fl

2

Fn+1 +ă ¨ ¨+

Fl n Fn+1

ď

F2 nFn+1

Fl´4

+

Fl4

+ă ă ă+

Fl´4 n

From the preceding expression immediately follows

Fl1+Fl2+ă ă ă+Fln

1

Fl´4

+

Fl´4

+ă ă ă+

Fl4 n

ěF2nF n+1

and this completes proof

Problem 2.5. Let α1,α2, ,αn be positive real numbers Prove that

FnFn+1

n

n ě

cyc

1F21+ă ă ă+nF2n

1 +ă ¨ ¨+αn

ě(F1F2¨ ¨ ¨Fn)2

(162)

Tạp chí onl ine của cộng đồng những người yêu T oán

Proof. First, we prove that, for all positive real numbersx1,x2, ,xn

1

n

n

ÿ

k=1

xk

!n ě

n

k=1

1xk+ă ă ă+nxk+n1

1+ă ă ă+n

x1x2ă ă ăxn (1) If Ak =

1xk+ăăă+nxk+n1

1+ăăă+n then by weighted AM-GM Inequality Ak (xk1x

2

k+1ă ă ăx n k+n1)

1

1+ăăă+n.

Taking the product over k = 1, 2, ,n gives the right-hand side inequality of (1).Again using AM-GM Inequality we get

n

ź

k=1

Ak ď

1

n

n

ÿ

k=1

Ak !n = n n ÿ

k=1

xk

!n

This proves the left-hand side inequality of (1)

To solve the given inequality, in (1) take xk = F2k and use the identity

n

ř

k=1

F2k =FnFn+1

Problem 2.6. Let nbe a positive integer Prove that

1

F1F3

+

F2F4

+ă ă ă+

FnFn+2

ě 2n

2

FnFn+1+F2n+2´1

(P.G.Popescu)

Proof. Using following identity

n

ÿ

k=1

FkFk+2 =

1

2 FnFn+1+F

2

n+2´1

and by Cauchy-Schwarz Inequality, we get n

ÿ

k=1

1

FkFk+2

ě n

2 n

ř

k=1

FkFk+2

= 2n

2

FnFn+1+F2n+2´1

Problem 2.7. Let nbe a positive integer Prove that

1

Fn+2

+

n

ÿ

k=1

d F2 k + F2 k+1

ă1+

n

ÿ

k=1

1

Fk

(163)

Tạp chí onl ine của cộng đồng những người yêu T oán Proof. Denote

L(n) =

Fn+2

+

n

ÿ

k=1

d F2 k + F2 k+1

,

R(n) =1+

n

ÿ

k=1

1

Fk

If we can prove that

L(n+1)´L(n)ăR(n+1)´R(n), (ně1)

the result would follow by induction, since thenL(n+1)ăL(n)´

R(n) +R(n+1)ăR(n+1) Therefore

Dn=R(n)´R(n´1)´(L(n)´L(n´1)) =

1

Fn

+

Fn+1

´

Fn+2

´ ?

F2n+1

FnFn+1

=x+y´ xy

x+y´

a

x2+y2 = x

2 +xy+y2´(x+y)?x2+y2

x+y

= x

2y2

(x+y)[x2+xy+y2+ (x+y)?x2+y2] ą0

where x = Fn1 , y= Fn1 +1

Problem 2.8. For x is a real number and nis a positive integer, prove the following inequality

Sn =

F2 F1 x + F3 F2 x +ă ă ă+

Fn+1

Fn

x

ěn+xlnFn+1

(Zdravko F.Starc)

Proof. From the AM-GM Inequality, we get

Sěn

Fn+1

Fn

xn

=nexpx

nlnFn+1

ěn+xlnFn+1,

where we have used the inequality exě1+x, x ě0 Problem 2.9. Prove that

FF1

1 F F2

2 ă ă ăF Fn n e

(Fn1)(Fn+11)

(164)

Tạp chí onl ine của cộng đồng những người yêu T oán

Proof. Recall that

n

ř

i=1

Fi = Fn+2 ´1 and n

ř

i=1

F2

i = FnFn+1 Since

f(x) = lnx functions is concave on (0,+∞) Using Jensen’s In-equality we get

n

ÿ

i=1

Fi

Fn+2´1

lnFi ďln n

ÿ

i=1

F2 i

Fn+2 ´1

! =ln

FnFn+1

Fn+2´1

ď FnFn+1

Fn+2´1

´1= (Fn´1)(Fn+1´1)

Fn+2´1

Thus,

n

ÿ

i=1

FilnFiď(Fn´1)(Fn+1´1)

Taking exponentials yields the result

Problem 2.10. Let ną2 be an integer Prove that n

ź

k=2 k

ÿ

j=1

1

(Fk+2´Fj´1)2

!

ě

FnFn+1

n F3F4ă ă ăFn

2

(Jose Luis Diaz-Barrero)

Proof. We first prove that

kFkFk+1 ě(Fk+2´1)2 (kě1.) (1)

For k =1, , 7, this can be verified directly If kě 8,then Fk+2 ă

2Fk+1 ă4Fk,and thereforekFkFk+1 ą(k/8)F2k+2 ěF2k+2 ą(Fk+2´1)2

Let kě2.The function

f(x) =

(Fk+2´x´1)2

, 0ďxďFk

is convex, as is seen from its second derivative From (1) and applying Jensen’s Inequality and using well known řkj=1Fj =

Fk+2´1, we get k

ÿ

j=1

1

(Fk+2´Fj´1)2

ě

k k´1

2

k

(Fk+2´1)2

ě

k k´1

2

k FkFk+1

(165)

Tạp chí onl ine của cộng đồng những người yêu T oán

Problem 2.11. Let ně0 be a non-negative integer Prove that

FLnn LFnn ď(FFnn++11)

(Juan Jose Egozcue)

Proof. We shall prove that, for all non-negative integers n,

FLnn LFnn ďFFn+1

2n ďF 2Fn+1

n+1 , (1)

with equality on the left-hand side only when n = or n = 1,

and on the right-hand side only whenn=1

Ifxandyare distinct positive real numbers, then, by the weighted AM-GM Inequality,

xyyxď

2xy x+y

x+y

Since x2+xyy ă?xyă x+2y,we have

xyyx ď(xy)x+2y ď

x+y

2

x+y

(2)

The cases n=0 and n=1 can be treated directly If ně2, then

)ăFnăLn and (2)gives

FLnn LFnn ď(FnLn) Fn+Ln

2 ď

Fn+Ln

2

Fn+Ln

From FnLn = F2n and Ln = Fn´1 +Fn+1 and Fn +Ln = 2Fn+1 so

given inequality is proved

Problem 2.12. Let nbe a positive integer Prove that n

k=1

F2k Lk

! ă

n

ÿ

k=1

F3k L2 k

!

ě (Fn+2´1)

5

(Ln+2´3)2

where Fn, respectively Ln represent the nth Fibonacci number respectively the nth Lucas number. (Batinetu-Giurgiu)

Proof. Using the following well known identitiesL1+L2+ +Ln=

Ln+2´3 and F1+F2+ +Fn=Fn+2´1we have n

k=1

Lk

!3 ă

n

ÿ

k=1

F2 k

Lk

! n ÿ

k=1

F3 k L2 k ! ě n ÿ

k=1

Fk

(166)

Tạp chí onl ine của cộng đồng những người yêu T oán

It is enough to prove the above inequality Applying the Holder’s inequality, we get

n

ÿ

k=1

Lk

!3/5 ă

n

k=1

F2k Lk

!1/5 ă

n

k=1

F3k L2 k

!1/5

=

n

ÿ

k=1

L3k/5

!3/5 ă

n

ÿ

k=1

F2 k

Lk

1/5!5

1/5

ă

n

ÿ

k=1

F3 k

L2 k

1/5!5 

1/5

ě

n

ÿ

k=1

L1k/5ă F

2/5 k

L1k/5 ă F3k/5 L2k/5

! =

n

ÿ

k=1

Fk Hence our inequality is proved

Problem 2.13. Let nbe a positive integer Prove that

1

n

n

ÿ

k=1

1

n k

ăFk

!n

ěen´F2n

(Jose Luis Diaz-Barrero)

Proof. The Cauchy-Schwarz Inequality gives

n

ÿ

k=1

1

n k

Fk

! n ÿ

k=1

n k Fk ! ěn2

Using

n

ÿ

k=1

n k

Fk =F2n this identity, we obtain

n

ÿ

k=1

1 n k Fk ě n

F2n

The well known inequality ex ě 1+x, x P

R, with x = Fn2n ´1

implies

n F2n

ěe1´

(167)

Tạp chí onl ine của cộng đồng những người yêu T oán

Problem 2.14. Let n be a positive integer For any real number

γą1, show that

γ

n

ÿ

k=1

Fk2γLk2(1´γ)+ (γ´1)L2k

ěFnFn+1

(Jose Luis Diaz-Barrero)

Proof. Using Bernoulli’s inequality, we have

1+ F2 k L2 k ´1 γ γ1 ď F k L2 k

Therefore, we obtain the inequality

F2kγL2k(1´γ)+ (γ´1)L2k´γF2k ě0

Using this inequality and the identity n

ř

k=1

F2

k =FnFn+1,and

sum-ming up, we get desired inequality

Problem 2.15. Let ně3 be a positive integer Prove that

b 1´F12

n

+b 1´L12

n

ą c 1´

Fn+1

F2n

2

(P.G.Popescu)

Proof. If ně3,the Harmonic-quadratic mean inequality gives

2

1

b

F2n

+b

L2n

ă g f f e b 1´F12

n

2 +

b 1´L12

n

2

2 =

d 1´

2 F2 n + L2 n

Note that this inequality is strict since Fn ‰ Ln Indeed Ln =

Fn+1+Fn´1 ąFn for ně2.Now, we will show that

1 F2 n + L2 n ą F n+1

F2 2n

holds for n ě Now if we use F2n = FnLn and 2Fn+1 = Fn+Ln then last inequality is equivalent to

(168)

Tạp chí onl ine của cộng đồng những người yêu T oán

Problem 2.16. Let nbe a positive integer Prove that n

ÿ

k=1

F2 k

a 1+F2

k

! n ź

k=1

(1+F2n) !n1

ďFnFn+1

(Jose Luis Diaz Barrero)

Proof. Since

n

ř

k=1

F2

k =FnFn+1, the proposed inequality is

equiva-lent to

n

ÿ

k=1

F2 k

a 1+F2

k

! n ź

k=1

(1+F2n) !n1

ď

n

ÿ

k=1

F2k

This may be proved by the AM-GM Inequality and Chebyshev inequalities as follows

n

ÿ

k=1

F2k

a 1+F2

k

! n ź

k=1

(1+F2n) !n1

ď

n

ÿ

k=1

F2k

a 1+F2

k !     n ř

k=1

a 1+F2

k n     ď n ÿ

k=1

F2k

Since both sequences

"

F2

k

?

1+F2

k

*

and a1+F2 k

(

have the same monotonicity

Problem 2.17. Prove that

arctan c

F2

n+F2n+1

2 +arctan c

L2

n+L2n+1

2 ěarctan

Fn+2

2 +arctan

Ln+2

2

(D.M.Batinetu-Guirgiu, student)

Proof. Using Fn+Fn+1 =Fn+2 and AM-GM Inequality we get

Problem 2.18. Let n be a positive integer Prove that

3 a F2 n+ a F2 n+1

3

a

F2 n+2

! a L2 n+ a L2 n+1

3

a

L2 n+2

!

ă?3

(169)

Tạp chí onl ine của cộng đồng những người yêu T oán

Proof. Since the function f(x) = ?3x2 is concave for all x ě 0,

Jensen’s inequality yields

3

c

(x+y)2

4 ą

3

?

x2+?3 y2

2 ,

for xě0, yě0, x ‰y

Under the same conditions, this inequality can be equivalently written as

3

?

x2+a3

y2 ăa3

2(x+y)2.

For x=Fn and y=Fn+1, we obtain

3 a F2 n+ b F2 n+1 ă

3

a

2(Fn+Fn+1) =

b 2F2

n+2

This implies

3

a

F2 n+

a

F2 n+1

3

a

F2 n+3

ă ?

2 (1)

Similarly, lettingx =Ln and y=Ln+1 yields

3

a

L2 n+

a

L2 n+1

3

a

L2 n+2

(2)

Multiplying inequalities (1) and (2) yields the desired result

Problem 2.19. Let na positive integer Prove that

1

FnFn+1

"

n

n ÿ

k=1

F2kn+

n

ź

k=1

F2k

# ě

n

ź

k=1

F(k1´1/n)

!2

(Jose Luis Diaz-Barrero)

Proof. Using FnFn+1 =

n

ř

k=1

F2

k, the proposed inequality may be written equivalently as

(n´1)

n

ř

k=1

F2n k

n +

n

ź

k=1

F2k ě

n

ź

k=1

F2k

!(n´1/n) n ÿ

k=1

F2k,

(170)

Tạp chí onl ine của cộng đồng những người yêu T oán

Problem 2.20. Let nbe a positive integer Prove that

F2nă

1

2nF

nFn+1

Fn+2´1

+

2n n

Fn+2´1

2n

(Jose Luis Diaz-Barrero)

Proof. After AM-GM Inequality, it suffices to show that

F2n ă

d

2n n

FnFn+1

Using following well known identities

F2n = n

ÿ

k=0

n k

Fk,

2n n = n ÿ

k=0

n k

2 ,

FnFn+1 = n

ÿ

k=0

F2k

Therefore the desired inequality follows immediately from Cauchy-Schwarz Inequality

Problem 2.21. Prove that

(F1´

a

F1F2+F2)2+(F2´

a

F2F3+F3)2+ă ă ă+(Fn

a

FnF1+F1)2 FnFn+1

(1) (L1´

a

L1L2+L2)2+(L2´

a

L2L3+L3)2+ă ă ă+(Ln

a

LnL1+L1)2 LnLn+12

(2)

for any positive integer n (D M Batinetu-Giurgiu)

Proof. Using the AM-GM inequality, we obtain

x+y

2 ě

?

xy+ b

x2+y2

2

2 ,

and so

x´?xy+yě c

x2+y2

(171)

Tạp chí onl ine của cộng đồng những người yêu T oán

for any positive real numbers x and y Thus, the left-hand side of (1), LHS, is

LHSě F

2 1+F22

2 +

F2 2+F23

2 +ă ¨ ¨+

F2 n+F21

2 =

n

ÿ

k=1

F2k

=FnFn+1

Inequality (2) is proved in the same way by using n

ř

k=1

L2k =LnLn+1´

2

Problem 2.22. Let tanuně1 be a sequence of real numbers

de-fined by a1 = 3,a2 = and for all n ě 3,an+1 = 12(a2n+1) Prove that

1+

n

ÿ

k=1

Lk

? 1+ak

!2

ă LnLn+1

(Jose Luis Diaz-Barrero)

Proof. Since

n

ř

k=1

L2

k = LnLn+1 ´ 2, the proposed inequality is

equivalent to

1+

n

ÿ

k=1

Lk

? 1+ak

!2

ă1+1

n

ÿ

k=1

L2k

or

n

ÿ

k=1

Lk

? 1+ak

!2 ă

n

ÿ

k=1

L2 k

2

By the Cauchy-Schwarz inequality, n

ÿ

k=1

Lk

? 1+ak

!2 ď

n

ÿ

k=1

L2k

! n ÿ

k=1

1 1+ak

!

We now prove that

n

ÿ

k=1

1

ak+1

(172)

Tạp chí onl ine của cộng đồng những người yêu T oán We have

an+1 =

1 2(a

2

n+1)ñan+1´1=

1 2(a

2 n´1)

ñan+1´1=

1

2(an+1)(an´1) ñ

an+1´1

=

an´1

´

an+1

ñ

an+1

=

an´1

´

an+1´1

Hence n

ÿ

k=1

1 1+ak

=

n

ÿ

k=1

ak´1

´

ak+1´1

=

a1´1

´

an+1´1

ă

and the problem is done

Problem 2.23. Let Tk = k(k2+1) for all kě1 Prove that n

ÿ

k=1

F2m+2

Tm k

ě

m(F

nFn+1)m+1

nmTm n+1

for any positive integer n ě and for any positive real number

m

(D.M Batinetu-Giuriu)

Proof. Let S=Sn =

n

ř

k=1

F2

k, and U=Un= n

ř

k=1

Tk It is well-known that S=FnFn+1, and it is easily derived that

Un =

n(n+1)(n+2)

6 =

nTn+1

3

We see that we may write the desired inequality as follows:

Sm+1 ďUm

n

ÿ

k=1

F2k

F2 k Tk m (1)

Now Holder’s Inequality for sums states that for any two non-negative sequences takukě1 and tbkukě1, we have for all n ‰ 1,

and for all pą1 and qą1 such that p1 +q1 =1:

n

ÿ

k=1

akbk ď n

ÿ

k=1

apk

!1/p n ÿ

k=1

bqk

!1/q

(173)

Tạp chí onl ine của cộng đồng những người yêu T oán

In particular, take p =

m m+1,

1 q =

1

m+1, where m is any positive

number Also, take ak = T 1/p

k ,bk = F2k/T 1/p

k Then Holder’s In-equality takes the form:

n

ÿ

k=1

F2k ď

n

ÿ

k=1

Tk

!m/(m+1) n ÿ

k=1

F2km+2 Tm

k

!1/(m+1)

or

SďUm/(m+1)

n

ÿ

k=1

F2k

F2k Tk

m!1/(m+1)

Raising both sides to the power q = m +1 yields the desired inequality in (1)

Problem 2.24. Prove that

F2n+1´1

F2n+4´3Fn+2´Ln+2+3

ě

n

for all ně1 (Ovidiu Furdui)

Proof. Using induction, it is easy to show that for ně1,

n

ÿ

k=1

Fk =Fn+2 ´1, n

ÿ

k=1

Lk =Ln+2´3, n

ÿ

k=1

F2k =F2n+1´1

In addition, F2k = FkLk follows immediately from Binet’s formu-las Finally, recall that Chebyshev inequality asserts that

n

ÿ

k=1

akbk ě

1

n

n

ÿ

k=1

ak

! n ÿ

k=1

bk

!

for all nondecreasing positive sequencestakuandtbku Combin-ing these results, we find

F2n+1´1

F2n+4´3Fn+2´Ln+2+3

= F2n+1´1 (Fn+2´1)(Ln+2´3)

=

n

ř

k=1

FkLk

n ř

k=1

Fk n

ř

k=1

Lk

ě

n

(174)

Tạp chí onl ine của cộng đồng những người yêu T oán

Problem 2.25. Prove that if mą0and pą0, then n

ÿ

k=1

Fm+1

Lmk

! n ÿ

k=1

Fpk+1 Lpk

!

ě (Fn+2´1)

m+p+2

(Ln+2´3)m+p

for any positive integer n (D.M Batinetu-Giurgiu)

Proof. Using a special case of Holder’s inequality, for two

se-quences taku and tbku we have n

ÿ

k=1

akăbk n

k=1

auk

!1/u n ÿ

k=1

bvk

!1/v

(1)

where u1 + 1v = We let ak = (Lk)m/m+1 and bk = (Lk)Fkm/m+1; and

we let u1 = mm+1 and v1 = m1+1 Then n

k=1

akăbk = n

ÿ

k=1

Fk = (Fn+2´1) (2)

Then on the RHS of (1) we have n

ÿ

k=1

Lk

!m/m+1 n ÿ

k=1

Fmk+1 Lm

k

!1/m+1

(3)

Raising (2) and (3) to the (m+1) power and further notingřnk=1Lk =

Ln+2´3 we get

(Fn+2´1)m+1 ď(Ln+2´3)m n

ÿ

k=1

Fmk+1 Lm

k

!

(4)

But now , for another value, say o, we also get the relation

(Fn+2´1)p+1 ď(Ln+2´3)p n

ÿ

k=1

Fpk+1 Lpk

!

(5)

Multiplying (4) by (5) and dividing throungh by the Lucas sums we obtain the stated inequality This completes the proof

Problem 2.26. Let n be a positive integer Prove that n

ÿ

k=1

FkF2k

! n ÿ

k=1

F2 k

?

Lk

!

ěF3nF n+1

(175)

Tạp chí onl ine của cộng đồng những người yêu T oán

Proof. We will show that the indicated inequality is a special of

Holder’s Inequality: n

ÿ

k=1

akbk ď n

ÿ

k=1

(ak)p

!1/p n ÿ

k=1

(bk)q

!1/q

(1)

wherep,qthe ak’s andbk’s are positive numbers, with p1+q1 =1 Let p = 3, q = 3/2, ak = (Fk)2/3(Lk)1/3, Bk = (Fk)4/3(Lk)´1/3 We see that the indicated quantities satisfy the conditions required ef Holder’s Inequality Then , by (1), we have:

n

ÿ

k=1

(Fk)2 ď n

ÿ

1

(Fk)2Lk

!1/3 n ÿ

k=1

(Fk)2(Lk)´1/2

!2/3

Cubing both sides and noting that (Fk)2Lk =FkF2k,we obtain: n

ÿ

k=1

(Fk)2

!3 ď

n

ÿ

k=1

FkF2k n

ÿ

1

(Fk)2(Lk)´1/2

!2

(2)

Finally, we use the well-known identity: n

ÿ

k=1

(Fk)2 =FnFn+1 (3)

Substituting the result of (3) into (2) yields the desired inequal-ity

Problem 2.27. Let nbe a positive integer, Prove that

g f f e n ÿ

k=1

F2 k+1 ď

1 2n   g f f e n ÿ

k=1

(Fk+ (n2´1)Lk) + g f f e n ÿ

k=1

((n2´1)F

k+Lk)

(Jose Luis Diaz-Barrero)

Proof. Starting with Minkowski’s inequality

n

ÿ

k=1

(ak+bk)p

!1p

ď

n

ÿ

k=1

apk

!p1

+

n

ÿ

k=1

bpk

!1p

,

let

(176)

Tạp chí onl ine của cộng đồng những người yêu T oán

By adding these two quantities together we have

ak+bk =2nmFk+1

Using these quantities in the given inequality we have the result n

ÿ

k=1

Fpk+1

!p1 ď

2nm

n

ÿ

k=1

[Fk+ (nm´1)Lk]p

!p1 +

n

ÿ

k=1

[(nm´1)Fk+Lk]p

!1p 

If we let m =2 and p=2then we have the required result

Problem 2.28. Prove that

F2n nF2

n n+1

n2n´1 ďF 2n+1

1 +F

2n+1

2 +ă ă ă+F 2n+1

n

for all integers ně1 (Ovidiu Furdui)

Proof. The well known formula

F21 +F22+ă ă ¨+F2n=FnFn+1

will be used In view of the functionxÑx2n on(0,)we get that

F2

1+F22+ă ă ă+F2n

n

2n ď F

2n+1 +F2

n+1

2 +ă ă ă+F2 n+1

n

n

or equivalently

F2nnF2nn+1 n2n´1 ďF

2n+1

1 +F

2n+1

2 +ă ă ă+F 2n+1

n

We are done

Problem 2.29. Let nbe a nonnegative integer Prove that

1

n+1

n

ÿ

k=0

ln(1+Fk)

!2

ďFnFn+1

(177)

Tạp chí onl ine của cộng đồng những người yêu T oán

Proof. For xě0,it is well-known that xěln(x+1) Thus,

n

ÿ

k=0

Fk ě n

ÿ

k=0

ln(1+Fk),

and so

n

ÿ

k=0

Fk

!2 ě

n

ÿ

k=0

ln(1+Fk)

!2

Since n

ř

k=0

F2

k=FnFn+1,by the Cauchy-Schwarz inequality,

(n+1)(FnFn+1) = n

ÿ

k=0

1 ! n

ÿ

k=0

F2k

! ě

n

ÿ

k=0

Fk

!2 ě

n

ÿ

k=0

ln(1+Fk)

!2

It follows that

1

n+1

n

ÿ

k=0

ln(1+Fk)

!2

ďFnFn+1

Problem 2.30. Let lbe a positive integer greater than or equal to Show that, for x1,

logFl+1Fl+2ăăăFl+nxn2

n

ÿ

k=1

logFl+kx

(Juan Jose Egozcue)

Proof. It follows from Cauchy-Schwarz Inequality that

n2 =

n

ÿ

k=1

a

lnFl+kă

1 ?

lnFl+k

!2

n

ÿ

k=1

lnFl+k

! n ÿ

k=1

1 lnFl+k

!

For x1, we have lnx0 Hence

n2lnx

lnFl+1Fl+2ă ¨ ¨Fl+n

ď

n

ÿ

k=1

lnx

lnFl+k

,

which completes the proof, because lnx/lna = logax for any

(178)

Tạp chí onl ine của cộng đồng những người yêu T oán

Problem 2.31. Let nbe a positive integer prove that n

ÿ

k=1

L2 k

a 1+L2

k

! n ź

k=1

(1+L2k) !21n

ďLnLn+1 ´2

(Sergio Falcon)

Proof. Using AM-GM Inequality, we can see that

n

ÿ

k=1

L2 k

a 1+L2

k

! n ź

k=1

(1+L2k) !21n

ď n

ř

k=1 L2

k

?

1+L2

k n

ř

k=1

a 1+L2

k n Since " L2 k ?

1+L2

k

*

and a1+L2 k

(

are non-decreasing sequences,we conclude that

n ř

k=1 L2

k

?

1+L2

k n

ř

k=1

a 1+L2

k

n ď

n

ÿ

k=1

L2k

a 1+L2

k

b 1+L2

k = n

ÿ

k=1

L2k

Since n

ř

k=1

L2k =LnLn+1´2, the inequality follows

Problem 2.32. Prove that

a) F2n+Fn2+1+5F2n+2 ą4?6aFnFn+1Fn+2,

b) L2n+L2n+1+5L2n+2 ą4 ?

6aLnLn+1Ln+2

for any positive integer n (D.M Batinetu-Giurgiu)

Proof. a) From

F2n+4 = (Fn+2+Fn+3)2 = (2Fn+3Fn+1)2 = ((2Fn)2+(3Fn+1)2)+12FnFn+1

and the inequality of arithmetic and geometric means we have that

(179)

Tạp chí onl ine của cộng đồng những người yêu T oán

That is, 2?6FnFn+1 ďFn+4.Therefore,

4?6aFnFn+1Fn+2 ď2Fn+4Fn+2 =2(2Fn+2+Fn+1)Fn+2 =4F2n+2+2Fn+1Fn+2

This and the inequality of arithmetic and geometric mean imply

2Fn+1Fn+2 ďF2n+F n+1

So,

4?6aFnFn+1Fn+2 ď4F2n+2+2Fn+1Fn+2 ă5F2n+2+F2n+F2n+1

b) The proof of

4?6aLnLn+1Ln+2 ă5L2n+2+L n+L

2 n+1

is similar to the previous proof It is enough to replace Fn with

Ln

Problem 2.33. Let nbe a positive integer Prove that n

ÿ

k=1

Fk+2 ě

nn+1

(n+1)n n

ź

k=1

L

n+1

n k+1´F

n+1

n k+1

Lk+1´Fk+1

!

(Jose Luis Diaz-Barrero)

Proof. Direct computation shows that equality holds with n =

1 Now, suppose that n ě If a and b are real numbers such that bąaą0, then, by Holder’s Inequality,

żb

a

tn1 dtď

żb

a

t dt

n1 żb a

dt

n´1

n

or, equivalently,

n n+1 ă

bnn+1 an+n1

ba

a+b

2 n1

Taking here a = Fk+1, b = Lk+1, noting that Fk+1 +Lk+1 = 2Fk+2,

and taking the product overk=1, ,n gives

nn

(n+1)n n

ź

k=1

L

n+1

n k+1´F

n+1

n k+1

Lk+1´Fk+1

! ď

n

ź

k=1

Fk+2

!n1

(180)

Tạp chí onl ine của cộng đồng những người yêu T oán

Problem 2.34. Let nbe a positive integer Prove that

a) FFn+1

n +F

Fn+2

n+1 +F Fn n+2 ăF

Fn n +F

Fn+1

n+1 +F Fn+2

n+2

b) FFn+1

n F

Fn+2

n+1F Fn n+2 ăF

Fn n F

Fn+1

n+1F Fn+2

n+2

(Jose Luis Diaz-Barrero)

Proof. First we will prove first part Given inequality is true if

n=1, So we need to proveně3.Note that

FFnn +FFn+1

n+1 +F Fn+2

n+2 ´

FFn+1

n +F

Fn+2

n+1 +F Fn n+2

=h(FFnn +FFn+1

n+1)´(F Fn+1

n +F

Fn n+1)

i

+h(FFn+2

n+2 ´F Fn

n+2)´(F Fn+2

n+1 ´F Fn n+1)

i

Therefore, our statement will be established if we prove that, for

ně3,

FFn+1

n +F

Fn n+1 ăF

Fn n +F

Fn+1

n+1 (1)

and

FFn+1

n+1 +F Fn n+1 ăF

Fn n +F

Fn+1

n+1 (2)

hold

In fact, we consider the integral

I1 = Fn+1

ż

Fn

Fxn+1lnFn+1´FxnlnFn

dx

Since Fn ăFn+1 ifně3,then, for FnďxďFn+1, we have

FxnlnFn ăFxn+1lnFn ăFxn+1lnFn+1

so we get I1 ą0

On the other hand, evaluating the integral, we obtain

I1 = Fn+1

ż

Fn

Fxn+1lnFn+1´FxnlnFn

dx= Fxn+1 ´Fxnˇˇ

Fn+1

Fn

=FFnn +FFn+1

n+1 ´ F Fn+1

n +F

Fn n+1

and (1) is proved

To prove (2), we consider the integral

I2 = Fn+2

ż

Fn

Fxn+1lnFn+1´Fxn+1lnFn+1

(181)

Tạp chí onl ine của cộng đồng những người yêu T oán

Since Fn+1 ăFn+2,then, for FnďxďFn+2, we have

Fxn+1lnFn+1 ăFxn+2lnFn+2

and I2

On the other hand, evaluating I2, we obtain

I2 = Fn+2

ż

Fn

Fxn+2lnFn+2´Fxn+1lnFn+1

dx= Fxn+2´Fxn+1ˇˇ

Fn+2

Fn

=

FFn+2

n+2 ´F Fn n+2

´

FFn+2

n+1 ´F Fn n+1

This completes the proof of first part

Now we will prove second part of our statement The proof will be done in two steps First, we will prove that

FFn+1

n F

Fn+2

n+1F Fn n+2 ą

Fn+Fn+1+Fn+2

3

Fn+Fn+1+Fn+2

(3)

Using weighted AM-GM Inequality we get

FFn+1

n F

Fn+2

n+1F Fn n+2 ă

FnFn+1+Fn+1Fn+2+Fn+2Fn

Fn+Fn+1+Fn+2

Fn+Fn+1+Fn+2

Inequality (3) will be established if we prove that

FnFn+1+Fn+1Fn+2+Fn+2Fn

Fn+Fn+1+Fn+2

Fn+Fn+1+Fn+2

ă

Fn+Fn+1+Fn+2

3

Fn+Fn+1+Fn+2

or, equivalently,

(Fn+Fn+1+Fn+2)2 ě3(FnFn+1+Fn+1Fn+2+Fn+2Fn) which is AM-GM Inequality

Finally, we will prove that

Fn+Fn+1+Fn+2

3

Fn+Fn+1+Fn+2

ăFFnn FFn+1

n+1F Fn+2

n+2

which is weighted HM-GM Inequality

(182)

Tạp

chí

onl

ine

của

cộng

đồng

những

người

yêu

T

oán

References

[1] P.G Popescu , J.L Diaz-Barrero, Certain Inequalities for Convex Functions Journal of Inequalities in Pure and Applied Mathematics, Vol 7, Issue 2, Article 41, 2006

(183)

T p c h í

online

cộng đồng

những n g i y ê u T o n

Tạp

chí

onl

ine

của

cộng

đồng

những

người

u

T

ốn TỐN HỌC TRONG MẮT AI

Nguyễn Quốc Khánh (Hà Nội)

Năm 1982, đạo diễn Trần Văn Thủy làm phim tài liệu, đặt tên “Hà Nội mắt ai", phim hay Bộ phim mở đầu tiếng đàn guitar tinh tế truyền cảm nghệ sĩ Văn Vượng, người sinh lớn lên Hà Nội

Nghệ sĩ Văn Vượng

Nghệ sĩ Văn Vượng vốn mong muốn lần tận mắt chứng kiến vẻ đẹp Hà Nội Làm thế, có lẽ ý đồ đạo diễn muốn mượn giới quan khiết nghệ sĩ mù lắng nghe tiếng động sống xung quanh Trong toán học có điều tương tự, đơi ngồi lại, thật bình tâm yên lặng để quan sát cảm thụ lấy giới quan bạn bè mình, tức điều mà thường bị phần tính, giống hư ảnh nhiều màu, lấn át

(184)

Tạp

chí

onl

ine

của

cộng

đồng

những

người

yêu

T

oán

các bạn đọc cảm thụ đó, mà bắt nguồn từ chuyện chia sẻ nhiều giới quan toán học người vậy,

Nhưng nói mục đích tơi bị nhiều trừu tượng q, chi lựa chọn lấy mục đích rõ ràng hơn, cụ thể trực diện hơn, chẳng hạn làm để người trở nên thích đọc sách? Tất nhiên, câu hỏi khó lắm, tồn ngành xuất cịn đau đầu để tìm câu trả lời mà chưa Rất may, NXB Kim Đồng nhận thức tầm quan trọng câu hỏi này, nên gần họ xuất sách có tựa đề lạ, “Bí kíp khiến bạn thích đọc sách” Cuốn sách dành cho “ngay với bạn khơng thích sách” Có lẽ với tựa dễ thương thế, sách này, với nhân vật

Mọt sách, hợp với bạn độc giả Epsilon, mà Ban biên tập vốn tự xác định, người yêu toán, tức Mọt toán, yêu toán, người yêu người u tốn Tơi hi vọng rằng, tương lai gần thơi, có sách đó, mà nhân vật đặt tên Mọt toán, chủ đề sách ấy, có lẽ “Bí kíp khiến bạn khơng cịn ghét sợ tốn”

Các bạn độc giả thông cảm cho tật ăn nói dài dịng tơi Tơi buộc phải làm để sau bạn bớt chê trách hơn, mà lại lựa chọn sách mà có lẽ số bạn có, đọc qua Cuốn sách thực lập kỉ lục làng sách Việt Nam, mà in lần đầu có năm, tới sách in in lại tới gần chục lần Với hàng vạn bán hết năm, tiểu thuyết

(185)

Tạp

chí

onl

ine

của

cộng

đồng

những

người

yêu

T

oán

Tới đây, tơi bắt đầu nghe thấy có tiếng thở dài đâu đó, có bạn nói thầm “Ơi tưởng giới thiệu sách gì, hóa sách này, sách tiếng cịn giới thiệu làm cho tốn giấy tốn mực” Ừ nhỉ, mà thực có bạn hỏi điều tương tự này, phải cảm ơn bạn, bạn đặt câu hỏi đúng! Tại lại câu hỏi đúng? Và rốt lại chọn giới thiệu

cuốn sách thừa phổ biến đây? Tơi nói lý

(186)

Tạp

chí

onl

ine

của

cộng

đồng

những

người

yêu

T

oán

chuyến phiêu lưu nhiều nhân vật tiểu thuyết huyền ảo này, nữ họa sĩ Thái Mỹ Phương Đã có lúc tơi ngạc nhiên mà Thái Mỹ Phương vẽ nhân vật xác với mường tượng tác giả tới thế, tất nhiên việc họa sĩ phải học nhiều tí tốn để có hình minh họa ý tưởng tốn học khó, nghịch lý Zermelo chẳng hạn, thần nhân vật có lẽ điều khó khăn công việc minh họa cho sách “đẹp” Và thế, bạn có thấy không, với danh tiếng hai tác giả, với minh họa xuất sắc Thái Mỹ Phương, việc “Ai Ky” bán chạy điều khó hiểu Vậy điều khó hiểu đây? Xin bạn đừng giật nghe tơi nói điều nhé, điều đặc biệt đây, lý mà tơi muốn giới thiệu sách tới với bạn, thực là một sách không hay!

Tới đây, xin bạn độc giả bình tĩnh! Vì tơi không mở tranh luận việc sách hay Epsilon Thay vào đó, tơi kể cho bạn nghe câu chuyện khác, hai tác giả sách Có lần, Giáo sư Ngơ Bảo Châu có hỏi tơi “Tại anh dành nhiều thời gian để viết số chuyên đề sơ cấp, mà khơng đọc nhỉ?” Cái chuyên đề mà Giáo sư Châu nhắc tới chuyên đề “Về việc giải đa thức ẩn” đăng Epsilon số Chuyên đề đăng

trang nhà Facebook Giáo sư thời gian trước, người chia sẻ đọc Nếu bạn tinh ý chút, nhận rằng, chuyên đề mười trang thực chất nội dung với toàn sách “Ai Ky” dày tới trăm trang, tất nhiên chuyên đề chứa đựng số nội dung cao cấp

(187)

Tạp

chí

onl

ine

của

cộng

đồng

những

người

yêu

T

oán

Vườn ươm tài Talinpa Tuần Châu, Hạ Long ngày đầu năm nay, câu chuyện tốn học yếu mà Giáo sư sử dụng để nói về, chủ đề giải phương trình đại số hai lần trước Tại lại phải lặp lặp lại chủ đề thế, lần lại định dạng khác nhau, lúc sách, lúc chun đề, lúc nói chuyện, lần lại có khác điển hình?

Trả lời câu hỏi khơng phải dễ Nhưng muốn bạn tạm thời quên “Ai Ky” để ngắm nghía tranh đặc biệt, tranh Bạn thấy nào? Tôi kể cho bạn nghe lai lịch tranh Bức tranh cô bé tên Mirella vẽ Mirella gái nhà toán học Nhà toán học lúc nhà thường hay ngồi chơi học toán gái Nhà toán học tức Giáo sư Nguyễn Tiến Zũng, người tự ví “con cừu đen của làng Tốn Việt Nam”

(188)

Tạp

chí

onl

ine

của

cộng

đồng

những

người

u

T

ốn

Hơm nay, ơng chủ khu vườn lại dẫn vào khu vườn khác, với niềm say mê Không ta ông chủ cho xem, chiêu đãi hoa thơm khu vườn, mà cịn tận tình bảo cách tạo nên hoa thơm Khu vườn có tên Tốn học

(Giáo sư Hà Huy Khoái giới thiệu sách "Bài giảng cho Mirella" Giáo sư Nguyễn Tiến Zũng)

Các giảng toán cho Mirella thực sách giáo khoa toán học cho tất người, đặc biệt cho muốn tìm hiểu vẻ đẹp tốn học mà cịn ngại tính tốn! Nói cho cùng, tốn học có hai phần “tính” “tốn” Nếu kỳ thi thường hay bắt thí sinh phải thạo “tính”, tác giả lại cho người đọc hiểu phần “toán”, tức phần chất toán học Hơn nữa, hiểu “tốn” việc “tính” tự nhiên trồng cây, gieo hạt giống Đã đến lúc người làm vườn cô bé Mirella bước vào khu vườn Toán học, với niềm vui người khám phá sáng tạo

(Giáo sư Hà Huy Khoái giới thiệu sách “Bài giảng cho Mirella”)

Hãy thử xem mục lục giảng mà Giáo sư Zũng giảng cho Mirella Đó tốn cơng chúa Dido khoanh đất ven bờ biển, tốn khỉ bán chuối, tốn bị gặm cỏ, toán đàn kiến dây, toán cắt ghép hình vng, tốn bánh xe hình vng, nhiều nhiều Có thể thấy, tất toán riêng lẻ, cách mà Giáo sư Zũng vô tỉ mỉ để giảng cho Mirella cách tiếp cận vấn đề cụ thể bước, bước, quan sát, suy luận, thực hiện, thí nghiệm, đạt tới hiểu biết cảm thụ tốn học thực thụ Vậy thật cách mà Giáo sư Khối ví von, cơng việc Giáo sư Zũng thực giống người làm vườn với tất say mê Và điều thực tốt Bởi dạy cho người khác yêu thích hay đam mê toán học, ngoại trừ cách thể cho người nghe bạn niềm đam mê thực bạn, để thơng qua cảm hứng mà bạn có gắng thể truyền tải cho họ, bạn cịn làm khác hơn?

(189)

Tạp

chí

onl

ine

của

cộng

đồng

những

người

yêu

T

ốn

cũng có nhiều bạn khơng Điều khơng đáng ngạc nhiên, có lẽ bên cạnh bạn thích làm vườn, ln cịn có bạn thích bơi biển, chẳng hạn chuyện Hyppasus tìm tồn số vô tỷ mà bị Pythagoras bắt phạt bơi tiếng BIỂN giống “Ai Ky” kể lại

(190)

Tạp

chí

onl

ine

của

cộng

đồng

những

người

yêu

T

oán

sức “rung động đến rùng mình” đọc cách “giảng tốn” phép ví von đến táo tợn này: “tên khai tẩu thoát khỏi nhà kho chuyên chứa tất loại toán tử, trở thành đối tượng bị truy nã khắp nơi, khơng tóm trở về, hắn, tức dấu khai căn, chạy khắp nơi khai trái phép tất người, mà làm thế, lúc cư dân xứ sở tí hon lùn tịt mãi” Hoặc này: “cả thành phố trở nên náo loạn, tất cậu bé số nghịch ngợm “thuổng” dấu nhân chạy đâu Thị trưởng định thành phố rơi vào tình trạng khẩn cấp, tử vong, cậu bé số mà dung dấu nhân chạm vào người biến Như buộc phải mời bác khổng lồ tới, bác khổng lồ tức số lớn vô cùng, có bác họa may trị cậu bé số 0, có bác tiêm chủng phịng bệnh số 0” Thật tuyệt!

(191)

Tạp

chí

onl

ine

của

cộng

đồng

những

người

yêu

T

ốn

Thành phố rơi vào tình trạng “giới nghiêm” cậu bé số “thuổng” dấu nhân chạy biến đâu

(192)

Tạp

chí

onl

ine

của

cộng

đồng

những

người

yêu

T

oán

cho dù bước vào phịng, thiếu bước đó, mà bạn khơng thể biết phịng có Cái kì ảo phịng nhận thức có lẽ vĩnh viễn chủ đề cốt lõi mà dành quan tâm thật lớn Ngày nay, nhiều học thuyết giáo dục đem nghiên cứu ứng dụng tràn lan, đơi khi, những điều mà Levshin sách “Ba ngày nước tí hon” đề cập đến dễ bị quên đi, đơn giản?

Bây bạn trở lại với chủ đề ban đầu mà tạm trì hỗn từ giờ, chủ đề “Ai Ky” lại sách hay, Giáo sư Ngô Bảo Châu lại liên tục thể nghiệm cách diễn đạt cho chủ đề tính giải phương trình đại số ẩn? Để làm việc đó, trước tiên, thử nghĩ niềm vui toán học tác giả sách trên, nói cách khác, thử cảm nhận giới quan toán học họ

(193)

Tạp

chí

onl

ine

của

cộng

đồng

những

người

yêu

T

oán

Giáo sư Nguyễn Tiến Zũng – cừu đen làng toán Việt Nam – dịch giả “Ba ngày nước tí hon” – tác giả “Các giảng toán cho Mirella”

Đối với sách Lev-shin, hẳn tác giả (cùng với vợ nhà văn Emilia Aleksandrove) phải có yêu thương trẻ tới viết giới dễ thương Trong giới ấy, bạn dễ dàng nhận thấy tất thứ dành cho tôn trọng nâng niu lớn Cái giới quan tốn học Levshin thực giới quan khiết trẻ thơ, đặt tảng triết lý người lao động khoa học thực thụ trải qua nhiều tháng năm “trận mạc”, nghĩa Q.S.T.K

Nhân nói tới “trận mạc”, tơi thấy muốn chia sẻ với bạn sách mà tơi đọc từ thuở vỡ lịng, nay, có lẽ số nơi mà tơi tìm cảm thấy hoang mang với sống xung quanh, sách“Những tấm lòng cao cả” nhà văn người Ý tên

(194)

Tạp

chí

onl

ine

của

cộng

đồng

những

người

yêu

T

oán

cuốn sách quân nhân, sau ngày giải ngũ, ông xách ba lô lên qua khắp chiến trường châu Âu, để hiểu chiến tranh, điều mà ông vừa kết thúc Với ý tưởng sau có tư liệu, viết điều chiến tranh, qua khắp vùng đất ấy, chứng kiến tất số phận người xứ ấy, Amicis lại nhận thấy nhân cách đầu đời trẻ chuyện tiên để có xã hội văn minh hơn, bớt khổ đau Hôm nhắc chuyện Amicis, cách để hiểu giới quan tốn học nói riêng giá trị sách kỳ diệu Levshin nói chung

Ở bên trên, bạn thấy ảnh đại diện Giáo sư Nguyễn Tiến Zũng, người mà gần lúc gặp gỡ, thấy niềm vui tiếng cười rạng rỡ Tôi không hiểu mà Giáo sư Zũng làm tốn vui vẻ suốt ngày Nhưng có lẽ nhờ việc ln tươi cười vậy, mà Giáo sư Zũng thực người trẻ trung toàn thể bạn bè đồng trang lứa Tin tốt dịng chữ giảng cho Mirella, bạn đọc cảm nhận niềm vui vậy, đó, thực sách hay dành cho tất người, không học sinh, mà phụ huynh em Tưởng tượng giới quan tốn học Giáo sư Zũng, có lẽ ta thấy giống người trẻ, dọc bờ biển, lắng nghe sóng biển, ngắm nhìn cánh chim chao lượn trời, cảnh hồng biển, lại cúi xuống mà mân mê chơi đùa với mảnh sò đủ màu sắc tươi vui Cái giới quan toán học ấy, rõ ràng giới diệu kỳ tuổi bé thơ khiết ngày nô đùa với số, hình họa, ý tưởng triền miên không dứt, tất nhiên niềm vui tiếng cười giòn giã

(195)

Tạp

chí

onl

ine

của

cộng

đồng

những

người

yêu

T

oán

Bước thứ 4, bước cuối sơ đồ Pólya “nhìn lại vấn đề” Có thể số người, bước nhàm chán, tơi lại bước đem lại nhiều “khoái cảm” làm toán Nếu ba bước đầu tiên, bạn phải chịu đựng nhiều ức chế, có nhiều áp lực, tới đây, bạn biết giải toán rồi, bạn muốn viết lại lời giải theo cách hay nhất, đẹp

Không nên bỏ qua bước Thứ bước thú vị, bạn có thời gian để ngồi lại “tỉa tót” lời giải Nhiều q trình tìm tịi thực kịch bản, bạn làm nhiều điều phức tạp, rắc rối, bạn phải đi lại lại nhiều lần bước thừa, nữa, chưa biết đường, bạn phải qua nhiều đường vịng, lúc này, bạn loại bỏ tất thừa đó, lọc bỏ thừa, bạn tìm đường ngắn để đến gần với chân lý tốn học

Giáo sư Ngơ Bảo Châu – Vườn ươm tài Talinpa, Quảng Ninh 2015

Tôi muốn nhấn mạnh vào chuyện Ngay đây, bạn thấy giới quan toán học Levshin cho ví von, trực giác tuyệt mĩ khái niệm toán học nhất, dựa trân trọng, lòng nhân ái, triết lý lao động khoa học chân chính; cịn giới tốn học Giáo sư Zũng lại bờ biển dài lộng gió, với thật nhiều cảnh tượng đẹp đẽ; giới quan tốn học Giáo sư Châu dường lại tự nhiên chứa đựng nhiều dấu ấn nghệ thuật, ngôn ngữ, triết học xung quanh Chính khác biệt ba giới quan khiến cho sách Levshin nhiều người đọc yêu thích, sách Giáo sư Zũng người đọc chút, người lại đọc dễ dàng, sách “Ai Ky”, best-seller thực sự, số người đọc được, cảm có lẽ khơng nhiều

(196)

Tạp

chí

onl

ine

của

cộng

đồng

những

người

yêu

T

oán

(197)

Tạp

chí

onl

ine

của

cộng

đồng

những

người

u

T

ốn

Tơi nghĩ, giới quan tốn học có xu hướng thưởng thức môn nghệ thuật, môn triết học, môn ngôn ngữ học khiến “Ai Ky” trở nên kén người cảm thụ Nhưng dù

sách thật đẹp, chẳng may có bạn mà cảm thụ giới quan toán học Giáo sư Châu, sung sướng Giáo sư nâng lên ly rượu vang thật ngon lành, bữa tối đầm ấm người thân quen

Trong bữa tối vui vẻ vậy, có lẽ bạn hỏi tơi rằng, điểm sách “Bài giảng cho Mirella”, mà lại không đăng ảnh bìa? Thế tơi mìm cười mà xin bạn bỏ qua thiếu sót ấy, bù lại, tơi vui lịng tặng cho

bạn cuốn“Đối thoại toán học”như Mong bạn thích sách này, bỏ q cho thiếu sót nhỏ tơi ,

inclusion-exclusion Franc¸ois Édouard Anatole Lucas bài toán tháp Hà Nội) cụng thc nghch oMă hm Măobius bng Măobius face lattice (topological ordering “phương phápchiều” Wikipedia RichardWesley Hamming đây. Tanya Khovanova. Stan Wagon). http://www.artofproblemsolving.com/community/c6h312083 http://www.artofproblemsolving.com/community/c6h355795p1932940 http://www.artofproblemsolving.com/community/c6h386167p2144390 https://nguyenvanlinh.wordpress.com/2015/03/28/vietnam-tst-2015-problem-5/ http://www.artofproblemsolving.com/community/c6h247411 (http: http://faculty.evansville.edu/ck6/encyclopedia/ETC.html http://bernard.gibert.pagesperso-orange.fr/ctc.html https://cms.math.ca/crux/ http://www.maa.org http://forumgeom.fau.edu/ http://www.cut-the-knot.org/pythagoras/CircleCircleEllipse.shtml http://artofproblemsolving.com,January16,2014. http://faculty.evansville.edu/ck6/tcenters/recent/hofstad.html.

Ngày đăng: 08/02/2021, 07:47

Tài liệu cùng người dùng

  • Đang cập nhật ...

Tài liệu liên quan